Fin 320-00 Fall 2022 Online Package R PageN
Fin 320-00 Fall 2022 Online Package R PageN
Online
Course Materials
Dr. Terrance Jalbert
1
Exclusive Use Agreement
The materials provided in this package of materials are for your exclusive use only. You
may not re-distribute materials contained in this package in any form.
2
Course Syllabus
3
Course Syllabus
Principles of Finance
Fin 320-002 CRN: 10109
University of Hawaii at Hilo
Fall Semester 2022
Please feel free to e-mail or phone me at home. If I am not home, please leave a message
and I will return your call.
Zoom Codes
Given the ever-changing nature of our current environment, I reserve the right to modify
this syllabus throughout the semester as I determine appropriate.
Class Etiquette
When attending Zoom classes, please turn on your camera. When attending
Zoom classes, clothing is required. Please wear appropriate attire. Appropriate
attire requires, at minimum, a shirt and shorts. Also, please be aware of what
might be in your surroundings and displaying on your camera.
When in close proximity to me, such as visiting during my office hours, I request
that you were a mask regardless of university policy. If you are not able or
willing to wear a mask, I will be happy to arrange a Zoom meeting with you.
4
Catalog Description: Introduction to concepts and techniques of business finance.
Topics include organizational forms, agency relationships, financial analysis and
planning, the capital formation process and capital markets, risk and return, time value of
money, stock and bond valuation, and capital budgeting.
Prerequisite Courses: ACC 201 or ACC 250
Calendar
August 22, First Day of Class
August 30, Last Day to Register or Add a Class
September 5 (Monday) Labor Day Holiday
September 13: Last Day to withdraw from a course without a grade of W
October 31: Last day to withdraw from a course with a grade of W
November 8: (Tuesday) Election Day, No Class
November 11 (Friday) Veteran’s Day, No Class
November 24, (Thursday) Thanksgiving Day Holiday
December 8: Last Day of Regular Classes
December 8: Last day to withdraw from the university with grades of W
Midterm Exam: I will announce the midterm exam date as the course progresses. I will
give you at least two weeks notice prior to any exam.
Third Exam: Your third exam will be Thursday, December 15, 4:10 PM -6:10 PM: We
will use our regular class time zoom code for the exam.
5
J. Assess the value of potential investments in projects, and make correct
decisions on weather or not to proceed with the investment.
Course Content
A. Course Topics:
1. Organizational Forms, corporate governance, taxation and implications for
dividend policy and capital structure.
2. Agency relationships and solutions to agency problems
3. Financial Statements, Cash Flow, and Taxes
4. Financial Statement Analysis
5. The Financial Environment: Markets, Institutions and Interest Rates
6. Risk and Return
7. Time Value of Money
8. Bonds and their Valuation
9. Stocks and their Valuation
10. Cash Flow Estimation
11. The Basics of Capital Budgeting
12. The Cost of Capital
B. Additional Topics to be covered should time permit
13. Derivative Securities
14. Capital Structure
15. Dividend Policy
Course Materials:
Required: One of two Texas Instruments Calculators (purchase one or the other, not
both)
6
It is critical that you get access to a calculator early in the semester! It is
impossible to successfully complete the course without the required
calculator. Do not wait until the last minute to purchase your calculator as
the demand for these calculators exceeds supply in Hilo during peak
purchasing times.
You must have access to the correct calculator to complete the course. You
will receive a failing grade for the course if you do not have the correct
calculator by the end of the 4th week of class.
If you can not afford to buy a calculator, you may be able to borrow one from a
friend. I have been requiring these calculators for some years at UH-Hilo, so
most UHH business students have one. If neither are possible, please let me
know and we will find a solution for you.
Required: For exams, if you are not in the classroom with me when taking the
exam, you must have a detachable webcam for your computer. The internal camera
on a laptop computer will not be sufficient.
Required: I will email you a package of materials for the course. The materials contain
class overheads, special readings, homework assignments and solutions to the homework
assignments. You should have the materials available for class each day. I strongly
encourage you to print the package. In my experience students are able to take
proper notes better, learn better, and are more successful on exams if they have the
printed package.
Teaching Methods:
The primary teaching style for this course is class lectures and outside-of-class
homework.
Evaluation Tools:
Homework:
Homework will be assigned on a regular basis. You are expected to hand in your
homework at the assigned time. However, I will accept late homework with a grading
penalty.
7
Attendance Quizzes:
I believe that you learn by attending and participating in the class. Thus, attendance is
required and expected. I expect you to arrive to the classroom at least a minute or two
prior to the class starting time. You are also expected to remain in the class for the entire
class period. Unlike some professors, my lecture is disrupted by late entrants, early
departures or leaving and re-entering of the class. As such, these activities should occur
only RARELY! If you enter class late, please enter quietly to minimize the disruption!!
There will be daily attendance quizzes. I will drop your lowest quiz score when I
compute your final grade. Please sit in the same spot each day to facilitate conducting
the quiz quickly. The goal of the quizzes is for you to be motivated to arrive to class on
time, participate positively and to remain in class for the entire class period.
You will receive credit for one quiz each day you are in class. If you arrive late, depart
early, or exit and return, you will receive ½ credit for the day. If you arrive late to class,
you are responsible to note to me after class that you entered the class to earn the ½ quiz
credit. If you miss a combined total of more than 9 classes, 18 tardy days (or any
combination of the two) regardless of reason, your grade will be reduced by an additional
1/3 letter grade.
Note that for virtual classes, turning on your camera and allowing me to see you assures
me that you are participating in class. If you do not turn your camera on, I use your
responses (or lack thereof) to my questions to determine if you are participating in class.
Each homework assignment and quiz receives an equal weight in determining your
overall homework score. However, due to the longer nature of the Chapter 8 homework,
it receives the weighting of two homework assignments.
Extra Credit: There will not be any extra credit work available in this course.
Group Work: You are encouraged to work on the homework and to otherwise study in
groups. However, you are responsible for the material individually on exams.
Course Grading:
Exam I 100 points
Exam II 100 points
Third Exam (partially comprehensive) 125 points
Homework and quizzes 25 points
Total 350 points
* I will give you at least two weeks notice on the exact dates of the midterm exams.
To earn an A in the course, you need to earn 318.5 points (91%) or more
To earn an A- in the course, you need to earn 311.5 points (89%) or more
To earn a B+ in the course, you need to earn 304.5 points (87%) or more
8
To earn a B in the course, you need to earn 288.75 points (82.5%) or more
To earn a B- in the course, you need to earn 276.5 points (79 %) or more
To earn a C+ in the course, you need to earn 269.5 points (77%) or more
To earn a C in the course, you need to earn 245 points (70%) or more
To earn a C- in the course, you need to earn 234.5 points (67%) or more
To earn a D in the course, you need to earn 217 points (62%) or more
Behavior Policy
You are expected to behave in a manner that is consistent with the UHH Student Code of
Conduct. You are responsible for making yourself aware of the responsibilities specified
for you in this document. The document can be accessed at:
http://hilo.hawaii.edu/studentaffairs/conduct/student_conduct.php
I specifically wish to call your attention to the following activities that are not allowed in
class. If I see you doing any of the following, you will lose your attendance points for the
day. I do not announce these penalties in class but, when applied, you will see them
reflected in your final grade. Repeat violators are subject to additional grade reductions
at my discretion.
You are disruptive to the class in any way
You are not engaged in the class
Using your computer
Use of headphones
Text messaging or otherwise using your telephone
Working on assignments, or studying for another class
Carrying on a side conversation with someone else in class. If you have
something relevant to the class to comment on, it is most welcome, but
please share with us all.
Sleeping
Determining if you have violated any of these rules will be made at the professor’s sole
discretion and may involve elements of subjective evaluation.
For in-person students, you must bring your computer to class on the exam day. You
must have internet access to take the exam.
For those taking the exam in a virtual session. You must have a detachable webcam for
the exams. You must turn on the camera and display the video through Zoom when you
are taking the exam. The internal camera on a laptop computer is not sufficient. I
reserve the right to ask you to display your work area with your camera when taking
exams including your cell phone, tablet, computer screen, workspace and your
9
surroundings. I will expect you to do this IMMEDIATELY upon my request with no
delay. I reserve the right to ask you to place your communication devices in a secure
location for exams.
Academic Dishonesty:
Do not cheat. The student conduct code and other university documents specify
significant penalties for cheating. These penalties will be enforced if I find you have
cheated in any way.
You should specifically note that I consider possessing or using exams from prior
semesters, in any form, an act of cheating.
I do not distribute my exams. Leaving the testing room with a copy of an exam, or
portion of an exam, in any form, is an act of cheating.
RECORDING POLICY
Students may record using various media during class for their own study purposes
provided they have obtained permission from the instructor prior to recording in class.
The recordings may not be shared and may not be posted on-line, e.g., YouTube, or on
other social media. You must delete all copies of the recordings at the end of the
semester.
COMPUTER SUPPORT
The UH System Information Technology Services (ITS) Academic Technologies section
provides technical support via email and telephone. Contact the ITS Help Desk for
support by calling toll free (800) 558-2669 or by sending an email to [email protected].
For more information on available services please see the ITS website at
http://www.hawaii.edu/askus/588.
DISABILITY SERVICES
Any student with a documented disability who would like to request accommodations
should contact the Disability Services Office - Student Services Center E230, 932-7623
(V), 932-7002 (TTY), [email protected] - as early in the semester as possible.
ACADEMIC ADVISING
Advising is an important resource designed to help students navigate the University and
the requirements of their individual majors. Students should consult with their advisor at
least once a semester to decide on courses, check progress toward graduation, and discuss
10
career options and other educational opportunities offered at UH Hilo. Advising is a
shared responsibility, but students have final responsibility for meeting degree
requirements.
ACADEMIC INTEGRITY
Intellectual development requires honesty, responsibility, and doing your own work. All
UH Hilo students are expected to know and abide by the Student Conduct Code
(https://hilo.hawaii.edu/studentaffairs/conduct/student_conduct.php). Acts of dishonesty,
for example, plagiarism (taking thoughts/ideas or words from others), cheating, collusion,
or other forms of academic dishonesty will possibly result in penalties, consequences,
and/or other disciplinary actions. If you have any doubts or questions about what
constitutes academic misconduct, please do not hesitate to contact me. For information
about your student rights, contact Karishma Kamath, Director of Student Conduct, 932-
7472, [email protected].
ACADEMIC SUCCESS
Kilohana: The Academic Success Center provides a range of free academic services and
resources to all currently enrolled UH Hilo students. Services include access to peer
student staff from a range of academic majors and course-related pdf resources.
Until further notice all tutorial sessions will take place over video.
You can schedule a session with one of our tutors by emailing [email protected].
Please state the course alpha and number (e.g. MATH 140) in the subject line so staff
know what you need assistance with. Kilohana is scheduled to begin service Monday,
August 31, 2020.
11
For immediate help, contact The Crisis Line of Hawaii 1-800-753-6879, the National
Suicide Prevention Hotline 1-800-273-8255 (suicidepreventionlifeline.org), or text
“Aloha” or “Hello” to the Crisis Text Line 741-741.
STUDENT CONDUCT
All members of UH Hilo have the right to pursue educational endeavors as part of the
institution's fundamental purposes of teaching, learning, and research. Students have the
responsibility to maintain standards of personal integrity (honesty, civility, and respect)
that are in harmony with the educational mission of the University; to respect the rights,
privileges, and property of others; and to observe University policies as well as national,
state, and local laws. The student conduct process holds students accountable for
violations of university policy and all UH Hilo students are expected to know and abide
by the Student Conduct Code
(https://hilo.hawaii.edu/studentaffairs/conduct/student_conduct.php).
STUDENTS OF CONCERN
University life should be challenging, not overwhelming. There may be times when
students experience severe stress or distress from adjusting to college life and adapting to
unforeseen changes/circumstances. Consider referring a friend, fellow classmate, or
yourself to the UH Hilo CARE Team who can provide information, resources, and
options for successfully navigating a challenging academic life. Typical referrals include
behavior which raise significant concerns and reasonably suggests that the behavior,
physical and/or emotional state:
TITLE IX SYLLABUS
The University of Hawaii is committed to providing a learning, working and living
environment that promotes personal integrity, civility, and mutual respect and is free of
all forms of sex discrimination and gender-based violence, including sexual assault,
sexual harassment, gender-based harassment, domestic violence, dating violence, and
stalking. If you or someone you know is experiencing any of these, the University has
staff and resources on your campus to support and assist you. Staff can also direct you to
resources that are in the community. Here are some of your options:
12
UH Hilo Counseling Services: SSC, room E-203, 932-7465
UH Hilo Medical Services: Campus Center, room 212, 932-7369
REPORTING INCIDENTS
If you wish to REPORT an incident of sex discrimination or gender-based violence
including sexual assault, sexual harassment, gender-based harassment, domestic violence,
dating violence or stalking as well as receive information and support*, contact:
* Please note that you do not have to file a complaint with the University to receive
institutional support or assistance.
For more information regarding sex discrimination and gender-based violence, the
University’s Title IX resources and the University’s Policy, Interim EP 1.204, go to:
http://www.hawaii.edu/titleix
13
Formula Sheets and Review Sheets for
Exams
14
Exam 1 Review Sheet
15
Finance 320 Exam 1: SAMPLE:
You will be allowed to use the formula sheet on the exam. The formula sheet will be identical to the one
that I have handed out prior to the exam. I will provide you a clean copy of the formula sheet on
the exam, so you may mark up the one I am giving you prior to the exam as you wish.
You will not be allowed to use the sample questions or review sheet on the exam.
Extra paper will not be necessary for the exam. I will provide plenty of room on the exam for you to
answer the questions.
Your exam will cover Chapters 1, 2, 4, 5, 6. You are responsible for Certifications in Finance, however, this topic
will only be tested on a surface level.
16
Fin 320 Sample Exam Questions
These questions are typical of questions that I might ask on an exam.
a. ___________
b. ___________
c. ___________
d. ___________
2) You are considering investing in a stock but you want to evaluate the expected return and
riskiness of the stock before you make the investment. Based on your best estimates, you have
accumulated the following information:
3.) The nominal risk free rate of interest has several components. These components are
a. The real risk free rate of interest and the liquidity premium
b. The inflation premium and the default risk premium
c. The real risk free rate of interest and the inflation premium
d. The liquidity premium and the inflation premium
e. The maturity risk premium and the default risk default risk premium.
17
Fin 320 Exam 1 Review SAMPLE
While I do try to make the review sheet as inclusive as possible, we may have discussed items in class
that are not covered on the review sheet. Anything that we covered in class is fair game for the exam.
Chapter 1:
a. Organizational Forms
Characteristics of organizational forms
How to determine if you are a partnership or a corporation
How you are taxed if you are a partnership or a corporation
How to calculate the amount of taxes you will have to pay if you are a partnership or a corporation
How do other organizational forms such as S corporations or limited liability companies fit into the
picture.
What are the characteristics of S corporations and limited liability companies.
b. Agency Problems
The agency problem between stockholders and managers
The agency problem between stockholders and bondholders
Readings:
#1 Does School Matter, an Empirical Examination of CEO Education, Compensation and Firm
Performance.
18
Chapter 4: Forecasting Financial Statements
a. How to use the RMA ratios to forecast the income statement and balance sheet starting from a
owners equity contribution figure or a sales figure.
Total Assets = Common Stock and Ret. Earn./percent of funds from this source
(1 + S 0 − L ) L = (1 + S 0 − N ) n1 (1 + X n− L ) n 2
( (1 + S 0 − L) L
(1 + S 0 − N ) n1
) 1
n2
− 1 = X n− L
Chapter 6:
1. Using subjective data K = ∑ PiKi
2.
Using historical data K = K avg =
∑ Kh
N
19
2. For a Portfolio
a. Expected Return
1. Using subjective data Kp = W 1K1 + W 2 K 2 + ...WnKn
2.Using Historical data Kp = K avgp =
∑ Khp
N
where Khp = W 1Kh1 + W 2 Kh 2 + ...WnKhn
Measures of Risk
a. Stand Alone
1. Standard deviation
2
a. from subjective (forward looking) data σ= ∑ ( ki − k ) pi
N −1
σ S
2. Coefficient of Variation CV = or
K K avg
b. Portfolio
How does combining stocks into a portfolio affect the risk of your investments?
1. Standard deviation
N −1
Sp
2. Coefficient of Variation CVp =
K avgp
20
How does portfolio size affect risk?
1. Confidence Intervals
a. Single Stock
CI 68 = K ± 1σ , orK avg ± 1S
CI 95 = K ±2σ , orK avg ± 2 S
CI 99.75 = K ±3σ , orK avg ± 3S
b. Portfolio
21
Exam 1 Formula Sheet
22
Fin 320 Principles of Finance: Exam 1 Formula Sheet SAMPLE
Assets
Cash
Accounts Receivable
Inventories
Total Current Assets
Gross Fixed Assets
Less Accumulated Deprecation
Net Fixed Assets
Total Assets
Sales
Cost of Goods Sold (COGS)
Other Expenses
Depreciation
Total Operating Costs
EBIT
Interest Expense
EBT
Taxes (rate)
Net Income
Net Income
+ Depreciation
Increases in Liabilities
+ Increases in Accounts Payable
+ Increases in Notes Payable
+ Increases in Accruals
+ Increases in Long-Term Debt
23
Increases in Assets
- Increases in Inventory
- Increases in Accounts Receivable
- Increases in other Current Assets
- Increases in Gross Fixed Assets
Cash Paid to and Received from Stockholders
+ New Investment by Stockholders
- Dividends Paid
= Change in Cash Position
Cash Balance
+ Change in Cash Position
= Cash Balance
Current Assets
Current Ratio =
Current Liabilities
24
Financial Statement Conventions for Chapter 4
Sales
Cost of Goods Sold (Sales – Gross Profit)
Gross Profit (%)
Operating Expenses (%)
Earnings Before Interest and Taxes (%)
(Operating Profit)
Interest Expense (%)
(Other Expenses)
Earnings Before Tax (%)
Assets
Cash and Equivalents % $
Trade Receivables % $
Inventory % $
Other Current Assets % $
Total Current Assets % $
Fixed Assets (Net) % $
Intangibles % $
Other Non-Current Assets % $
Total Assets 100% $
Liabilities
Notes Payable % $
Current Maturity LT Debt % $
Accounts Payable % $
Income Taxes Payable % $
All Other Current % $
Total Current Liabilities % $
Long Term Debt % $
Deferred Taxes % $
All other Non-Current Liabilities % $
Common Stock and Retained Earnings % $
Total Liabilities and Equity 100%
Total Assets = Common Stock and Ret. Earn./percent of funds from this source
25
k = k ∗ + IP + DRP + LP + MRP , krf = k ∗ + IP
(1 + S 0 − L ) L = (1 + S 0 − N ) n1 (1 + X n− L ) n 2
(((11++SS 0−L )L
0 − N)
n1
) 1
n2
− 1 = X n− L
K = ∑ PiKi
K = K avg =
∑ Kh
N
Kp = K avgp =
∑ Khp where Khp = W 1Kh1 + W 2 Kh2 + ...WnKhn
N
S=
∑ (kh − K avg ) 2
N −1
σ S
CV = or
K K avg
Sp =
∑ (khp − K avgp) 2
N −1
Sp
CVp =
K avgp
CI 68 = K ± 1σ , orK avg ± 1S CIp 68 = K avgp ± 1Sp
26
CI 95 = K ±2σ , orK avg ± 2 S CIp95 = K avgp ± 2 Sp
CI 99.75 = K ±3σ , orK avg ± 3S CIp99.75 = K avgp ± 3Sp
^ ^
σij = ∑
[( Ki − Ki ) X ( Kj − Kj )]
N −1
σij
ρij =
σiXσj
27
Review Sheet for Exam 2
28
Review Sheet for Exam 2
Fin 320 Principles of Finance
SAMPLE
29
Chapter 8:
Time Value of Money Terminology and tools for determining the time value of money
A. Single Sum
B. payment
C. annuity and annuity due
D. perpetuity, perpetuity due and growing perpetuity
E. uneven Cash Flow Streams
F. Complex Problems
G. combination Problems
H. amortization
I. effective annual interest rates
J. Time Lines
K. Simple interest versus compound interest
L. Formula’s for calculating the present value and the future value of a single sum.
How to calculate:
30
Chapter 10:
Concepts
Stock Calculations
A. Dividend Valuation
The constant growth dividend valuation model
The zero growth dividend valuation model
The supernormal growth dividend valuation model
A. Comparables Method
B. From Share Price to firm Value
C. Computing the Required Rate of Return
Historical method
Using the Security Market Line
Chapter 9:
Bond Concepts
Bond Calculations
A. Coupon Payment
B. Value of a bond today (annual and semiannual coupon payments)
C. The effect of a change in interest rates on the value of a bond.
D. Yield to Maturity
E. Yield to Call
31
Exam 2 Formula Sheet
32
Formula Sheet for Second Exam: Principles of Finance Fin 320
Stream of Cash Yes Limited Number Yes Equal Time Yes Equal Dollar Yes
Annuity
Flows? of Cash Flows? Spacing? Amount?
No No No No
No No
No
Combination Combination
Problem Problem
A B C D E F G H
|----------|----------|----------|----------|----------|----------|----------|
First Middle Last
Cash Flow Cash Flows Cash Flow
PV PV FV FV
OA AD OA AD
PMT 1
PVOP =
kreq
PMT 0
PVPD = + PMT 0
kreq
33
PMT 1 PMT − 0 (1 + g )
PVGOP = =
kreq − g kreq − g
PMT 0 (1 + g )
PVGPD = + PMT 0
kreq − g
^ Do(1 + g ) D1
Po = =
Kreq − g Kreq − g
^ Do
Po =
Kreq
Dt = Dt-1(1+g)
^
P o = EPS X PEC
^ ^
V o = P o X SO
34
Exam 3 Review Sheet
35
Review Sheet for Exam 3
Fin 320 Principles of Finance
SAMPLE
B. Your exam will cover Chapters 8, 10, 9, 14, 13 and 11, only as far as we cover in class
C. Your Chapter 13 homework is due on the day of the examination.
D. We will have an in class review for the exam during our regular class time on.
E. Class will not meet on DATE. However, I will be available in my office during our
regular class times for consultation on. We have been meeting an extra 5 minutes each day
throughout the semester so we have earned this day off.
F. I will come through the library on the evening of DATE.
G. I will be in my office on DATE from about TIME.
H. There will be approximately 20 questions on the exam. The primary focus of the exam is on
problem solving.
I. I will provide you with a clean copy of the formula sheet for the exam.
J. You may use the formula sheet and your calculator on the exam. You can not use this review
sheet for the exam.
K. Make sure to bring your calculator to the exam.
L. While I try to make the review sheet comprehensive, I do not guarantee them. Any materials
we have covered in class, reading or in the homework are fair game for the exam.
M. I will e-mail your exam grade to you if you would like. I will request your e-mail address on
the exam for that purpose.
N I do not distribute my final exams but you may view your exam in my office.
Chapter 8:
Time Value of Money Terminology and tools for determining the time value of money
A. Single Sum
B. payment
C. annuity and annuity due
H. perpetuity, perpetuity due and growing perpetuity
I. uneven Cash Flow Streams
J. Complex Problems
K. combination Problems
H. amortization
I. effective annual interest rates
K. Time Lines
L. Simple interest versus compound interest
M. The mathematical formula for computing the present value of a single sum
N. The mathematical formula for computing the future value of a single sum.
36
How to Calculate
Chapter 10:
Concepts
Stock Calculations
A. Dividend Valuation
The constant growth dividend valuation model
The zero growth dividend valuation model
The supernormal growth dividend valuation model
B. Comparables Method
C. From Share Price to firm Value
D. Computing the Required Rate of Return
Historical method
Using the Security Market Line
Chapter 9:
Bond Concepts
A. Borrowing from a bank versus through a bond
B. Face Value
K. Maturity
L. Issue Date
M. Coupon Payment
N. Coupon Rate
37
O. Callable Bonds
L. Convertible Bond
M. Default Risk
Bond Calculations
A. Coupon Payment
B. Value of a bond today (annual and semiannual coupon payments)
C. The effect of a change in the interest rate on the value of a bond.
D. Yield to Maturity
F. Yield to Call
Chapter 14:
a. Gathering information about the cash flows
Chapter 13
a. Mutually Exclusive versus Independent Projects
b. Normal versus non-normal projects
c. The required rate of return
d. Know how to calculate, the decision rule for, the advantages, the disadvantages, and when to use
each of the following
I. payback period
II. Net Present Value
III. Internal Rate of Return.
IV. Profitability Index
38
Exam 3 Formula Sheet
39
Formula Sheet for Second Exam: Principles of Finance Fin 320
Stream of Cash Yes Limited Number Yes Equal Time Yes Equal Dollar Yes
Annuity
Flows? of Cash Flows? Spacing? Amount?
No No No No
No No
No
Combination Combination
Problem Problem
A B C D E F G H
|----------|----------|----------|----------|----------|----------|----------|
First Middle Last
Cash Flow Cash Flows Cash Flow
PV PV FV FV
OA AD OA AD
PMT 1
PVOP =
kreq
PMT 0
PVPD = + PMT 0
kreq
40
PMT 1 PMT − 0 (1 + g )
PVGOP = =
kreq − g kreq − g
PMT 0 (1 + g )
PVGPD = + PMT 0
kreq − g
^ Do(1 + g ) D1
Po = =
Kreq − g Kreq − g
^ Do
Po =
Kreq
Dt = Dt-1(1+g)
^
P o = EPS X PEC
^ ^
V o = P o X SO
Total Revenues
-Operating Costs (excluding depreciation)
-Depreciation
=Earnings before Interest and Taxes (EBIT)
-Interest
=Earnings Before Taxes (EBT)
-Tax
= Net Income
+ Depreciation
= Net Operating Cash Flows
Equipment Cost
Installation
Increase in Net Working Capital (NWC)
Sales Price
Tax on Gain on Sale
Return of Net Working Capital (NWC)
41
DepreciableBase − SalvageValue
AnnualDepreciation =
DepreciableLife
− LastNegCumCashFlow
PB = BaseYear +
FollowingCashFlow
NPV
PI =
Costof Pr oject
42
The University of Hawaii at Hilo Finance Program
Rev 8/20/2020 Subject to Change
Be sure to check the University course catalog for the latest updates and additional details
*Finance 220: Personal Finance (Fall and Spring Semester, depends upon instructor availability)
Finance 320: Principles of Finance (Fall and Spring Semester)
Finance 321: Investment & Securities Analysis (Spring Semester)
Finance 322: Corporate Finance (Spring Semester)
Finance 370: Principles of Real Estate (Spring Semester, depends upon Instructor availability)
Finance 325: Small Business Finance (Even Year Fall Semesters)
Finance 412: Option and Futures (Odd Year Fall Semesters)
Sequencing: You must take Finance 320 first. After successfully completing Finance 320, you may take any
other finance course in any sequence. For example, you can take Finance 412 before taking Finance 321.
These course schedules change from time to time. Many factors can affect course offerings including
illnesses, sabbaticals, and faculty availability. Please check the University website for current schedules.
* Finance 220 cannot be used to satisfy upper division business elective requirements. But it does fulfill a
Global and Community Citizenship (GCC), Social Science (DS) or Quantitative Reasoning B (GQ) requirement.
Bachelor of Business Administration students, with majors in business, using catalogs from 2011-2012 or
later, must earn at least one concentration (Finance, Management or Marketing) as part of the BBA major.
Students may earn multiple concentrations by completing the necessary requirements for each. Courses
counted for one concentration may be used to partially complete the requirements for another concentration.
Finance Concentration (rev 2012-2013) and later. Business Administration Students electing to concentrate
in Finance must complete the following courses with a grade of C or better.
Finance 320: Principles of Finance (required for all business and accounting majors)
Finance 321: Investment & Securities Analysis
Finance 322: Corporate Finance
Two additional upper division courses with a FIN alpha
Two additional upper division courses from any business alpha (only 1 may be Econ on some catalogs)
CERTIFICATE IN FINANCE
The certificate in finance is designed for non-business majors who wish to pursue specific study in the area of
finance. To earn the certificate in finance, students must complete the following courses each with a grade
of C or better. They must also complete a Bachelor’s degree in any area.
43
CONCENTRATIONS
You must earn at least one concentration as part of the BBA Degree.
EXAMPLE
In this example, an Accounting Major could add a Business Major with a Finance
Concentration by taking four extra classes beyond those required for the
Accounting Major. You would have a BBA Degree with Major in Accounting and
Business Administration with a Finance Concentration.
44
Licenses Certifications and Salaries in Finance
45
Salaries in Finance
Actuary $70,000-160000
Source (Salary.com, 2018)
46
License in Finance
In order to do many things in finance you must have a license. These license
are similar to a drivers license for your car. It says you have met the
minimum requirements to do this particular thing in finance.
Certifications
Investments
First awarded in 1963, the CFA charter has become known as the gold standard of
professional credentials within the global investment community. Around the world,
employers and investors recognize the CFA designation as the definitive standard for
measuring competence and integrity in the fields of portfolio management and investment
analysis.
The Chartered Market Technician (CMT) Program is a certification process in which candidates
are required to demonstrate proficiency in a broad range of technical analysis subjects.
Administered by the Accreditation Committee of the Market Technicians Association (MTA), the
CMT Program consists of three levels: Level 1 and 2 are multiple choice exams; at level 3,
candidates have the option of writing a research paper or taking a third(essay) exam
The CFS is the oldest designation in the mutual funds industry. After you complete the program
your knowledge about mutual funds will be head and shoulders above the competition. The
information and marketing ideas you will learn are unparalleled. Studies research reports, industry
trends, new ideas and products, sales presentations and marketing tips among others.
The CFS program is a 60-hour self-study program. The cost of the program is $1,165. The cost
includes everything: registration, shipping, materials, testing, and the first year's certification fee.
47
Materials include: Morningstar’s Guide to Mutual Funds, Selected Readings I, Selected Readings
II, Student Manual with sample questions, and the Mutual Fund & Variable Annuity Directory.
The final exam is given at Thomson Prometric Centers which are located throughout the country.
The exam is administered by the NASD
The Certified Mutual Fund Specialist Program gives students - most often, financial services
employees - a broad overview of the mutual fund business. The program looks at the industry, its
history, regulation, providers, structure, operations, distribution, marketing and the popularity of
open-end mutual funds in the United States. National Investment Company Service Association
(NICSA) offers the program, which is made up of three courses: Introduction to the Mutual Fund
Industry Course, Mutual Fund Operations Course, Mutual Fund Distribution & Marketing Course
Completion of the three courses earns the student a certificate from NICSA in the specific area of
study. The program is based on "Purely American Invention: The U.S. Open-End Mutual Fund
Industry," a book by Lee Gremillion. Dr. Gremillion is a former partner with
PricewaterhouseCoopers LLP, where he led the Investment Management and Securities
Operations consulting group in the Midwest. Dr. Gremillion is now a professor at Capella
University. He also holds a doctoral degree from Harvard. Like the book, the interactive courses
help students understanding the breadth of mutual fund operations and their context.
Board Certified in Asset Allocation (BCAA) is the only designation of its kind. It was created to
address the frustration of investors, brokers, and advisors who now understand the value of a
properly-constructed portfolio. Yet, the general public remains leery of relying on in-house
solutions and proprietary products that lack objectivity.
The 60-hour self-study BCAA program is appealing for several reasons: (1) the materials are
completely neutral and have no hidden agenda, (2) content is comprehensive, logical, and easy to
understand, (3) the information gained can be applied to all of your clients, (4) it is the only asset
allocation certification program in the country, and (5) the designation will set you apart from your
present and future competition.
The three final exams are given at Thomson Prometric Centers and administered by the NASD.
Students must also complete two open-book case studies. The $1,165 BCAA program is fully
accredited and counts as four units toward the MSFS graduate degree.
Many investors suffered huge losses over the past few years. Everyone is looking for answers and
guidance. Peace of mind for investors comes from finding a knowledgeable advisor they trust in
and believe. The Board Certified in Securities (BCS) designee is just such a person.
The BCS program is a 60-hour self-study program. The cost includes everything: registration,
shipping, materials, testing, and the first year's certification fee. The three final exams are given at
Thomson Prometric Centers and administered by the NASD. Students must also complete two
open-book case studies. The BCS program is fully accredited and counts as four units toward the
MSFS graduate degree.
There are only two designations for those who specialize in individual securities: Chartered
Financial Analyst (CFA) and Board Certified in Securities (BCS). The CFA program is designed
48
for portfolio managers, not advisors who deal with the general public. The CFA program costs
several thousand dollars and takes several years to complete. The BCS program was developed for
the practitioner who has a personal relationship with an individual or company. The BCS materials
can be completed in 60 hours or less and cost just $1,165.
The Certified Mutual Fund Specialist Program gives students - most often, financial services
employees - a broad overview of the mutual fund business. The program looks at the industry, its
history, regulation, providers, structure, operations, distribution, marketing and the popularity of
open-end mutual funds in the United States. National Investment Company Service Association
(NICSA) offers the program, which is made up of three courses: Introduction to the Mutual Fund
Industry Course, Mutual Fund Operations Course, Mutual Fund Distribution & Marketing Course
Completion of the three courses earns the student a certificate from NICSA in the specific area of
study. The program is based on "Purely American Invention: The U.S. Open-End Mutual Fund
Industry," a book by Lee Gremillion. Dr. Gremillion is a former partner with
PricewaterhouseCoopers LLP, where he led the Investment Management and Securities
Operations consulting group in the Midwest. Dr. Gremillion is now a professor at Capella
University. He also holds a doctoral degree from Harvard. Like the book, the interactive courses
help students understanding the breadth of mutual fund operations and their context.
Why has the CERTIFIED FINANCIAL PLANNER™ certification become so sought after by
consumers and the financial planners who serve them? The answer is simple. The public is
looking for a planner who has demonstrated a commitment to competency, and financial
professionals want an established certification that sets them apart in a globally expanding
financial planning profession. CFP Board research shows consumers increasingly rely on
credentials when selecting a financial adviser. You will be equipped to provide truly personalized
services to clients and maintain high levels of financial planning professionalism and expertise.
The CFP®, CERTIFIED FINANCIAL PLANNER™ and certification marks are the most
recognized financial planning marks in the world. Unlike an educational designation offered by a
college or university, CFP certification is based on independently established public interest
standards. As a result, it prepares you for a career-long commitment to meeting the ever-changing
needs of your clients. Examples of the benefits of CFP certification include:
A financial planning designation for the insurance industry awarded by the American College of
Bryn Mawr. ChFCs must meet experience requirements and pass exams covering finance and
investing. They must have at least three years of experience in the financial industry, and have
studied and passed an examination on the fundamentals of financial planning, including income
tax, insurance, investment and estate planning.
49
Accredited Financial Counselor
Accredited Financial Counselors have certified skills to assist individuals and families in the
complex process of financial decision making. The exam is a two part exam that is administered
by the Association for Financial Counseling and Planning Education. www.afcpe.org
Corporate Finance
The Certified Financial Manager (CFM) certification provides Members who are involved with
corporate cash management, financing and investment decisions, and risk management with a
means for further demonstrating an expanded skill set. This exam provides an in-depth measure of
competence in areas such as financial statement analysis, working capital policy, capital structure,
valuation issues, and risk management.
Treasury Management
Certified Cash Manager (CCM)
http://www.afpnj.org/ccm.html not the official website
The Association for Financial Professionals, AFP, has established the Certified Cash Manager
(CCM) designation to help provide it's members and the business community with a body of
knowledge necessary for functioning in the ever changing global economy which exists today.
Each June the AFP offers a standardized exam covering the various disciplines of treasury
management. The CCM designation is awarded to those individuals who display a comprehensive
understanding of this material. This material includes the banking and payment systems,
disbursements, collections and cash concentration, bank relationship management, short-term
borrowing and investing, cash flow forecasting, and international cash management. The exam is
administered by the AFP in various cities across the United States. For further information
concerning this program, please see the AFP's certification page.
www.afponline.org
The CTP sets the standard in the financial profession and is a symbol of excellence. It signifies that
an individual has demonstrated the knowledge and skills required to perform competently in today's
complex treasury environment. As a result of recent changes in corporate America, treasury and
finance professionals face more public scrutiny than ever before. They must now demonstrate a
mastery of risk management, capital structure, mergers and acquisitions and corporate
governance, in addition to cash management topics. With the CTP designation, AFP recognizes
the significant increase in knowledge required of treasury professionals on the job.
50
Valuation
Accredited Senior Appraiser (ASA)
www.appraisers.org
Each accredited member of the American Society of Appraisers has earned a professional
designation in one or more specialized areas of appraisal. To receive the accreditation, the
appraiser must pass intensive courses/written examinations, submit representative appraisal
reports, an appraisal experience log and evidence of a college degree or its equivalent.
Every accredited appraiser must start his or her ASA membership as a Candidate member. In order
to be accepted for Candidate membership, the prospective Candidate must be interviewed and
approved by his or her local chapter. Subsequently, each Candidate must pass ASA's Ethics
Examination and an examination on the Uniform Standards of Professional Appraisal Practice
(USPAP) within a specified period of time. (The Uniform Standards are published each year by
The Appraisal Foundation, authorized by Congress as the source of appraisal standards and
appraiser qualifications.)
The Certified Business Appraiser Accreditation Program is one of the most important components
of the Institute's professional development curriculum. Members who meet established criteria are
awarded the prestigious professional designation of Certified Business Appraiser (CBA). This
distinction, which denotes a level of competence attained only by the most accomplished business
appraisers, grants its recipients special recognition and prestige among fellow appraisers, the
courts and throughout the business appraisal community.
Applicant must pass a comprehensive written examination on current business valuation theory
and practice. Content and administration of the examination shall be as described in the
application handbook, under heading "Written Examination." Arrangements may be made for the
CBA examination to be supervised by a private proctor. Please contact IBA headquarters for
qualifying details.
51
Accredited Valuation Analyst (AVA)
www.nacva.com
The National Association of Certified Valuation Analysts (NACVA) trains and certifies
Accredited Valuation Analysts (AVA) to perform business valuations as a service to both the
consulting community and the users of their services. Through training and rigorous testing,
AVAs demonstrate they are qualified to provide capable and professionally executed valuation
services. NACVA requires training as a prerequisite to certification to assure that practitioners
have the knowledge and understanding necessary to perform competent services, and to assure a
level of consistency and continuity in their work product. Users of valuation services benefit by
having greater confidence that the service they receive is professional in quality, adheres to
industry standards of practice, and meets a level of expertise the Association deems credible and
worthy of one of its certified members. AVAs must hold a business degree from an accredited
institution of higher education and demonstrate substantial business valuation experience, among
other requirements.
Credit Analysis
The Credit Analysis Certifications are all offered by the National Association of Credit
Management www.nacm.org
The Credit Business Associate (CBA) is an academic-based designation which signals mastery of
three business-credit related disciplines: basic financial accounting, business credit principles and
introductory financial statement analysis. There is no minimum work experience requirement for
this designation level and the course work needed to qualify for this designation can be obtained
through colleges, local NACM Affiliated Association programs, self-study or nationally-
sponsored programs.
52
Certified Credit Executive (CCE)
The Certified Credit Executive® (CCE®) is NACM's executive level designation which endorses
its achievers as capable of managing the credit function at an executive level. Candidates must
pass a rigorous, four hour exam which tests application skills in the areas of accounting, finance,
domestic and international credit concepts, management and law.
Public Finance
Since its inception in 1994, the CGFM has become the standard by which government financial
management professionals are measured. Its education, experience and ethics requirements have
served to elevate the most seasoned financial professionals. The CGFM program spans the public,
private and academic sectors in the United States and abroad. It recognizes the unique skills and
experience of government financial management professionals. It identifies those who have
knowledge in many functional areas and can apply it in a government setting.
The Certified Public Finance Officers Program (Certification Program) of the Government Finance
Officers Association of the United States and Canada (GFOA) is a broad educational self-study
program designed to verify knowledge in the disciplines of government finance. The Certification
Program is governed by the Council on Certification. Technical and administrative support for the
program is provided by Radford University's Governmental and Nonprofit Assistance Center.
To earn the designation of Certified Public Finance Officer (CPFO), candidates must pass a series
of five examinations covering the major disciplines of public finance.
Insurance
Certified Risk Manager (CRM)
www.scic.com
The Certified Risk Managers International (CRM) designation demonstrates that you are
knowledgeable in all areas of managing risks, hazards, and exposures. The five courses give you
in-depth knowledge about today’s highest priorities – identifying, analyzing, controlling,
financing, and administering operational risks – as well as political risks, catastrophic loss
exposures, third-party exposures, fiduciary exposures, employee injury exposures, juridical risks,
legal risks, and more – whether insurable or not. The skills you learn will make you more
proactive and valuable to your organization in discovering how risks can interrupt the flow of
earnings and how to protect against it.
The Certified Insurance Counselors (CIC) Program has been the insurance industry's premier,
proven source for practical, real-world education since 1969. Designed to cover important aspects
of the insurance field, each of five institutes is 20 hours of instruction
53
Chartered Property Cacualty Underwriter (CPCU)
Real Estate
ABR – Accredited Buyer Representative is a designation from the Real Estate Buyer’s Agent
Council that trains experienced REALTORS® in specifically representing the real estate
consumer.
The ABR designation is awarded to real estate practitioners who complete a comprehensive two-
day REBAC course in buyer representation, achieve a passing grade on the written examination,
demonstrate practical experience by completing and closing five real estate transactions in which
the candidate functioned as a buyer's representative and who maintain a membership in good
standing with the National Association of REALTORS® and the Real Estate BUYER'S AGENT
Council. Web page at www.rebac.net
CPM – Certified Property Manager designation is the oldest and most prestigious
achievement in property management. It is awarded to property managers whose
experience, education and ethical standards warrant such distinction. CPMs are experts
in managing apartments, office buildings, commercial centers and homeowners’
associations and are informed on tax laws insurance regulations, and critical investment
factors.
To become a CPM, one must complete a combination of core and elective requirements.
One will need 260 points --160 required points and 100 elective points - in addition to
meeting some other criteria unrelated to the point system.
Institute of Real Estate Management (IREM). Web page at www.irem.org.
54
Other
The Certified Business Manager (CBM) program provides a generalized business education at the
graduate level, covering essential subject matter useful to business practitioners. The CBM
program is designed to complement existing specialty certifications, to validate a business
professional’s practical business experience and knowledge, and to act as an efficient preparatory
tool for a graduate business program.
The credential takes a comprehensive MBA curriculum and presents it in a certification format,
teaching and measuring the skills and knowledge required to become a successful manager.
The CBM Exam covers ten essential areas/modules of business (i.e. Accounting, Finance,
Information Technology, Marketing Management, Human Resource Management, International
Business and Quality Management) and their cross-functional applications in three, three hour-
long parts. Each part contains 150 multiple-choice questions.
The REBC designation signifies that an individual working in the employee benefits field has
successfully completed a comprehensive five-course program. The number of electives provides
an opportunity for individuals to specialize within the broader employee benefits field. The
designation is especially suitable for professionals who are selling or servicing the group
insurance, health insurance, retirement planning, and incentive compensation markets. More than
1,250 REBCs have been awarded, including those granted by the College since assuming
ownership of the designation from the National Association of Health Underwriters.
The designation requires five courses - three which are required and two electives. Students also
must meet specified experience requirements, maintain ethical standards, and agree to comply
with both The American College's Code of Ethics and Procedures and applicable continuing
education requirements.
55
CHAPTER 1
An Overview of Financial
Management
56
Organizational Forms
Selecting an organizational form is one of the first decisions you
must make if when you decide to start a company
57
Corporate Characteristics
1. Limited Liability –
2. Continuity of Life
58
3. Centralized Management
59
Taxation
60
Partnership (or Pass Through Taxation)
Given:
Firm Earnings $100,000
When appropriate, corporate Tax Rate = 25%
When Appropriate Personal Tax Rate = 34%
All earnings of the firm are paid out to the owners
61
Corporate Taxation (or Double Taxation)
The corporation is taxed on its earnings. When the earnings are paid to the
owners, they are also taxed at the personal level.
Given:
Firm Earnings $100,000
When appropriate, corporate Tax Rate = 25%
When Appropriate Personal Tax Rate = 34%
All earnings of the firm are paid out to the owners
Earnings $100,000
Corporate Tax Rate X .25
Corporate Taxes Paid = $ 25,000
Earnings $100,000
less Corporate Taxes Paid - $ 25,000
Earnings paid to owners = $ 75,000
62
C-corporations have all four characteristics. However, they are
subject to double taxation.
2. Borrow Money
Because interest payments are a tax deductible expense
for the corporation, the effects of double taxation can be
reduced by borrowing money
3. Tax Inversion
Tax Inversions involve a corporation merging with
another foreign corporation to avoid U.S. taxes.
63
S-Corporations
64
Limited Liability Company (LLC)
65
Agency Problems
An Agency Problem Occurs when two people work together
but their goals are not necessarily the same.
a. Take Perks
b. Expand the firm to a large size
c. Reduce the risk of the firm
d. Oppose takeover threats
e. Keep a failing company alive longer than is optimal
66
II. Agency Problems between the owners and the bondholders
(lenders).
67
4. Methods to reduce the conflict between owners and
bondholders.
c. Personal Guarantees
d. Require Collateral
68
Business Application Forms for the State of Hawaii
http://hawaii.gov/dcca/
http://hawaii.gov/dcca/breg/registration/dllc/
http://hawaii.gov/dcca/breg/registration/registratio
n-form-info.html
https://hbe.ehawaii.gov/BizEx/home.eb
http://portal.ehawaii.gov/help-center-questions/file-
bb1.html
69
70
71
72
73
74
75
76
77
Chapter 1 Homework Assignment and Solution
78
Chapter 1
Homework Assignment
3. Which characteristics does a sole proprietorship or partnership have and how are
they taxed?
7. Discuss the agency problem between the firm’s owners and the firm’s
managers. How can the firms’ owners minimize this agency problem?
79
Chapter 1 Homework Solutions
a. Limited Liability – refers to how much money can potentially be lost by the
owners of the company if the company fails. If the company has limited
liability the loss is limited to the amount of money invested in the firm. If the
company does not have limited liability, the owners can lose their investment
in the firm and all of their personal wealth as well.
b. Continuity of Life – refers to what happens when one of the owners dies. If a
firm has Continuity of life, when one of the owners dies the company
continues without interruption. If the firm does not have continuity of life,
when one of the owners dies, the business is dissolved.
80
2. Give and example of pass-through taxation and double taxation.
Given:
Corporate Earnings $200,000
Corporate Tax Rate 30%
Personal Tax Rate 36%
All earnings of the corporation are paid out to the owners
Since there is no corporate taxes under the pass through taxation system, the net
earnings paid to the stockholders is the full $200,000.
81
Example of Double Taxation (using the same information from the previous
example).
Earnings $200,000
Corporate Tax Rate x .30
Corporate Taxes Paid = $60,000
Earnings $200,000
Less Corporate Taxes Paid - 60,000
Earnings paid to the owners = $140,000
3. Which characteristics does a sole proprietorship or partnership have and how are
they taxed?
82
5. What limitations are S-corporations subject to?
7. Discuss the agency problem between the firm’s owners and the firm’s
managers. How can the firms’ owners minimize this agency problem?
An Agency Problem that occurs when two people who work together but have
differing goals. An agency problem occurs between the stockholders of the firm
and the owners of the firm. The owners of the company would like the managers
to maximize their wealth. The managers of the company are more interested in
maximizing their own wealth. So here is a conflict between the two parties and
we called this conflict an agency problem.
1. Things managers can to maximize their own wealth at the expense of the
owners.
a. Take Perks
b. Expand the firm to a large scale
c. Reduce the risk of the firm
d. Oppose takeover threats
e. Keep a failing company alive longer than is optimal
83
CHAPTER 2
84
Chapter 2: Financial Statements,
Cash Flows and Taxes
85
Four Basic Financial Statements:
1. Balance Sheet:
Shows you everything you own, everything you owe, and the
net worth of your company on a specific day.
2. Income Statement:
Shows you the sources and uses of cash over some time
period.
Shows you what was done with the earnings of the company.
86
Finacial Statement Example This is for a Taxi Cab Business. For simplicity, assume that the cars are magical and never wear out.
Balance Sheet
Balance Sheet
Balance Sheet
Balance Sheet
Purchase Taxi
Statement at
Another Car
12-31-2014
Purchasing
12/31/2013
Investment
Retained
Retained
Earnings
Earnings
Dividend
$10,000
after the
Cab for
$9,000
Yr End
yr end
Initial
Year
After
year
Action
Balance Sheet 1-Jan-13 2-Jan-13 12/31/2013 12/31/2014
Assets
Cash 10000 1000 11000 31000 22000 12000
Cars 0 9000 9000 9000 18000 18000
Total Assets 10000 10000 20000 40000 40000 30000
Liabilities and Equity
Common Stock 10000 10000 10000 10000 10000 10000
Retained Earnings 0 0 10000 30000 30000 20000
Total Liabilities and Equity 10000 10000 20000 40000 40000 30000
Income Statement
Sales 50000 60000
other Expenses Including Tax and Interest -40000 -40000
Net Income 10000 20000
Statement of Retained Earnings
Retained Earnings Previous Year 0 10000
+ Net Income for This year 10000 20000
- Dividends paid 0 0
= New Retained Earnings 10000 30000
Cash Flow Statement
Net Income 10000 20000
+- Adjustments 0 0
= Change in Cash Position 10000 20000
Old Cash 1000 11000
+ Change in Cash Position 10000 20000
= New Cash 11000 31000
87
Hilo Clothing Market
Balance Sheet
12-31-2012 12-31-2013
Assets
Cash $100,000 $193,200
Accounts Receivable $75,000 $175,000
Inventories $150,000 $175,000
Total Current Assets $325,000 $543,200
Gross Fixed Assets $800,000 $1,000,000
Less Accumulated Deprecation $100,000 $225,000
Net Fixed Assets $700,000 $775,000
Total Assets $1025,000 $1,318,200
88
Hilo Clothing Market
Income Statement
Year Ending Year Ending
12-31- 2012 12-31-2013
89
Hilo Clothing Market
Statement of Retained Earnings for Year Ending 2013
90
Evaluation of Financial Statements:
Ratio Analysis
The market value of equity tells you how much money it would cost
to purchase the entire equity stake in the company.
= 100,000 X 36.15
= $3,615,000
The P/E ratio tells us that if earnings do not change from their current levels,
how long it will take in years before the firm will earn back the amount of
money that we paid for the stock.
= 14.11 times
The P/E ratio tells us that if earnings do not change from their current levels,
it will take 14.11 years before the firm will earn back the amount of money
that we paid for the stock.
91
Total Asset Turnover
Tells you how many dollars of sales the company produced for
each dollar of assets.
𝑆𝑆𝑆𝑆𝑆𝑆𝑆𝑆𝑆𝑆
𝑇𝑇𝑇𝑇𝑇𝑇𝑇𝑇𝑇𝑇 𝐴𝐴𝐴𝐴𝐴𝐴𝐴𝐴𝐴𝐴 𝑇𝑇𝑇𝑇𝑇𝑇𝑇𝑇𝑇𝑇𝑇𝑇𝑇𝑇𝑇𝑇 =
𝑇𝑇𝑇𝑇𝑇𝑇𝑇𝑇𝑇𝑇 𝐴𝐴𝐴𝐴𝐴𝐴𝐴𝐴𝐴𝐴𝐴𝐴
2,300,000
𝑇𝑇𝑇𝑇𝑇𝑇𝑇𝑇𝑇𝑇 𝐴𝐴𝐴𝐴𝐴𝐴𝐴𝐴𝐴𝐴 𝑇𝑇𝑇𝑇𝑇𝑇𝑇𝑇𝑇𝑇𝑇𝑇𝑇𝑇𝑇𝑇 =
1,318,200
So in this case the company had $1.748 of sales for each dollar of
assets that it had.
Inventory Turnover
Tells you how many dollars of sales the company produced for
each dollar of inventory.
𝑆𝑆𝑆𝑆𝑆𝑆𝑆𝑆𝑆𝑆
𝐼𝐼𝐼𝐼𝐼𝐼𝐼𝐼𝐼𝐼𝐼𝐼𝐼𝐼𝐼𝐼𝐼𝐼 𝑇𝑇𝑇𝑇𝑇𝑇𝑇𝑇𝑇𝑇𝑇𝑇𝑇𝑇𝑇𝑇 =
𝐼𝐼𝐼𝐼𝐼𝐼𝐼𝐼𝐼𝐼𝐼𝐼𝐼𝐼𝐼𝐼𝐼𝐼
2,300,000
𝐼𝐼𝐼𝐼𝐼𝐼𝐼𝐼𝐼𝐼𝐼𝐼𝐼𝐼𝐼𝐼𝐼𝐼 𝑇𝑇𝑇𝑇𝑇𝑇𝑇𝑇𝑇𝑇𝑣𝑣𝑒𝑒𝑒𝑒 =
175,000
In this case, the company had $13.14 of sales for each dollar of
inventory it had.
92
Return on Assets (ROA)
𝑁𝑁𝑁𝑁𝑁𝑁 𝐼𝐼𝐼𝐼𝐼𝐼𝐼𝐼𝐼𝐼𝐼𝐼
𝑅𝑅𝑅𝑅𝑅𝑅𝑅𝑅𝑅𝑅𝑅𝑅 𝑜𝑜𝑜𝑜 𝐴𝐴𝐴𝐴𝐴𝐴𝐴𝐴𝐴𝐴𝐴𝐴 =
𝑇𝑇𝑇𝑇𝑇𝑇𝑇𝑇𝑇𝑇 𝐴𝐴𝐴𝐴𝐴𝐴𝐴𝐴𝐴𝐴𝐴𝐴
256,200
𝑅𝑅𝑅𝑅𝑅𝑅𝑅𝑅𝑅𝑅𝑅𝑅 𝑜𝑜𝑜𝑜 𝐴𝐴𝐴𝐴𝐴𝐴𝐴𝐴𝐴𝐴𝐴𝐴 =
1,318,200
= 0.1944
The company had $0.19 of net income for each dollar of assets.
93
Return on Equity (ROE)
Tells you how much net income you had for each dollar of
equity invested in the company.
𝑁𝑁𝑁𝑁𝑁𝑁 𝐼𝐼𝐼𝐼𝐼𝐼𝐼𝐼𝐼𝐼𝐼𝐼
𝑅𝑅𝑅𝑅𝑅𝑅𝑅𝑅𝑅𝑅𝑅𝑅 𝑜𝑜𝑜𝑜 𝐸𝐸𝐸𝐸𝐸𝐸𝐸𝐸𝐸𝐸𝐸𝐸 =
𝐶𝐶𝐶𝐶𝐶𝐶𝐶𝐶𝐶𝐶𝐶𝐶 𝐸𝐸𝐸𝐸𝐸𝐸𝐸𝐸𝐸𝐸𝐸𝐸
256,200
𝑅𝑅𝑅𝑅𝑅𝑅𝑅𝑅𝑅𝑅𝑅𝑅 𝑜𝑜𝑜𝑜 𝐸𝐸𝐸𝐸𝐸𝐸𝐸𝐸𝐸𝐸𝐸𝐸 = = 0.3326
770,200
This says the company earned $0.3326 for each dollar of common
equity utilized.
94
Equity Multiplier
The Equity Multiplier tell you how many dollars of assets you are
controlling for each dollar of equity you have invested in the firm.
𝑇𝑇𝑇𝑇𝑇𝑇𝑇𝑇𝑇𝑇 𝐴𝐴𝐴𝐴𝐴𝐴𝐴𝐴𝐴𝐴𝐴𝐴
𝐸𝐸𝐸𝐸𝐸𝐸𝐸𝐸𝐸𝐸𝐸𝐸 𝑀𝑀𝑀𝑀𝑀𝑀𝑡𝑡𝑡𝑡𝑡𝑡𝑡𝑡𝑡𝑡𝑡𝑡𝑡𝑡 =
𝐶𝐶𝐶𝐶𝐶𝐶𝐶𝐶𝐶𝐶𝐶𝐶 𝐸𝐸𝐸𝐸𝐸𝐸𝐸𝐸𝐸𝐸𝐸𝐸
$1,318,200
𝐸𝐸𝐸𝐸𝐸𝐸𝐸𝐸𝐸𝐸𝐸𝐸 𝑀𝑀𝑀𝑀𝑀𝑀𝑀𝑀𝑀𝑀𝑀𝑀𝑀𝑀𝑀𝑀𝑀𝑀𝑀𝑀 = = 1.7115
$770,200
So the firm has accumulated $1.7115 of total assets for each dollar
of common equity invested in the firm.
95
Profit Margin (Sometimes called Net Profit Margin)
Profit Margin tells you how much net income the firm is producing
for each dollar of sales.
𝑁𝑁𝑁𝑁𝑁𝑁 𝐼𝐼𝐼𝐼𝐼𝐼𝐼𝐼𝐼𝐼𝐼𝐼
𝑃𝑃𝑃𝑃𝑃𝑃𝑃𝑃𝑃𝑃𝑃𝑃 𝑀𝑀𝑀𝑀𝑀𝑀𝑀𝑀𝑀𝑀𝑀𝑀 =
𝑆𝑆𝑆𝑆𝑆𝑆𝑆𝑆𝑆𝑆
$256,200
𝑃𝑃𝑃𝑃𝑃𝑃𝑃𝑃𝑃𝑃𝑃𝑃 𝑀𝑀𝑀𝑀𝑀𝑀𝑀𝑀𝑀𝑀𝑀𝑀 =
$2,300,000
Debt to Assets
Debt to Assets tells which portion of the firm's assets are financed
by debt.
𝑇𝑇𝑇𝑇𝑇𝑇𝑇𝑇𝑇𝑇 𝐿𝐿𝐿𝐿𝐿𝐿𝐿𝐿𝐿𝐿𝐿𝐿𝐿𝐿𝐿𝐿𝐿𝐿𝐿𝐿𝐿𝐿
𝐷𝐷𝐷𝐷𝐷𝐷𝑡𝑡 𝑡𝑡𝑡𝑡 𝐴𝐴𝐴𝐴𝐴𝐴𝐴𝐴𝐴𝐴𝐴𝐴 =
𝑇𝑇𝑇𝑇𝑇𝑇𝑇𝑇𝑇𝑇 𝐴𝐴𝐴𝐴𝐴𝐴𝐴𝐴𝐴𝐴𝐴𝐴
548,000
𝐷𝐷𝐷𝐷𝐷𝐷𝐷𝐷 𝑡𝑡𝑡𝑡 𝐴𝐴𝐴𝐴𝐴𝐴𝐴𝐴𝐴𝐴𝐴𝐴 =
1,318,200
So, 41.57 percent of the firm's assets have been financed with debt
and the remainder is financed with equity.
96
Payout Ratio
The payout ratio address the issue of what you did with the money
you earned.
$11,000
𝑃𝑃𝑃𝑃𝑃𝑃𝑃𝑃𝑃𝑃𝑃𝑃 𝑅𝑅𝑅𝑅𝑅𝑅𝑅𝑅𝑅𝑅 = = 0.0429
$256,200
This indicates that we paid 4.29 percent of our net income to the
stockholders as a dividend.
97
The Current Ratio
The current ratio tells us the ability of the firm to pay its bills in the
short term.
Current Assets
Current Ratio =
Current Liabilities
$325,000
Current Ratio = = 0.929
$350,000
$543,200
Current Ratio = =1.365
$398,000
In 2012, the company had $0.929 in current assets for each dollar
of current liabilities.
98
Basic Dupont Analysis
Recall that:
𝑁𝑁𝑁𝑁𝑁𝑁 𝐼𝐼𝐼𝐼𝐼𝐼𝐼𝐼𝐼𝐼𝐼𝐼
𝑅𝑅𝑅𝑅𝑅𝑅𝑅𝑅𝑅𝑅𝑅𝑅 𝑜𝑜𝑜𝑜 𝐸𝐸𝐸𝐸𝐸𝐸𝐸𝐸𝐸𝐸𝐸𝐸 =
𝐶𝐶𝐶𝐶𝐶𝐶𝐶𝐶𝐶𝐶𝐶𝐶 𝐸𝐸𝐸𝐸𝐸𝐸𝐸𝐸𝐸𝐸𝐸𝐸
256,200
𝑅𝑅𝑅𝑅𝑅𝑅𝑅𝑅𝑅𝑅𝑅𝑅 𝑜𝑜𝑜𝑜 𝐸𝐸𝐸𝐸𝐸𝐸𝐸𝐸𝐸𝐸𝐸𝐸 = = 0.3326
770,200
99
And further expand it to:
100
Two methods for determining if a ratio is good bad or average
101
Step 3: Compare your firm to the Industry
A B
Sales $3,000,000 $3,000,000
Operating Expenses $1,600,000 $1,400,000
R&D Expenditures $0 $300,000
EBIT $1,400,000 $1,300,000
102
If we examine EBIT on its face, it appears as though
Company A is performing better than company B,
when company B is actually performing better.
2. Balanced Scorecard
103
Taxes
“The Only Two Things that you can be sure of in life are Death
and Taxes” Unknown
104
Computing Your Taxes
Salary
+ Investment Income
+- Gain or Loss on Business
- Contributions to Tax Deferred Accounts (IRA, 401K)
- Deductions
- Exemptions
= Taxable Income
Salary $116,000
+ Investment Income $0
+ Gain from Business $4,000
- Contributions to 401K $6,000
- Contributions to IRA $2,000
- Deductions $12,100
- Exemptions 3X 3,900 $11,700
= Taxable Income $88,200
105
Step 2: Gather data from Tax Table
Step 3: Solve for the Tax owed by using the following formula:
Tax = Tax on Base Amt + Tax Rate (Taxable Inc - Base Amt)
= $6577.50 + 0.28(88,200-43,850)
= $18,995.50
106
Tax Table
Single Individuals
Range Base Amt Tax on Tax Rate Avg. Rate at
Base Amt Top of
Bracket
Corporations
Range Base Amt Tax on Tax Rate Avg. Rate at
Base Amt Top of
Bracket
0 to $50,000 $0 $0 15% 15%
$50,000-$75,000 $50,000 $7,500 25% 18.3%
$75,000-$100,000 $75,000 $13,750 34% 22.3%
$100,000-$335,000 $100,000 $22,250 39% 34.0%
$335,000-$10mil $335,000 $113,900 34% 34.0%
$10mil-$15mil $10mil $3,400,000 35% 34.3%
$15mil-$18.33mil $15mil $5,150,000 38% 35%
above 18.33 mil 18.33mil $6,416,667 35% 35%
107
Using a Business to Reduce your Personal Taxes
Through careful planning, a business can be used to offset income
that you earn from your job.
Suppose that you work at a job where you earn $45,000 per year.
You are single and you have no children. You enjoy fishing and
go fishing almost every weekend. You eat the fish that you catch.
Salary $45,000
+ other income -0-
- Deductions $6,050
- Exemptions $3,900
Taxable Income $35,050
= $6,401.50
108
Now suppose that rather than eating the fish you catch, you obtain
a commercial fisherman’s license and sell the fish that you catch to
the local fish distributor. Because you have a profit motive, you
are in business and can deduct all expenses associated with the
activity.
Suppose that you go out fishing and you catch $47,000 of fish.
The operating expenses associated with catching the fish (gas, bait
and so forth) total $37,000. So you have a gross profit of $10,000
($47,000 - $37,000). However, because you have a business many
additional expenses are tax deductible.
109
Step 1: Compute the Taxable profit or loss from the Business
Sales $47,000
Operating Expenses $37,000
Gross Profit $10,000
Other Expenses
Depreciation on Boat, Truck
And Computer $10,000
½ of house rent $6,000
Internet Connection $250
Travel Expenses
(to Los Angeles to investigate the purchase of a new boat
and then to visit your friend Sandy) $2,000
Taxable Profit -$8,250
Salary $45,000
+ other income -8,250
- Deductions $6,050
- Exemptions $3,900
Taxable Income $26,800
= $4,091.50
Thus, by selling your fish at the market rather than eat them, you
reduce your taxes by:
110
Chapter 2 Homework and
Solutions
Problem #1. You have been assigned the problem of creating financial statements for a company for the
next year. You have the following information. The company was started on December 31,
2012.
Assets
Cash $2,000
+ Inventory $20,000
Buildings and Equipment $10,000
Less Accumulated Depreciation $0
+ Net Buildings and Equipment $10,000
Total Assets $32,000
Liabilities
Bank Loan $10,000
Owners Equity
Common Stock $22,000
Retained Earnings $0
You have created the following income statement for the year ending December 31, 2013.
Sales $80,000
- Cost of Goods Sold $20,000
- Administrative Expenses $5,000
- Labor $30,000
- Depreciation $1,000
=Earnings Before Interest and Tax $24,000
- Interest $1,000
=Earnings before Taxes $23,000
- Taxes @ 20 percent $4,600
=Net Income $18,400
Other information:
All sales are cash sales
All expenses except depreciation are cash expenses
No principal payments were paid on the bank loan.
The company paid $5,000 in dividends from cash.
Inventory Levels do not change during the year.
Your job is to create a retained earnings statement for the year ending December 31, 2013, and a balance
sheet for December 31, 2013.
Note: For this problem, the cash balance for the December 31, 2013 balance sheet is $16,400.
111
Problem 2: Today is December 31, 2018. Today, we started the Hilo Tourism Company. To start the
business, the owners have invested $25,000 into the business from their own pocket. In addition,
the company borrowed $15,000 at a 10 percent simple interest rate. Thus, the company has
$40,000 to work with. The company will use $10,000 of this money for operating cash. The
company will use the remaining money to purchase a tourism van for $30,000.
The Hilo Tourism Company will contract our services to Royal Cruise Line who has agreed to pay
us $160,000 in the year 2019 for providing tours to high-class customers from the Hilo dock to the
Volcano. All sales are cash sales. Hilo Tourism Company will have the following expenses in the
year 2019. The van will wear out by $10,000 per year. Insurance will cost $12,000 per year. It
will cost $20,000 per year for the driver and $16,000 per year for the tour guide. Fuel and repairs
combined will cost $40,000 per year. All operating expenses except depreciation are cash
operating expenses. The company is in a 10 percent tax bracket. One half of the firm’s 2019 net
income will be paid out to the owners as a dividend and the other half of the firm’s net income
will be retained within the firm. The company will make interest payments on the loan only.
Problem 3: Today is December 30, 2012. We will start a staple gun manufacturing company. To
start the company we needed $20,000 to purchase equipment. We need $5,000 for
operating funds. Thus to start the company we needed a total of $25,000. We obtained the
money to start the business by investing $15,000 from our pocket and by borrowing the
remaining $10,000 at a 5 percent simple interest rate. In addition to paying the interest
payments on the loan each year, we will pay off $1,000 of the amount that we owe on the
loan each year (the first payment on the loan will be made one year from today).
The firm will sell 200,000 staplers each year at a price of $3.00 each. Cost of
producing the staplers will be $2.00 each. In addition, sales and administrative
expenses will be $0.18125 per stapler. All sales are cash sales. All operating expenses
are cash operating expenses. The machine will wear out buy $2,000 per year. The
company is in a 10 percent tax bracket. One-fourth of the firm’s net income will be
paid out to the owners as a dividend and the other three-fourths of the firms net income
will be retained within the firm.
B. Create an income statement for the year ending December 31, 2013.
C. Create a statement of cash flows for the year ending December 31, 2013.
D. Create a statement of retained earnings for the year ending December 31, 2013.
F. Create an income statement for the year ending December 31, 2014
G. Create a statement of retained earnings for the year ending December 31, 2014.
H. Create a statement of cash flows for the year ending December 31, 2014
112
Problem 4: Today is December 30, 2012. Terry and Mercedes started a tourism company called Hilo
Tours today. In order to start the company we needed $53,000 to purchase a van and an additional
$5,000 for operating funds. Thus to start the company we needed a total of $58,000. We got the
money to start the business by investing $15,000 from our pocket and by borrowing the remaining
$43,000 at a 10 percent simple interest rate. In addition to paying the interest payments on the
loan each year, we will pay off $10,000 of the amount that we owe on the loan each year (the first
payment on the loan will be made one year from today).
Hilo Tours will contract our services to another company called Valley Tours. Valley Tours has
agreed to pay us $275 per day for providing tours from Hilo to the Waipio Valley. Valley Tours
will provide us with work 250 days per year. The van is expected to wear out by $8,000 per year.
We are responsible for providing the van and all fuel, a driver, a tour guide, and a snack for the
passengers. The van will drive 200 miles per working day. It will cost $40 per working day for
the driver and $30 per working day for the tour guide. It will cost $15 per day to provide the
snack. Fuel will cost $.10 per mile and repairs for the van will be $.20 per mile. All operating
expenses are cash operating expenses. The company is in a 10 percent tax bracket. One half of
the firm’s net income will be paid out to the owners as a dividend and the other half of the firms
net income will be retained within the firm.
B. Create an income statement for the year ending December 31, 2013.
C. Create a statement of cash flows for the year ending December 31, 2013.
D. Create a statement of retained earnings for the year ending December 31, 2013.
F. Create an income statement for the year ending December 31, 2014
G. Create a statement of retained earnings for the year ending December 31, 2014.
H. Create a statement of cash flows for the year ending December 31, 2014
113
Problem 5: You are provided the following balance sheet and income statement. In addition, you are
informed that the current stock price of the firm is $10 per share and there are 5,000 shares
outstanding. Use this information to compute the following:
Assets
Cash $2,000
+ Inventory $20,000
Buildings and Equipment $10,000
Less Accumulated Depreciation $0
+ Net Buildings and Equipment $10,000
Total Assets $32,000
Liabilities
Bank Loan $10,000
Owners Equity
Common Stock $22,000
Retained Earnings $0
You have created the following income statement for the year ending December 31, 2010.
Sales $80,000
- Cost of Goods Sold $20,000
- Administrative Expenses $5,000
- Labor $30,000
- Depreciation $1,000
=Earnings Before Interest and Tax $24,000
- Interest $1,000
=Earnings before Taxes $23,000
- Taxes @ 20 percent $4,600
=Net Income $18,400
114
Problem 6: You are provided the following Income Statement and Balance Sheet for Microsoft.
MICROSOFT CORP
BALANCE_SHEET
Period End: Jun 30, 2016
(In Millions)
2016 2015
Assets
Current assets:
Cash and cash equivalents $ 6,510 $ 5,595
Short-term investments (including securities loaned of $204 and $75) 106,730 90,931
------------------------------------------------------------------------------------------------------------------------------- -----------------
Total cash, cash equivalents, and short-term investments 113,240 96,526
Stockholders' equity:
Common stock and paid-in capital - shares authorized 24,000;
outstanding 7,808 and 8,027 68,178 68,465
Retained earnings 2,282 9,096
Accumulated other comprehensive income 1,537 2,522
115
MICROSOFT CORP
INCOME_STATEMENT
Period End: Jun 30, 2016
Revenue:
Product $ 61,502 $ 75,956 $ 72,948
Service and other 23,818 17,624 13,885
------------------------------------------------------------------------------------------ ---------------- ----------------
Total revenue 85,320 93,580 86,833
------------------------------------------------------------------------------------------ ---------------- ----------------
Cost of revenue:
Product 17,880 21,410 16,681
Service and other 14,900 11,628 10,397
------------------------------------------------------------------------------------------ ---------------- ----------------
Total cost of revenue 32,780 33,038 27,078
------------------------------------------------------------------------------------------ ---------------- ----------------
Gross margin 52,540 60,542 59,755
Research and development 11,988 12,046 11,381
Sales and marketing 14,697 15,713 15,811
General and administrative 4,563 4,611 4,677
Impairment, integration, and restructuring 1,110 10,011 127
------------------------------------------------------------------------------------------ ---------------- ----------------
Operating income 20,182 18,161 27,759
Other income (expense), net (431) 346 61
------------------------------------------------------------------------------------------ ---------------- ----------------
Income before income taxes 19,751 18,507 27,820
Provision for income taxes 2,953 6,314 5,746
------------------------------------------------------------------------------------------ ---------------- ----------------
Net income $ 16,798 $ 12,193 $ 22,074
----------------- ---------------- ----------------
116
In addition to the above data, you have checked online and found that the current price of Microsoft stock
is $69.08. Using the Microsoft Data, calculate the following ratios for 2016:
7. Much of the HTTP company’s financial records were destroyed in a recent fire. You have recovered
some information and wish to use this information to recreate as many account balances as possible.
You have recovered the following information:
a. Total Assets
b. Sales
c. Profit Margin
d. Common Equity
e. Current Assets
f. Beginning of year Inventory
117
Chapter 2 Homework Solutions
Solution to Problem 1
Retained Earnings Statement for the year ending December 31, 2013
Assets
Cash $16,400
Buildings and Equipment $10,000
Less Accumulated Depreciation $1000
+ Net Buildings And Equipment $9,000
+ Inventory $20,000
= Total Assets $45,400
Liabilities
Bank Loan $10,000
Owners Equity
118
Solution to Problem #2
Assets:
Cash $10,000
Van $30,000
Sales $160,000
- Driver $20,000
- Tour Guide $16,000
- Fuel and Repairs $40,000
- Insurance $12,000
- Depreciation $10,000
- Taxes $6,050
119
C. Statement of Cash Flows for Year Ending December 31, 2019
120
Solution to Problem #3
Assets:
Cash $5,000
Equipment $20,000
Accumulated. Depreciation 0
Net Equipment $20,000
Total Assets $25,000
Sales $600,000
-Cost of Production $400,000
-Sales and Adm. $36,250
- Depreciation $2,000
- Taxes $16,125
121
C. Statement of Cash Flows for Year Ending December 31, 2013
122
E. Balance Sheet as of December 31, 2013
Assets:
Cash $114.843.75
Equipment $20,000
Accumulated. Depreciation $2,000
Net Equipment $18,000
Total Assets $132,843.75
Sales $600,000
-Cost of Production 400,000
-Sales and Admin $36,250
- Depreciation $2,000
- Taxes $16,130
123
H. Statement of Cash Flows for Year Ending December 31, 2014
Assets:
Cash $224,721.25
Equipment $20,000
Accumulated. Depreciation $4,000
Net Equipment $16,000
Total Assets $240,721.25
124
Solution to Problem #4
Assets:
Cash $5000
Equipment $53,000
Accumulated. Depreciation 0
Net Equipment $53,000
Total Assets $58,000
Sales $68,750
Operating Expenses
Driver $10,000
Guide $7,500
Repairs $10,000
Fuel $5,000
Meals $3,750
- Operating Expenses $36,250
- Depreciation $8,000
- Taxes $2,020
125
C. Statement of Cash Flows for Year Ending December 31, 2013
126
E. Balance Sheet as of December 31, 2013
Assets:
Cash $12,090
Equipment $53,000
Accumulated. Depreciation $8,000
Net Equipment $45,000
Total Assets $57,090
Sales $68,750
Operating Expenses
Driver $10,000
Guide $7,500
Repairs $10,000
Fuel $5,000
Meals $3,750
- Operating Expenses $36,250
- Depreciation $8,000
- Taxes $2,120
127
H. Statement of Cash Flows for Year Ending December 31, 2014
Assets:
Cash $19,630
Equipment $53,000
Accumulated. Depreciation $16,000
Net Equipment $37,000
Total Assets $56,630
128
Solution to Problem 5:
$50,000
=
$18,400
= 2.717 times
The P/E ratio tells us that if earnings do not change from their current levels, it will take 2.717
years before the firm will earn back the amount of money required to purchase the company today.
C. Inventory Turnover
𝑆𝑆𝑆𝑆𝑆𝑆𝑆𝑆𝑆𝑆
𝐼𝐼𝐼𝐼𝐼𝐼𝐼𝐼𝐼𝐼𝐼𝐼𝐼𝐼𝐼𝐼𝐼𝐼 𝑇𝑇𝑇𝑇𝑇𝑇𝑇𝑇𝑇𝑇𝑇𝑇𝑇𝑇𝑇𝑇 =
𝐼𝐼𝐼𝐼𝐼𝐼𝐼𝐼𝐼𝐼𝐼𝐼𝐼𝐼𝐼𝐼𝐼𝐼
80,000
𝐼𝐼𝐼𝐼𝐼𝐼𝐼𝐼𝐼𝐼𝐼𝐼𝐼𝐼𝐼𝐼𝐼𝐼 𝑇𝑇𝑇𝑇𝑇𝑇𝑇𝑇𝑇𝑇𝑇𝑇𝑇𝑇𝑇𝑇 = =4
20,000
Inventory Turnover tells is that we have 4 dollars of sales for each dollar of inventory.
𝑆𝑆𝑆𝑆𝑆𝑆𝑆𝑆𝑆𝑆
𝑇𝑇𝑇𝑇𝑇𝑇𝑇𝑇𝑇𝑇 𝐴𝐴𝐴𝐴𝐴𝐴𝐴𝐴𝐴𝐴 𝑇𝑇𝑇𝑇𝑇𝑇𝑇𝑇𝑇𝑇𝑇𝑇𝑇𝑇𝑇𝑇 =
𝑇𝑇𝑇𝑇𝑇𝑇𝑇𝑇𝑇𝑇 𝐴𝐴𝐴𝐴𝐴𝐴𝐴𝐴𝐴𝐴𝐴𝐴
$80,000
𝑇𝑇𝑇𝑇𝑇𝑇𝑇𝑇𝑇𝑇 𝐴𝐴𝐴𝐴𝐴𝐴𝐴𝐴𝐴𝐴 𝑇𝑇𝑇𝑇𝑇𝑇𝑇𝑇𝑇𝑇𝑇𝑇𝑇𝑇𝑇𝑇 =
32,000
So in this case the company had $2.50 of sales for each dollar of assets that it had.
129
E. Return on Assets
𝑁𝑁𝑁𝑁𝑁𝑁 𝐼𝐼𝐼𝐼𝐼𝐼𝐼𝐼𝐼𝐼𝐼𝐼
𝑅𝑅𝑅𝑅𝑅𝑅𝑅𝑅𝑅𝑅𝑅𝑅 𝑜𝑜𝑜𝑜 𝐴𝐴𝐴𝐴𝐴𝐴𝐴𝐴𝐴𝐴𝐴𝐴 =
𝑇𝑇𝑇𝑇𝑇𝑇𝑇𝑇𝑇𝑇 𝐴𝐴𝐴𝐴𝐴𝐴𝐴𝐴𝐴𝐴𝐴𝐴
18,400
𝑅𝑅𝑅𝑅𝑅𝑅𝑅𝑅𝑅𝑅𝑅𝑅 𝑜𝑜𝑜𝑜 𝐴𝐴𝐴𝐴𝐴𝐴𝐴𝐴𝐴𝐴𝐴𝐴 =
32,000
= 0.575
So the company had $0.575 of net income for each dollar of assets.
F. Return on Equity
𝑁𝑁𝑁𝑁𝑁𝑁 𝐼𝐼𝐼𝐼𝐼𝐼𝐼𝐼𝐼𝐼𝐼𝐼
𝑅𝑅𝑅𝑅𝑅𝑅𝑅𝑅𝑅𝑅𝑅𝑅 𝑜𝑜𝑜𝑜 𝐸𝐸𝐸𝐸𝐸𝐸𝐸𝐸𝐸𝐸𝐸𝐸 =
𝐶𝐶𝐶𝐶𝐶𝐶𝐶𝐶𝐶𝐶𝐶𝐶 𝐸𝐸𝐸𝐸𝐸𝐸𝐸𝐸𝐸𝐸𝐸𝐸
18,400
𝑅𝑅𝑅𝑅𝑅𝑅𝑅𝑅𝑅𝑅𝑅𝑅 𝑜𝑜𝑜𝑜 𝐸𝐸𝐸𝐸𝐸𝐸𝐸𝐸𝐸𝐸𝐸𝐸 =
22,000
The company produced $0.8364 of net income for each dollar of common equity employed.
G. Profit Margin
𝑁𝑁𝑁𝑁𝑁𝑁 𝐼𝐼𝐼𝐼𝐼𝐼𝐼𝐼𝐼𝐼𝐼𝐼
𝑃𝑃𝑃𝑃𝑃𝑃𝑃𝑃𝑃𝑃𝑃𝑃 𝑀𝑀𝑀𝑀𝑀𝑀𝑀𝑀𝑀𝑀𝑀𝑀 =
𝑆𝑆𝑆𝑆𝑆𝑆𝑆𝑆𝑆𝑆
$18,400
𝑃𝑃𝑃𝑃𝑃𝑃𝑃𝑃𝑃𝑃𝑃𝑃 𝑀𝑀𝑀𝑀𝑀𝑀𝑀𝑀𝑀𝑀𝑀𝑀 =
$80,000
The firm produces $0.23 of net income for each dollar of sales.
130
H. Debt to Assets
𝑇𝑇𝑇𝑇𝑇𝑇𝑇𝑇𝑇𝑇 𝐿𝐿𝐿𝐿𝐿𝐿𝐿𝐿𝐿𝐿𝐿𝐿𝐿𝐿𝐿𝐿𝐿𝐿𝐿𝐿𝐿𝐿
𝐷𝐷𝐷𝐷𝐷𝐷𝐷𝐷 𝑡𝑡𝑡𝑡 𝐴𝐴𝐴𝐴𝐴𝐴𝐴𝐴𝐴𝐴𝐴𝐴 =
𝑇𝑇𝑇𝑇𝑇𝑇𝑇𝑇𝑇𝑇 𝐴𝐴𝐴𝐴𝐴𝐴𝐴𝐴𝐴𝐴𝐴𝐴
10,000
𝐷𝐷𝐷𝐷𝐷𝐷𝐷𝐷 𝑡𝑡𝑡𝑡 𝐴𝐴𝐴𝐴𝐴𝐴𝐴𝐴𝐴𝐴𝐴𝐴 =
32,000
31.25 percent of the firm's assets have been financed with debt and the remainder is financed
with equity.
I. Equity Multiplier
The company has $1.4545 of total assets for each dollar of common equity.
J. Dupont Analysis
𝑅𝑅𝑅𝑅𝑅𝑅 = 0.8364
131
Solution to Problem 6:
= 7,925,000,000 X $69.08
= $547,459,000,000
Which is what it would cost to buy the entire company. (Note that we are assuming the 7,925
figure indicated for shares outstanding applies to 2016.
= 32.5907 times
The P/E ratio tells us that if earnings do not change from their current levels, it will take 32.5907
years before the firm will earn back the amount of money that we paid for the stock.
85,320,000,000
𝑇𝑇𝑇𝑇𝑇𝑇𝑇𝑇𝑇𝑇 𝐴𝐴𝐴𝐴𝐴𝐴𝐴𝐴𝐴𝐴 𝑇𝑇𝑇𝑇𝑇𝑇𝑇𝑇𝑇𝑇𝑇𝑇𝑇𝑇𝑇𝑇 =
193,694,000,000
So in this case the company had $0.4405 of sales for each dollar of assets that it had in 2016.
D. Inventory Turnover
85,320,000,000
𝐼𝐼𝐼𝐼𝐼𝐼𝐼𝐼𝐼𝐼𝐼𝐼𝐼𝐼𝐼𝐼𝐼𝐼 𝑇𝑇𝑇𝑇𝑇𝑇𝑇𝑇𝑇𝑇𝑇𝑇𝑇𝑇𝑇𝑇 =
2,251,000,000
= 37.90
In this case, the company had $37.90 of sales for each dollar of inventory it had.
E. Return on Assets
𝑁𝑁𝑁𝑁𝑁𝑁 𝐼𝐼𝐼𝐼𝐼𝐼𝐼𝐼𝐼𝐼𝐼𝐼
𝑅𝑅𝑅𝑅𝑅𝑅𝑅𝑅𝑅𝑅𝑅𝑅 𝑜𝑜𝑜𝑜 𝐴𝐴𝐴𝐴𝐴𝐴𝐴𝐴𝐴𝐴𝐴𝐴 =
𝑇𝑇𝑇𝑇𝑇𝑇𝑇𝑇𝑇𝑇 𝐴𝐴𝐴𝐴𝐴𝐴𝐴𝐴𝐴𝐴𝐴𝐴
132
16,798,000,000
𝑅𝑅𝑅𝑅𝑅𝑅𝑅𝑅𝑅𝑅𝑅𝑅 𝑜𝑜𝑜𝑜 𝐴𝐴𝐴𝐴𝐴𝐴𝐴𝐴𝐴𝐴𝐴𝐴 =
193,694,000,000
= 0.0867
The company had $0.0867 of net income for each dollar of assets.
F. Return on Equity
Tells you how much net income you had for each dollar of equity invested in the company.
16,798,000,000
𝑅𝑅𝑅𝑅𝑅𝑅𝑅𝑅𝑅𝑅𝑅𝑅 𝑜𝑜𝑜𝑜 𝐸𝐸𝐸𝐸𝐸𝐸𝐸𝐸𝐸𝐸𝐸𝐸 =
71,997,000,000
This says the company earned $0.2333 for each dollar of common equity utilized.
G. Profit Margin
Profit Margin tells you how much net income the firm is producing for each dollar of sales.
𝑁𝑁𝑁𝑁𝑁𝑁 𝐼𝐼𝐼𝐼𝐼𝐼𝐼𝐼𝐼𝐼𝐼𝐼
𝑃𝑃𝑃𝑃𝑃𝑃𝑃𝑃𝑃𝑃𝑃𝑃 𝑀𝑀𝑀𝑀𝑀𝑀𝑔𝑔𝑖𝑖𝑖𝑖 =
𝑆𝑆𝑆𝑆𝑆𝑆𝑆𝑆𝑆𝑆
$16,798,000,000
𝑃𝑃𝑃𝑃𝑃𝑃𝑃𝑃𝑃𝑃𝑃𝑃 𝑀𝑀𝑀𝑀𝑀𝑀𝑀𝑀𝑀𝑀𝑀𝑀 =
$85,320,000,000
So the firm produces $0.19688 of net income for each dollar of sales.
H. Debt to Assets
Debt to Assets tells which portion of the firm's assets are financed by debt.
𝑇𝑇𝑇𝑇𝑇𝑇𝑇𝑇𝑇𝑇 𝐿𝐿𝐿𝐿𝐿𝐿𝐿𝐿𝐿𝐿𝐿𝐿𝐿𝐿𝐿𝐿𝐿𝐿𝐿𝐿𝐿𝐿
𝐷𝐷𝐷𝐷𝐷𝐷𝐷𝐷 𝑡𝑡𝑡𝑡 𝐴𝐴𝐴𝐴𝐴𝐴𝐴𝐴𝐴𝐴𝐴𝐴 =
𝑇𝑇𝑇𝑇𝑇𝑇𝑇𝑇𝑇𝑇 𝐴𝐴𝐴𝐴𝐴𝐴𝐴𝐴𝐴𝐴𝐴𝐴
121,697,000,000
𝐷𝐷𝐷𝐷𝐷𝐷𝐷𝐷 𝑡𝑡𝑡𝑡 𝐴𝐴𝐴𝐴𝐴𝐴𝐴𝐴𝐴𝐴𝐴𝐴 =
193,694,000,000
So, 62.830 percent of the firm's assets have been financed with debt and the remainder is financed
with equity.
133
I. Payout Ratio
$1.44
𝑃𝑃𝑃𝑃𝑃𝑃𝑃𝑃𝑃𝑃𝑃𝑃 𝑅𝑅𝑅𝑅𝑅𝑅𝑅𝑅𝑅𝑅 = = 0.6725
$2.12
This indicates that we paid 67.25 percent of our net income to the stockholders as a dividend.
J. Current Ratio
𝐶𝐶𝐶𝐶𝐶𝐶𝐶𝐶𝐶𝐶𝐶𝐶𝐶𝐶 𝐴𝐴𝐴𝐴𝐴𝐴𝐴𝐴𝐴𝐴𝐴𝐴
𝐶𝐶𝐶𝐶𝐶𝐶𝐶𝐶𝐶𝐶𝐶𝐶𝐶𝐶 𝑅𝑅𝑅𝑅𝑅𝑅𝑅𝑅𝑅𝑅 =
𝐶𝐶𝐶𝐶𝐶𝐶𝐶𝐶𝐶𝐶𝐶𝐶𝐶𝐶 𝐿𝐿𝐿𝐿𝐿𝐿𝐿𝐿𝐿𝐿𝐿𝐿𝐿𝐿𝐿𝐿𝐿𝐿𝐿𝐿𝐿𝐿
139,660,000,000
𝐶𝐶𝐶𝐶𝐶𝐶𝐶𝐶𝐶𝐶𝐶𝐶𝐶𝐶 𝑅𝑅𝑅𝑅𝑅𝑅𝑅𝑅𝑅𝑅 =
59,357,000,000
= 2.3529
So, the company has $2.3529 dollars of current assets for each dollar of current liabilities.
K. Equity Multiplier
The company has $2.6903 of total assets for each dollar of common equity invested in the firm.
L. Dupont Analysis
𝑅𝑅𝑅𝑅𝑅𝑅 = 0.2333
134
Solution to Problem 7. To solve this problem, you should use the information you have available to
derive the missing data.
500,000
a. We know Return on Assets (ROA) = 0.50 and we know: 𝑅𝑅𝑅𝑅𝑅𝑅 = 0.5 = ,
𝑇𝑇𝑇𝑇𝑇𝑇𝑇𝑇𝑇𝑇 𝐴𝐴𝐴𝐴𝐴𝐴𝐴𝐴𝐴𝐴𝐴𝐴
Solving this equation for the missing variable we learn that: Total Assets = $1,000,000
b. We know that Total Asset Turnover (TAT) = 0.25, and we know that:
𝑆𝑆𝑆𝑆𝑆𝑆𝑆𝑆𝑆𝑆 𝑆𝑆𝑆𝑆𝑆𝑆𝑆𝑆𝑆𝑆
𝑇𝑇𝑇𝑇𝑇𝑇 = 2.5 = =
𝑇𝑇𝑇𝑇𝑇𝑇𝑇𝑇𝑇𝑇 𝐴𝐴𝐴𝐴𝐴𝐴𝐴𝐴𝐴𝐴𝐴𝐴 $1,000,000
d. We know that Return on Equity (ROE) = 0.25, and we know Net Income equals $500,000.
$500,000
𝑅𝑅𝑅𝑅𝑅𝑅 = 0.25 = . Solving for the missing variable tells us Common Equity =
𝐶𝐶𝐶𝐶𝐶𝐶𝐶𝐶𝐶𝐶𝐶𝐶 𝐸𝐸𝐸𝐸𝐸𝐸𝐸𝐸𝐸𝐸𝐸𝐸
$2,000,000
e. We know the Current Ratio (CR) = 4. We know that Current Assets = $25,000.
𝐶𝐶𝐶𝐶𝐶𝐶𝐶𝐶𝐶𝐶𝐶𝐶𝐶𝐶 𝐴𝐴𝐴𝐴𝐴𝐴𝐴𝐴𝐴𝐴𝐴𝐴 𝐶𝐶𝐶𝐶𝐶𝐶𝐶𝐶𝐶𝐶𝐶𝐶𝐶𝐶 𝐴𝐴𝐴𝐴𝐴𝐴𝐴𝐴𝐴𝐴𝐴𝐴
We know that 𝐶𝐶𝐶𝐶 = 4 = = .
𝐶𝐶𝐶𝐶𝐶𝐶𝐶𝐶𝐶𝐶𝐶𝐶𝐶𝐶 𝐿𝐿𝐿𝐿𝐿𝐿𝐿𝐿𝐿𝐿𝐿𝐿𝐿𝐿𝐿𝐿𝐿𝐿𝐿𝐿𝐿𝐿 $25,000
Solving for the missing variable we learn that Current Assets = $100,000
𝑆𝑆𝑆𝑆𝑆𝑆𝑆𝑆𝑆𝑆 $2,500,000
We know that 𝐼𝐼𝐼𝐼 = 3 = =
𝐵𝐵𝐵𝐵𝐵𝐵𝐵𝐵𝐵𝐵𝐵𝐵𝐵𝐵𝐵𝐵𝐵𝐵 𝑜𝑜𝑜𝑜 𝑌𝑌𝑌𝑌𝑌𝑌𝑌𝑌 𝐼𝐼𝐼𝐼𝐼𝐼𝐼𝐼𝐼𝐼𝐼𝐼𝐼𝐼𝐼𝐼𝐼𝐼 𝐵𝐵𝐵𝐵𝐵𝐵𝐵𝐵𝐵𝐵𝐵𝐵𝐵𝐵𝐵𝐵𝐵𝐵 𝑜𝑜𝑜𝑜 𝑌𝑌𝑌𝑌𝑌𝑌𝑌𝑌 𝐼𝐼𝐼𝐼𝐼𝐼𝐼𝐼𝐼𝐼𝐼𝐼𝐼𝐼𝐼𝐼𝐼𝐼
Solving for the missing variable we learn that Beginning of the Year Inventory = $833,333.33
135
Chapter 4
Forecasting Financial Statements
136
Forecasting Financial Statements
Step 2: Identify the industry that your firm is in. Sometimes there is not a
report on the exact industry that you wish to be in . In these cases,
you should get the report for the industry you believe is the closest
to that in which your firm will operate. Suppose our company is a
Retail Jewelry store.
Step 3: Obtain the reports for the industry that you will operate in and use
this report to develop your financial statements.
137
Sample Industry Data
Jewelry Industry
Historical Average Current Data Sorted by Sales Current Data Sorted by Total Assets
Sales 0-
Data 1M 1M-5M T.A. 0-500 500-2M
% Assets % % % %
10.1 Cash and Equivalents 9.5 11.4 10.4 9.5
8.9 Trade Receivables 10.5 9.5 9.1 10.1
25.5 Inventory 23.5 25.9 31.1 24.1
0.1 All Other Current 0.2 0.4 0.1 0.2
44.6 Total Current 43.7 47.2 50.7 43.9
35.4 Fixed Assets (Net) 41.1 38.8 35.5 41.5
12.5 Intangibles (Net) 2.2 1.5 1 0.5
7.5 All Other Non-Current 13.0 12.5 12.8 14.1
100 Total 100 100 100 100
Liabilities
12.5 Notes Payable-Short Term 10.5 9.5 11.5 8.5
10.7 Current Mat. LTD 4.5 3.5 2.5 5.5
15.3 Trade Payables 17.7 14.4 15.3 15.1
0.5 Income Taxes payable 0.9 0.7 0.2 1.3
1.2 All Other current 14.5 11.4 8.5 7.2
40.2 Total Current 48.1 39.5 38 37.6
35 Long Term Debt 28.5 45.3 35.5 38.4
0.9 Deferred Taxes 0.5 0.8 0.2 1.1
1.5 All Other Non-Current 4.1 2.4 3.1 3.7
22.4 Net Worth 18.8 12 23.2 19.2
100 Total Liab. & Net Worth 100 100 100 100
Income Data
100 Net Sales 100 100 100 100
35.5 Gross Profit 36.2 37.3 35.5 40.1
28.2 Operating Expenses 30.1 27.5 31.2 28.5
7.3 Operating Profit 6.1 9.8 4.3 11.6
4.4 All Other Expenses 3.3 4.1 2.4 2.1
2.9 Profit Before Taxes 2.8 5.7 1.9 9.5
Ratios
4.3 Sales/Total Assets 3.8 4.6 4.0 4.1
138
Step 4: Create your Financial Statements
Part A: Decide on a size for your firm. You may do this based on
Sales or Total Assets.
Part B: Select the column that corresponds with the size of your
firm. In this case, our company will have sales between $0
and $1,000,000, so we should select the first column of
current data sorted by sales for our analysis.
Sales $400,000
Cost of Goods Sold (Sales – Gross Profit) $255,200
Gross Profit (36.2%) $144,800
Operating Expenses (30.1%) $120,400
Earnings Before Interest and Taxes (6.1%)
(Operating Profit) $24,400
Interest Expense (3.3%)
(Other Expenses) $13,200
Earnings Before Tax (2.8%) $11,200
139
Part D: Estimate the Assets of the Company. Because you have
estimated the sales of the company, you can also estimate the
Total Assets of the Company. To do this, observe the
Sales/Total Assets ratio reported in RMA. In the case of the
Jewelry industry the ratio is 3.8. (Note that when using
actual RMA data there will be three numbers for each ratio.
You will generally want to select the middle number, which is
the ratio for the middle 1/3 of companies)
140
Part E: Estimate the Balance Sheet of the Company
Assets
Cash and Equivalents 9.5% $
Trade Receivables 10.5% $
Inventory 23.5% $
Other Current Assets 0.2% $
Total Current Assets 43.7% $
Fixed Assets (Net) 41.1% $
Intangibles 2.2% $
Other Non-Current Assets 13.0% $
Total Assets 100% $
Liabilities
*. Note that RMA refers to Net Worth. This is exactly the same thing as
Common Stock and Retained Earnings in the language we have
learned in class.
141
With this information, and knowing that our total assets will equal
$105,263.16. We can estimate our balance sheet in this way:
Assets
Cash and Equivalents 9.5% $10,000.00
Trade Receivables 10.5% $11,052.63
Inventory 23.5% $24,736.84
Other Current Assets 0.2% $210.53
Total Current Assets 43.7% $46,000.00
Fixed Assets (Net) 41.1% $43,263.16
Intangibles 2.2% $2,315.79
Other Non-Current Assets 13.0% $13,684.21
Total Assets 100% $105,263.16
Liabilities
If you are starting a business and your estimates differ substantially from
what you obtain from this approach, you should explain the difference.
142
Now suppose that instead of having an estimate of sales, you know only that
you have $40,000 to invest in the business. Suppose further that you want to
retain full ownership of the business. How can you estimate the financial
statements in this case?
In this case we begin with the balance sheet and work our way back to the
income statement/
Starting again from the RMA information. However, in this case, we will
use the current data sorted by assets in RMA to make our estimates. To do
this, we must first make an estimate of the total assets of the firm based on
our plan to invest $40,000 into the company. In this case, our firm will
certainly have below $500,000 of total assets, so we should use the first
column of the data.
Assets
Cash and Equivalents 10.4% $
Trade Receivables 9.1% $
Inventory 31.1% $
Other Current Assets 0.1% $
Total Current Assets 50.7% $
Fixed Assets (Net) 35.5% $
Intangibles 1.0% $
Other Non-Current Assets 12.8% $
Total Assets 100% $
Liabilities
143
First, estimate the total assets:
= $40,000/.232= $172,413.79
You can then compute the remaining values as a percentage of the total.
Assets
Cash and Equivalents 10.4% $17,931.0
Trade Receivables 9.1% $15,689.65
Inventory 31.1% $53,620.69
Other Current Assets .1% $172.41
Total Current Assets 50.7% $87413.79
Fixed Assets (Net) 35.5% $61,206.90
Intangibles 1.0% $1,724.14
Other Non-Current Assets 12.8% $22,068.97
Total Assets 100% $172,413.79
Liabilities
144
Next, you need to estimate the sales of the firm. We can do this based
on the Sales/Total Assets Ratio, just as we did before. In this case, we
know the total assets ($172,413.79) and must estimate the sales of the
firm. We also know that other firms have a Sales/total asset ratio of
4.0. Thus:
4.0 = Sales/$172,413.79
Using this sales figure and our industry information, we can estimate
our income statement as follows:
Sales $689,655.16
Cost of Goods Sold (Sales – Gross Profit) $444,827.58
Gross Profit (35.5%) $244,827.58
Operating Expenses (31.2%) $215,172.41
Earnings Before Interest and Taxes (4.3%)
(Operating Profit) $29,655.17
Interest Expense (2.4%)
(Other Expenses) $16,551.72
Earnings Before Tax (1.9%) $13,103.45
145
Chapter 4
Forecasting Financial Statements
Homework Assignment
146
1. You wish to start a firm that will manufacture Lawn Mowers. You have
classified your firm as being in the Lawn Mower Manufacturing Business. You
have obtained the Risk Management Associates (RMA) Industry Financial
Statement Information on the following pages.
Using the RMA industry financial information and your sales estimate of
$2,000,000 per year, as we did in class, estimate the financial statements of your
firm.
2. You wish to start a firm that will manufacture Lawn Mowers. You have
classified your firm as being in the Lawn Mower Manufacturing Business. You
have obtained the Robert Morris and Associates (RMA) Industry Financial
Statement Information on the following pages.
Using the RMA industry financial information and your plan to invest $150,000
from your own pocket into the business, as we did in class, estimate the financial
statements of your firm.
147
Sample Industry Data
Historical Average Current Data Sorted by Sales Current Data Sorted by Total Assets
Sales 0-
Data 1M 1M-5M T.A. 0-500 500-2M
% Assets % % % %
5.5 Cash and Equivalents 6.5 4.5 7.3 5.4
35.1 Trade Receivables 28.7 25.8 30.1 28.4
25 Inventory 33.3 29.4 30.1 36.5
0.3 All Other Current 0.1 0.3 0 0.1
65.9 Total Current 68.6 60 67.5 70.4
30.1 Fixed Assets (Net) 28.7 33.2 26.3 27.1
0.2 Intangibles (Net) 0.1 0.6 0.3 0.2
3.8 All Other Non-Current 2.6 6.2 5.9 2.3
100 Total 100 100 100 100
Liabilities
5.4 Notes Payable-Short Term 6.1 8.2 7.2 6.9
4.5 Current Mat. LTD 3.6 5.5 4.2 5.6
25.5 Trade Payables 27.6 24.6 22.1 28.6
0.3 Income Taxes payable 0.2 0.1 0.4 0.3
0.1 All Other current 0.3 0.5 0.7 0.2
35.8 Total Current 37.8 38.9 34.6 41.6
21.5 Long Term Debt 33.1 24.2 25.1 31.6
4.6 Deferred Taxes 5.1 3.3 4.9 5.7
5.7 All Other Non-Current 7.9 4.3 5.1 6.3
32.4 Net Worth 16.1 29.3 30.3 14.8
100 Total Liab. & Net Worth 100 100 100 100
Income Data
100 Net Sales 100 100 100 100
17.1 Gross Profit 20.4 22.3 17.5 23.5
15.1 Operating Expenses 11.5 14.5 14.5 18.3
2 Operating Profit 8.9 7.8 3 5.2
0.2 All Other Expenses 0.1 0.1 0.1 0.1
1.8 Profit Before Taxes 8.8 7.7 2.9 5.1
Ratios
2.4 Sales/Total Assets 2.1 3.2 2.7 2.9
148
Chapter 4 Homework Solutions
Question 1
In this case, the following income statement figures are relevant from the RMA figures
Sales $2,000,000
Cost of Goods Sold (Sales – Gross Profit) $1,554,000
Gross Profit (22.3%) $446,000
Operating Expenses (14.5%) $290,000
Earnings Before Interest and Taxes (7.8%)
(Operating Profit) $156,000
Interest Expense (0.1%)
(Other Expenses) $2,000
Earnings Before Tax (7.7%) $154,000
149
Estimate the Balance Sheet of the Company
Assets
Cash and Equivalents 4.5% $28,125
Trade Receivables 25.8% $161,250
Inventory 29.4% $183,750
Other Current Assets 0.3% $1,875
Total Current Assets 60.0% $375,000
Fixed Assets (Net) 33.2% $207,500
Intangibles 0.6% $3,750.00
Other Non-Current Assets 6.2% $38,750
Total Assets 100% $625,000
Liabilities
150
Question 2:
In this case, we only know that we will invest $150,000 of our own money into the
business.
Starting from the RMA information for the balance sheet with total assets between zero
and $500,000
= $150,000/.303= $495,049.51
You can then compute the remaining values as a percentage of the total.
Assets
Cash and Equivalents 7.3% $36,138.61
Trade Receivables 30.1% $149,009.90
Inventory 30.1% $149,009.90
Other Current Assets .0% $0
Total Current Assets 67.5% $334,158.42
Fixed Assets (Net) 26.3% $130,198.02
Intangibles 0.3% $1,485.15
Other Non-Current Assets 5.9% $29,207.92
Total Assets 100% $495,049.51
Liabilities
151
To estimate the sales of the firm. We do this based on the Sales/Total Assets
Ratio, just as we did before. In this case, we know the total assets ($495,049.51)
and must estimate the sales of the firm. We also know that other firms have a
Sales/total asset ratio of 2.7. Thus:
2.7 = Sales/$495,049.51
Using this sales figure and our industry information, we can estimate our income
statement as follows:
Sales $1,336,633.68
Cost of Goods Sold (Sales – Gross Profit) $1,102,722.79
Gross Profit (17.5%) $233,910.89
Operating Expenses (14.5%) $193,811.88
Earnings Before Interest and Taxes (3.0%)
(Operating Profit) $40,099.01
Interest Expense (0.1%)
(Other Expenses) $1,336.63
Earnings Before Tax (2.9%) $38,762.38
152
CHAPTER 5
153
Chapter 5: The Financial
Environment: Markets, Institutions,
and Interest Rates
In this chapter you will learn about financial markets, how and
where firms can obtain money, Interest Rates, and Theories of
short term versus long-term borrowing.
Financial Markets
Definition of a Market
Classification of Markets
Physical Assets: Something that you can lay your hands on.
a. Art
b. Real Estate
c. Cars
a. Currency
b. Shares of Stocks
c. Deposits in a Commercial Bank
154
Classification of Financial Markets
155
Methods for Transferring Capital From Savers to Businesses’
1. Direct Transfers
Stocks or Bonds
-------------------------------------
Business Savers
----------------------------------
Money
156
Primary Markets
Three Primary Ways Capital is Transferred Between Savers and
Borrowers
Direct Transfer
Indirect Transfers
157
Financial Intermediaries
1. Commercial Banks
4. Credit Unions
6. Mutual Funds
7. Pension Funds
158
Changing Role of Financial Institutions
Secondary Markets
Once a security has initially been issued, it frequently trades
on a secondary market.
159
The Cost of Money
Borrowed Money
Equity Investments
The rent is the return that investors expect to receive in the form of
dividends and capital gains.
1. Production Opportunities
The more people like to consume today, the more firms that
wish to borrow money will have to pay to get those people
not to consume but rather to save.
3. Risk
160
4. Expected Inflation
6. Federal Deficits
161
Components of the Interest Rate
A. The Real Risk Free Rate -- K*
The reward to investors for postponing consumption
B. Inflation Premium -- IP
The additional interest required to compensate investors for
the loss of purchasing power due to increasing prices.
D. Liquidity Premium -- LP
The additional interest required to hold investments that can
not be quickly and easily converted to cash.
K = K* + IP + DRP + LP + MRP
Or
K = Krf + DRP + LP + MRP
162
The Yield Curve or the Term Structure of Interest
Rates
The yield curve shows the relationship between interest rates and
time to maturity.
We want to see how the interest rate changes with the amount of
time that we borrow money for.
2012 inflation 2%
2013 inflation 4%
IP1 + IP 2 + ...IPn
IP =
N
IP 2012 + IP 2013
IP =
2
2+4
IP = =3
2
163
Three Candidate Shapes for the Yield Curve
Interst Rates
1.5
0.5
0
1 year 5 year 10 year 20 year 30 year
Interst Rates
Interst Rates
164
Theories of the shape of the Yield Curve and implications for firm
behavior
Method 2: Borrow Money for 3 years and pay off the loan.
Then Borrow Money a second time for 3 years.
165
Theories
1. Expectations Theory
3. Market Segmentation
Implies that it pays to look for bargains for long term or short
term borrowing.
166
Predicting Interest Rates
(1 + S 0 − L) L = (1 + S 0 − N ) n1 (1 + X n− L ) n 2
(1 + S 0 − L ) L
n1
= (1 + X n− L ) n 2
(1 + S 0 − N )
and
( (1 + S 0 − L) L
(1 + S 0 − N ) n1
) 1
n2
− 1 = X n− L
167
Suppose we wish to predict the 16 year interest rate that will
prevail 14 years from today.
|---------------------------------------------------------------|
0 30 year
Rate = 6.4 percent
|-------------------------------|--------------------------------|
0 14 Years 30 year
168
We solve for Xn-L as follows
( (1 + S 0 − L ) L
(1 + S 0 − N ) n1
) 1
n2
− 1 = X n− L
( (1 + 0.064) 30
(1 + 0.051)14
) 1
16
− 1 = X n− L
( 6.4306
2.0065
) 1
16
−1 = X n− L
1
X n−L = (3.205) − 1 = 3.2050.0625 − 1
16
0.0755 = X n− L
169
Chapter 5 Homework and Solutions
170
Chapter 5 Homework
2. Today is January 1, 2014. Suppose that investors expect annual future inflation rates
to be as follows:
Assume that the real risk-free rate of interest will remain at a constant 2 percent
per year for the foreseeable future. Also assume that maturity risk premiums on
Treasury Bonds and Bills are 0.10 percent for 1-year securities. For securities
having maturities in more than one year, maturity risk premiums increase 0.1
percent for each year to maturity, up to a limit of 1.0 percent on 10-year or
longer term T-bonds. Calculate the interest rate on 1,2,3,4,5, 10 and 20-year
Treasury Securities, and plot the yield curve.
171
3. You are given the following data. Plot a yield curve based on this data.
Term Rate
6 month 5.4%
1 year 5.6%
2 year 5.8%
3 year 5.9%
4 year 6.0%
5 year 6.1%
10 year 6.3%
20 year 6.5%
30 year 6.6%
4. Suppose that at the beginning of 2014, the expected inflation rate for 2014 was 3
percent, for 2015, 4 percent, for 2016, 2 percent and for 2017 and thereafter, 1
percent. Assume that there is no default risk premium, liquidity premium or
maturity risk premium.
a. What is the average expected inflation rate over the 5-year period 2014-2018?
b. What average nominal risk-free interest rate would, over the 5-year period
from 2014 through 2018, be expected to produce a 3 percent real risk-free rate
of return on 5-year Treasury Securities?
c. Now assume a real rate of 2 percent and a maturity risk premium which starts
at 0.1 percent for one-year securities and increases by 0.1 percent each year.
Furthermore, assume that inflation will occur as indicated earlier in the
problem. Estimate the interest rate in January 2014 on bonds that mature in
1,2,3,4,5, 10 and 20 years, and draw a yield curve based on these data.
5. Suppose that the current rate of interest on 30 year fixed rate mortgages is 6.5 percent.
Suppose that the current rate on 15 year fixed rate mortgages is 6 percent. Using
the techniques that we learned in class, predict what the 15 year mortgage rate
will be 15 years from today.
172
Solutions to Homework for Chapter 5:
Question 1
a. A money market is a financial market for debt securities with a maturity of less
than one year.
Capital markets are markets for long-term debt securities (with a maturity of more
than one year) and corporate stocks.
b. Primary markets are markets in which newly issued securities are traded for the
first time. That is the company is selling securities to investors. Secondary
markets are markets where already existing securities are traded between two
different investors.
e. A mutual fund is a company that pools the funds of many investors together to
make investments.
f. Production opportunities are the returns that you will receive from investing in
productive assets, such as an oven for your restaurant. The time preference for
consumption says that people would rather consume today than at some future
point in time.
g. The real risk-free rate of interest is the reward that you receive for investing today
and consuming later, rather than consuming today. The nominal interest risk-free
rate of interest is the real risk-free rate of interest plus an additional interest
component called an inflation premium which
compensate for inflation.
h. The inflation premium is the additional interest added to the real risk free rate to
compensate the investor for the loss of purchasing power caused by inflation.
i. Default risk is the risk that a borrower will not pay the interest on the loan, or will
not pay the principal on the loan as it comes due.
j. Liquidity refers to the ability to convert an asset to cash quickly and at a fair
price. The liquidity premium is the additional interest rate to compensate
investors for investing in something that can not be quickly converted to cash.
173
k. The term structure of interest rates is the relationship between the return that you
will receive if you invest in a bond (if you lend the US government money for
example), and the length of time that you make the loan for. A yield curve is a
graph that shows the relationship between the interest rate and the amount of time
that the money is committed to an investment for.
l. A normal yield curve is a yield curve that slopes upward. It is called normal
because the yield curve normally takes this shape. An abnormal (or inverted)
yield curve is a yield curve that slopes downward. A third shape of the yield
curve is a flat yield curve. It occurs when the interest rate you receive on the
investment does not depend on the amount of time you make the loan for.
m. The expectations theory says that expectations about future inflation determines
the relationship between short term interest rates and long term interest rates.
n. The liquidity preference theory says that borrowers would rather borrow for the
long term. Lenders would rather lend for the short term. In order to get lenders to
make long term loans, they must be given an interest incentive called a liquidity
premium.
o. The market segmentation theory says that there are groups of people who like to
lend and borrow long term. They do not consider investing in the short-term
market even if there is a great interest rate advantage to doing so. There are
different groups of people who like to lend and borrow short term. They do not
consider investing in the long-term market even if there is a great interest rate
advantage to doing so. Thus there is no particular relationship between interest
rates for long term investments and interest rates for short-term investments.
Question 2
First, we must calculate the average rate of inflation that we should expect to exist over
each of the time periods under consideration.
Period Inflation for this period Average inflation over the past n
years.
1 year 7 =7/1 = 7
2 years 6 = (7 + 6)/2 = 6.5
3 years 3 = (7 + 6 + 3)/3 = 5.33
4 years 3 = (7 + 6 + 3 + 3)/4 = 4.75
5 years 3 = (7 + 6 + 3 +3 + 3)/5 = 4.4
10 years 3 = (7 + 6 + 3 + 3 + 3 + 3 + 3 + 3 + 3 + 3)/10 = 3.7
20 years 3 = (7 + 6 + 3*18)/20 = 3.35
174
Now we can calculate the interest rate. We can use the formula k = k* + IP + DRP + LP
+ MRP
Now you are ready to graph the yield curve. In order to do this you will make a graph
with the years on the bottom axis and k on the upper axis.
10
6
Rate
5 Seri…
0
1 2 3 4 5 10 20
Term
175
Question 3
6.8
6.6
6.4
6.2
6
Rate
Series2
5.8
5.6
5.4
5.2
5
0.5 1 2 3 4 5 10 20 30
Term
Question 4
Part a: to do this part we must simply calculate the average level of inflation for the next
5 years as follows:
Part b: In order to calculate this part, we can use the formula k = k* + IP + DRP + LP +
MRP. Because there is no DRP, LP or MRP, in this case, the formula is also equal to Krf
= K* + IP. We want to solve for k in the formula (or equivalently Krf). For this part we
will assume that there is no default risk premium, liquidity premium or maturity risk
premium.
176
Year k* + IP + DRP + LP + MRP = k
5 3 2.2 0 0 0 = 5.2
Part C:
To do this part we must first calculate the average inflation for each of the time periods
under consideration.
Period Inflation for this period Average inflation over the past n
years.
1 3 3.0
2 4 3.5
3 2 3.0
4 1 2.5
5 1 2.2
10 1 1.6
20 1 1.3
Now we can calculate the interest rate. We can use the formula k = k* + IP + DRP + LP
+ MRP
177
Question 4: Yield Curve
4
Rate
3 Serie…
0
1 2 3 4 5 10 20
Term
5.
( (1 + S 0 − L) L
(1 + S 0 − N ) n1
) 1
n2
− 1 = X n− L
((1(1++00.065
.06)
)
) 30
15
1
15
− 1 = X n− L
6.6144 1
( )15 − 1 = 𝑋𝑋𝑛𝑛−𝐿𝐿
2.3966
0.0700 = X n − L
178
1
CHAPTER 6
179
2
180
3
181
4
182
5
CALCULATING RETURNS
A. Expected Returns
I. Subjective Method:
183
6
For Stock A
For Stock B
184
7
Return
Year Kh
2007 30%
2008 20%
2009 13%
2010 -10%
2011 -20%
2012 39%
185
8
Kh
K K avg
N
Return
Year Kh
2007 30%
2008 20%
2009 13%
2010 -10%
2011 -20%
2012 39%
Kh 72
K K avg = 12.0%
N 6
186
9
B. REALIZED RETURNS
95 80 5
Kr .25 or 25%
80
187
10
188
11
Kreq 5 1.2(14 5)
= 15.8%
Suggested Measures:
a. Price to Book Ratio
b. Industry
c. Size of the Firm
189
12
A. Expected Returns
190
13
Kh
Kh1 Kh2
2007 30% 29%
2008 20% 31%
2009 13% 30.58%
2010 -10% -17%
2011 -20% -16%
2012 39% 14.42%
191
14
Kh
Kh1 Kh2 Khp
2007 30% 29% .5(30) + .5(29) = 29.5
2008 20% 31% .5(20) + .5(31) = 25.5
2009 13% 30.58% .5(13) + .5(30.58) = 21.79
2010 -10% -17% .5(-10) + .5(-17) = -13.5
2011 -20% -16% .5(-20) +.5(-16) = -18
2012 39% 14.42 .5(39) + .5(14.42) = 26.71
Khp
Kp K avgp
N
192
15
Suppose we buy 100 shares of IBM stock for $50 per share
and 200 shares of Microsoft stock for $70 per share. During
the year IBM pays a $1.00 per share dividend and Microsoft
pays a $1.50 per share dividend. At the end of the year the
price of IBM stock is $51 per share and the price of
Microsoft stock is $73 per share. What is the realized rate of
return on this portfolio.
193
16
= 0.0579 or 5.79%
194
17
Also Acceptable
Kreqp .7(0.158) .3(0.176) 0.1634
195
18
Measures of Risk
1. Standard Deviation
2. Variance
3. Coefficient of Variation
Portfolio
1. Standard Deviation
2. Variance
3. Coefficient of Variation
4. Beta
196
19
Note: ( )
2 2
Three inputs are required to solve the formula ki, k and pi
197
20
198
21
(kh K avg ) 2
S = standard deviation
N 1
2
2 (kh K avg )
S = variance
N 1
Recall that
199
22
Return
Year Kh
2007 30%
2008 20%
2009 13%
2010 -10%
2011 -20%
2012 39%
Kh 72
K K avg = 12.0%
N 6
200
23
(kh K avg ) 2 2
S
= standard deviation, S 2
(kh K avg )
= variance
N 1 N 1
2 2,626
2 (kh K avg )
Variance = S 525.2
N 1 6 1
(kh K avg ) 2
Standard Deviation = S 525.2 22.92
N 1
201
24
For a portfolio
202
25
= 12+22.92 = 34.92
= 12-22.92 = -10.92
= 12+2(22.92) = 57.84
= 12-2(22.92) = -33.84
= 12+3(22.92) = 80.76
= 12-3(22.92) = -56.76
203
26
204
27
S
CV or
K K avg
For a Portfolio
Sp
CVp
K avgp
10
B 100% 10 0 .1
100
205
28
(khp K avgp) 2
Sp = Standard Deviation
N 1
(khp K avgp) 2
S 2p = Variance
N 1
206
29
207
30
(khp K avgp) 2
Standard Deviation = Sp 470.19 21.69
N 1
Note: See page 8 for computation details of K avg See page 23 for computations details of S. See
page 14 for computation details of K avgp Note that the expected return on this portfolio is equal to
the expected returns on the two stocks in the portfolio. The standard deviation on the portfolio,
however; is lower than the standard deviation on either stock in the portfolio.
208
31
209
32
1. Covariance
^ ^
ij
[( Ki Ki ) X ( Kj Kj )]
N 1
210
33
2. Correlation Coefficient:
ij
ij
iXj
211
34
212
35
Beta
213
36
Beta of 1:
Beta > 1:
Beta < 1:
Recall that Beta was an input into the Capital Asset Pricing
Model. We used the Capital Asset Pricing model to compute
the required rate of return on a stock.
214
37
215
38
Chapter 6 Homework
1. Given the following subjective information, compute the expected return, the standard
deviation and the coefficient of variation.
2. Given the following historical information and assuming that the past is a good
estimate of what will happen in the future, compute the expected return, the standard
deviation and the coefficient of variation.
3. You bought a stock for 35 dollars at the beginning of the year. During the year the
stock paid a dividend of $2.00. You sold the stock at the end of the year for $36.
What is your realized rate of return on this stock?
4. You will invest in a portfolio. You will invest 60 percent of your funds in stock 1 and
40 percent of your funds in stock 2. You have the following historical information
about the returns on the two stocks. What is the expected return on the portfolio
assuming that what has happened in the past is a good estimate of what will happen in
the future. Calculate the standard deviation and coefficient of variation of the
portfolio.
2008 10 % 15 %
2009 12 % 11 %
2010 22 % 40 %
2011 10 % 20 %
2012 15 % 15 %
5. You have checked in Value Line and discovered the Beta for Wall Mart stock is 1.1.
You expect the return on the market next year will be 15 percent. You have checked
with the bank and they will give you 6 percent interest if you put your money in a risk
free account with them. Using the SML, what is the required rate of return on this
stock?
216
39
6. You are interested in making an investment that has a 20 percent expected return and a
5 percent standard deviation. Compute a 68, 95, and 99.75 percent confidence
interval for the realized return on this stock.
217
40
1.
State Pi Ki PiKi − ( − ) ( − )
Boom .3 25 % 7.5 7.4 54.76 16.428
Normal .6 15 % 9 -2.6 6.76 4.056
Recession .1 11 % 1.1 -6.6 43.56 4.356
∑ = = 17.6 ∑( − ) = 24.84
= ( − )
= √24.84 = 4.98
4.98
= = 0.2829
17.6
∑( ℎ − ) = 1946
∑( ℎ − )
= = √486.5 = 22.06
−1
22.06
= = = 1.765
12.5
218
41
4.
Year Kh1 Kh2 Khp
Standard Deviation.
2008 12 -4.36 19
2009 11.6 -4.76 22.66
2010 29.2 12.84 164.87
2011 14 -2.36 5.57
2012 15 -1.36 1.85
∑( ℎ − ) = 213.95
∑( ℎ − )
= 53.49
−1
∑( )
= = 7.31
7.31
= = = 0.4468
16.36
6.
219
42
CI 68 K 1
= 20+5 = 25
= 20-5 = 15
= 20+2(5) = 30
= 20-2(5) = 10
= 20+3(5) = 35
= 20-3(5) = 5
We can be 99.75 percent confident that our realized return will be between
35% and 5%.
220
Chapter 7
Business and Financial Risk
221
Chapter 7: Business and Financial Risk
B. Operating Leverage
C. Financial Leverage
D. Combined Leverage
Leverage involves using fixed costs to magnify the potential return to a firm.
Consider a company that sells “Meatloaf” impersonator MP3 songs for $2.00 each.
The company is faced with the decision to rent or purchase the servers that will
deliver the music to its customers. Renting the delivery will cost $1.50 per song
sold in variable costs. In addition, if we rent the equipment our fixed operating
costs will be $25,000 per year. If we purchase the equipment, our variable costs
per song will decline to $1.00 per song. However, our fixed operating costs will
increase to $175,000 per year. The company is in a 30 percent tax bracket. The
company currently borrows $300,000 at a 10 percent interest rate. We wish to
compare these two alternatives:
Note: fixed costs remain constant regardless of how many units you sell. Variable
costs depend upon the number of units that you sell. If I open a store to sell t-shirts
in the mall, variable costs would include what we paid to acquire the t-shirts from
our supplier. Fixed costs would include the price of the Yellow Pages Add that I
take out in the newspaper.
222
To begin the analysis, compute the income statements under each scenario. To
begin with, assume that we will sell 500,000 songs per year.
Rent Buy
Sales $1,000,000 $1,000,000
Variable Costs $750,000 $500,000
Fixed Costs $25,000 $175,000
EBIT $225,000 $325,000
I $30,000 $30,000
EBT $195,000 $295,000
Tax $58,500 $88,500
NI $136,500 $206,500
Clearly, in this analysis, the company would be better off by purchasing the
equipment.
However, what happens if you have overestimated the number of people who want
to download the song “Paradise by the Dashboard Light” Instead of selling
500,000 songs, you only sell 250,000 songs. How does this fact change the above
analysis?
In this case, the company would be better off by renting the equipment.
223
Break Even Analysis:
Operating Break Even Analysis tells us the number of units we must sell in order
to have an Earnings Before Interest and Taxes of $0. Alternatively, it is the
number of units we must sell to cover our fixed and variable operating costs.
Notation:
Q = Quantity Sold
P = Price per Unit
V = Variable Cost per Unit
F = Total Fixed Costs
QBE = Break Even Quantity
𝑭𝑭
𝑸𝑸𝑸𝑸𝑸𝑸 =
𝑷𝑷 − 𝑽𝑽
224
If we rent the equipment:
25,000
𝑄𝑄𝑄𝑄𝑄𝑄 = = 50,000 𝑈𝑈𝑈𝑈𝑈𝑈𝑈𝑈𝑈𝑈
2.00 − 1.50
That is: If we rent the equipment, we will have a zero EBIT if our sales are
50,000 units. At sales levels above 50,000 units, we will earn a positive
EBIT. At sales levels below 50,000 units, we will earn a negative EBIT.
The final result is that if we purchase the machine we make a larger profit if our
sales are good. However, if we rent the machine, we make a larger profit if our
sales are poor.
225
Business Risk and Operating Leverage
𝑄𝑄(𝑃𝑃 − 𝑉𝑉)
𝐷𝐷𝐷𝐷𝐷𝐷 =
𝑄𝑄(𝑃𝑃 − 𝑉𝑉) − 𝐹𝐹
Renting Equipment
Purchasing Equipment
226
Financial Risk and Degree of Financial Leverage
Financial risk is the risk associated with the way that the firm is financed. The
Degree of Financial Leverage (DFL) tells us how much our Earnings Before Taxes
(EBT) changes when EBIT change by a small amount.
Consider the example above with 500,000 units of sales and purchasing the
equipment. In one instance we finance the company with $300,000 of debt that
costs 10 percent interest. Alternatively, we are considering financing the firm with
$100,000 of debt at 10 percent interest and obtaining the remainder of the needed
funds from stock issues.
$300,000 $100,000
Debt Debt
Sales $1,000,000 $1,000,000
Variable Costs $500,000 $500,000
Fixed Costs $175,000 $175,000
EBIT $325,000 $325,000
I $30,000 $10,000
EBT $295,000 $315,000
Tax $88,500 $94,500
NI $206,500 $220,500
𝐸𝐸𝐸𝐸𝐸𝐸𝐸𝐸
𝐷𝐷𝐷𝐷𝐷𝐷 =
𝐸𝐸𝐸𝐸𝐸𝐸𝐸𝐸 − 𝐼𝐼
227
With $300,000 of debt
325,000
𝐷𝐷𝐷𝐷𝐷𝐷 =
325,000 − 30,000
DFL =1.1017
This DFL can be interpreted as follows: If EBIT goes up by 1 percent, EBT will
go up by 1.1017 percent. If EBIT goes down by 1 percent, EBT will go down by
1.1017 percent.
DFL = 1.0317
Notes:
228
Combined Leverage
Combined Leverage tells us how much our Earnings Before Taxes (EBT) changes
when sales change by a small amount. It measures the combined effects of all
fixed costs.
$300,000
Debt
Sales $1,000,000
Variable Costs $500,000
Fixed Costs $175,000
EBIT $325,000
I $30,000
EBT $295,000
Tax $88,500
NI $206,500
Recall further that we have previously calculated the DOL for this scenario to be:
Recall also that we have previously calculated the DFL for this
scenario to be: 1.1017.
This figure can be interpreted as: If sales increase by 1 percent, EBT will increase
by 1.6949 percent. If sales decrease by 1 percent, EBT will decrease by 1.6949
percent.
229
Chapter 7:
Business and Financial Risk
Homework Assignment and Solution
230
Chapter 7: Business and Financial Risk Homework
Questions:
1. Consider a baseball bat manufacturing firm. The firm has total fixed costs of $10,000 per
month. The firm sells the bats it manufactures for $50 each. Variable costs to manufacture each
bat are $25 each. What is the breakeven quantity?
2. You are assigned the job of analyzing the breakeven quantity for General Motors
Corporation. General Motors has fixed costs of $40,000,000,000 per year. The average sales
price for its cars are $20,000. The average variable costs per car are $15,000. How many cars
must General Motors sell to break even?
3: You have calculated the breakeven quantity of sales for RipYouOff Used Cars Dealership to
be 110. The company has total fixed costs of $800,000. The company sells its cars for $25,000
each. Calculate the variable cost per unit for RipYouOff?
4. The “Damn Big Pizza Company”, has estimated that it will sell 200,000 pizza’s this
year at an average price of $11.95 each. The company has fixed costs of $500,000. In addition,
variable costs of producing each Pizza are $5.95.
a. Compute the operating break even point and the degree of operating leverage for
TDBPC.
b. What will happen to the firm’s Degree of Operating Leverage if a machine is purchased
that increases fixed costs by $150,000 per year, but reduce our cost of producing each
pizza by $0.95?
5. The “Delivery in a Bikini Pizza Company” (there once was a company that provided this
service) has estimated the following income statements before and after it makes certain changes
to the company. The company sells 1,000,000 meals per year currently and expects to sell
1,500,000 meals per year after the change.
Before After
Change Change
Sales $10,000,000 $15,000,000
Variable Costs $9,000,000 $13,000,000
Fixed Costs $750,000 $1,250,000
EBIT $250,000 $750,000
I $150,000 $500,000
EBT $100,000 $250,000
Tax $20,000 $62,500
NI $80,000 $187,500
Compute the Degree of Operating Leverage, the Degree of Financial Leverage and the Degree of
Combined Leverage before and after the change.
231
Business and Financial Risk Homework Solutions
$10,000
1. 𝑄𝑄𝑄𝑄𝑄𝑄 = = 400 𝑏𝑏𝑏𝑏𝑏𝑏𝑏𝑏
$50−$25
𝐹𝐹 40,000,000,000
2. 𝑄𝑄𝑄𝑄𝑄𝑄 = = = 8,000,000 𝑐𝑐𝑐𝑐𝑐𝑐𝑐𝑐 𝑝𝑝𝑝𝑝𝑝𝑝 𝑦𝑦𝑦𝑦𝑦𝑦𝑦𝑦
𝑃𝑃−𝑉𝑉 20,000−15,000
𝐹𝐹
3. 𝑄𝑄𝑄𝑄𝑄𝑄 =
𝑃𝑃−𝑉𝑉
$800,000
110 =
$25,000 − 𝑉𝑉𝑉𝑉𝑉𝑉𝑉𝑉𝑉𝑉𝑉𝑉𝑉𝑉𝑉𝑉 𝐶𝐶𝐶𝐶𝐶𝐶𝐶𝐶𝐶𝐶 𝑈𝑈𝑈𝑈𝑈𝑈𝑈𝑈
500,000
𝑄𝑄𝑄𝑄𝑄𝑄 = = 83,333
11.95 − 5.95
b.
𝑄𝑄(𝑃𝑃 − 𝑉𝑉) 200,000(11.95 − 5.00)
𝐷𝐷𝐷𝐷𝐷𝐷 = = = 1.878
𝑄𝑄(𝑃𝑃 − 𝑉𝑉) − 𝐹𝐹 200,000(11.95 − 5.00) − 650,000
5. Before Change
𝑄𝑄(𝑃𝑃 − 𝑉𝑉) 1,000,000(10.00 − 9.00)
𝐷𝐷𝐷𝐷𝐷𝐷 = = =4
𝑄𝑄(𝑃𝑃 − 𝑉𝑉) − 𝐹𝐹 1,000,000(10.00 − 9.00) − 750,000
𝐸𝐸𝐸𝐸𝐸𝐸𝐸𝐸 250,000
𝐷𝐷𝐷𝐷𝐷𝐷 = = = 2.5
𝐸𝐸𝐸𝐸𝐸𝐸𝐸𝐸 − 𝐼𝐼 250,000 − 150,000
DCL = 4 X 2.5 = 10
232
After Change
𝑄𝑄(𝑃𝑃 − 𝑉𝑉) 1,500,000(10.00 − 8.67)
𝐷𝐷𝐷𝐷𝐷𝐷 = = = 2.68
𝑄𝑄(𝑃𝑃 − 𝑉𝑉) − 𝐹𝐹 1,500,000(10.00 − 8.67) − 1,250,000
𝐸𝐸𝐸𝐸𝐸𝐸𝐸𝐸 750,000
𝐷𝐷𝐷𝐷𝐷𝐷 = = = 3.0
𝐸𝐸𝐸𝐸𝐸𝐸𝐸𝐸 − 𝐼𝐼 750,000 − 500,000
233
Chapter 8: Time Value of Money
234
Chapter 8 Readings
235
THIS MANUSCRIPT APPEARS IN THE JOURNAL OF THE AMERICAN ACADEMY OF BUSINESS
CAMBRIDGE, Vol. 2(1), September 2002, p. 72-79
A New Method for Teaching the Time Value of Money
Terrance Jalbert, University of Hawaii at Hilo
ABSTRACT
Students frequently experience difficulty in identifying the appropriate time value of money (TVM) technique to
apply to a problem. This paper surveys the TVM presentation in seven popular introductory finance textbooks. A
new presentation technique is then developed. The presentation technique is based on a simple method for
identifying the appropriate TVM technique to apply to any problem. TVM techniques conducive to applying the
calculations in a generalized setting are then presented. Visual aids are provided to assist students in selecting
correct techniques. By using these techniques students are able to more easily identify appropriate TVM techniques.
INTRODUCTION
Many techniques have been developed for presenting the time value of money (TVM). Despite this considerable
effort on the part of instructors, students frequently experience difficulty identifying the appropriate technique to
apply to a specific problem (Eddy and Swanson, 1996). However, it is well known that a pedagogy, which works
well with one audience, does not necessarily work well with another (Bloom, 1956). Thus, the development of new
and different techniques that appeal to various audiences is beneficial. This paper develops a new technique for
teaching the TVM. The technique is specifically intended to appeal to students that benefit from precise definitions
and visual aids. The technique affords instructors a new tool in their arsenal to teach students TVM concepts. The
paper begins by surveying how seven popular introductory finance textbooks address the TVM issue. Next, a new
approach for teaching the TVM is presented. The approach begins with a simple method for distinguishing between
a single sum of money, annuity, perpetuity, growing perpetuity and uneven cash flow stream. Cash flows are
distinguished by examining conditions that must be met in order for a series of cash flows to qualify for each
classification. TVM techniques conducive to applying the calculations in a generalized setting are then presented.
Finally visual aids are provided to walk students through selecting the appropriate TVM technique for a problem.
Students nearly unanimously experience difficulty in identifying the appropriate technique to apply to TVM
problems. While the TVM issue is complex, some of the difficulty can be attributed to the approach that finance
texts take to the issue. This contention is confirmed by Eddy and Swanson who argue that instructors do not
sufficiently develop a frame of reference which begins with simple learning objectives focused on individual topics
and progresses to higher levels of understanding (Eddy and Swanson, 1996). This section contains a survey of the
approaches used in seven popular finance texts to present TVM concepts. The seven books examined in this study
are: Stanley Block and Geoffrey Hirt (BH) eighth edition of Foundations of Financial Management, 2) Charles
Moyer, James McGuigan and William Kretlow (MMK) eighth edition of Contemporary Financial Management, 3)
Zvi Bodie and Robert Merton (BM), first edition of Finance 4) Gary Emery (EM), first edition of Corporate Finance
Principles and Practice 5) Arthur Keown, David Scott, John Martin, and William Petty (KSMP), seventh edition of
Basic Financial Management, 6) William R. Lasher (LA), second edition of Practical Financial Management, and 7)
Eugene Brigham, Louis Gapenski and Michael Ehrhardt (BGE), ninth edition of Financial Management Theory and
Practice. These books are believed to be a representative cross section of texts used in introductory finance courses.
The survey revealed three common areas of concern regarding TVM presentations. The first concern is the balance
authors must make between overly detailed explanations and explanations that are overly simplistic. Regardless of
the level of detail provided, basic elements of the concept must be incorporated to ensure a complete understanding.
Failure to include the basic elements leads to a presentation that is imprecise and confusing. In each of the finance
texts surveyed, a definition for an annuity is provided. EM (p. 100) defines an annuity to be a multipayment
problem with equal periodic cash flows. BH (p. 235) define an annuity to be a series of consecutive payments or
receipts of equal amount. MMK (p. 137) define an annuity to be the receipt or payment of equal cash flows per
period for a specified amount of time. KSMP (p. 178) define an annuity to be a series of equal dollar payments for a
236
specified number of years. LA (p.142) defines an annuity to be a stream of equal payments, made or received,
separated by equal intervals of time. BM (p. 100) define an annuity to be a level stream of cash flows. BGE (p.
250) define an annuity to be a series of equal payments made at fixed intervals for a specified number of periods.
These definitions vary widely in their precision. Definition variations are also found with regard to the discussion of
uneven cash flow streams. EM, BH, KSMP and BM, do not address the issue of uneven cash flow streams in their
TVM chapters. MMK (p. 147), LA (p. 168), and BGE (258) introduce uneven cash flow streams as a means of
resolving the TVM problem that occurs because of unequal payment amounts. These definitions are generally
imprecise and leave open questions that the students must infer from example problems. More precise definitions
should help students more easily identify situations where the technique is appropriate.
The second concern that the survey revealed is incomplete explanations of how TVM techniques can be utilized.
Specifically, the surveyed texts do not sufficiently address the issue of the present and future values of annuities due
and ordinary annuities. Each of the books surveyed addresses the issue by noting that an ordinary annuity involved
receiving the payments at the end of each year and an annuity due involves receiving the payments at the beginning
of each year. While the statements made are technically correct, they lack generality. Students are frequently
unable to generalize the techniques to advanced applications such as deferred annuities.
The third concern is the introduction of TVM techniques in multiple chapters throughout the book. Spreading the
presentation of TVM techniques across multiple chapters increases the difficulty of integrating the material into a
broad understanding. This problem is common in the presentation of growing perpetuities. The technique is
frequently included in the stock valuation chapter rather than the TVM chapter. BM, BH, MMK, KSMP, LA and
BGE do not discuss growing perpetuities as part of the TVM chapter. EM (p. 105) is the only author to discuss
growing perpetuities as part of his TVM chapter.
In the remainder of this paper, a new approach for presenting TVM techniques is provided. The methodology
developed in this paper addresses each of the three issues identified in the survey. The method incorporates
definitions that allow students to more easily identify appropriate TVM techniques to apply to a problem. It
compactly discusses TVM techniques in one location and clearly distinguishes between each of the techniques. It is
developed so that students can generalize the techniques to advanced problems. The method begins by identifying
characteristics of cash flows that must be examined to determine which TVM technique is appropriate for the
problem. Visual aids are presented in Figure 1 and Table 1. These visual aids are often helpful for students to
internally package the materials in their minds. Readers should consult these visual aids throughout the following
discussion.
To determine the appropriate TVM technique to apply to a problem, four characteristics of the cash flows must be
analyzed. By examining these four characteristics, the appropriate TVM technique for any basic problem can be
identified. Moreover, the examination reveals situations where advanced techniques are required. The four
characteristics to be examined are: 1) Is there a series of cash flows? 2) Is the number of cash flows limited? 3) Is
there equal time spacing between each of the cash flows? and 4) Is the dollar amount of each cash flow equal? The
first characteristic to examine is if there is a single cash flow or a series of cash flows. A single cash flow implies
that there is only one cash flow, whereas a series of cash flows implies that there are more than one cash flow. An
investment that promises to pay $100 per year at the end of each of the following three years is a series of cash
flows. The investment provides three cash flows of $100 each. Alternatively, an investment that promises to pay a
single amount of $1,000, five years from today is not a series of cash flows. If a problem involves a single sum, it is
not necessary to examine the remaining characteristics. The correct technique is to compute the present or future
value of a single sum. Techniques for computing the present and future values of single sums are well known and
are not discussed here. If a problem involves a series of cash flows, the remaining three characteristics must be
examined to determine the appropriate TVM technique to apply to the problem. The second characteristic that must
be examined is if there are a limited number of cash flows or if the cash flows continue into infinity. An investment
that promises to pay a $100 cash flow on January 1st of each of the next three years has a limited number of cash
flows. An investment that pays a $100 cash flow on January 1st each year into infinity has an infinite number of
cash flows. The third characteristic to be examined is if there is an equal amount of time between each of the cash
flows. This means that the cash flows must occur at some constant time interval such as one cash flow per month,
or one cash flow per year. An investment that promises to pay us $100 on January 1st of each of the next three years
237
has equal time spacing. There is a one-year interval between each of the cash flows. On the other hand, an
investment that promises to pay $100 on January 1st of the next year, nothing on January 1st of the second year, $100
on January 1st of the third year and $100 on January 1st of the fourth year does not have equal time spacing. In this
case, the interval between the first two cash flows is two years while the interval between the third and fourth is one
year. Finally, The fourth characteristic to be examined is if each cash flow is for an equal dollar amount or for
varying dollar amounts. An investment that promises to pay $100 per year at the end of each of the next three years
has equal dollar amounts as each cash flow is for $100. An investment that promises to pay $100 the first year and
$200 the second year does not have equal dollar amounts. While there are a series of cash flows, the cash flows are
not for the same dollar amount. By examining these four characteristics, the appropriate technique to apply to any
TVM problem can be identified.
ANNUITIES
If the answer to each of the four questions in the previous section is yes, a series of cash flows qualifies as an
annuity. An annuity thus has the following characteristics: 1) there is series of cash flows, 2) the number of cash
flows limited, 3) there is equal time spacing between each of the cash flows and 4) the dollar amount of each cash
flow equal. The formal definition of an annuity that follows is a series of cash flows where the number of cash
flows is limited, there is an equal time spacing between each of the cash flows, and the dollar amount of each cash
flow is for an equal amount. Defining an annuity in this way explicitly invites the student to examine each of the
four characteristics. BGE is the only book surveyed that addresses each of the four characteristics in their definition.
Once it is established that a series of cash flows qualifies as an annuity, the present value or the future value of the
cash flows can be computed using techniques designed specifically for an annuity. Two options are available for
calculating the present value of an annuity using most financial calculators and TVM tables. The first option is to
compute the value of the cash flows at the time of the first cash flow by computing the present value of an annuity
due. The second option is to compute the value of the cash flows one time period before the first cash flow by
computing the present value of an ordinary annuity.
Suppose a loan obligates the borrower to deposit three $100 cash flows into an account. The first cash flow will be
made three years from today. The second cash flow will be made four years from today and the third cash flow will
be made five years from today. The appropriate discount rate for these cash flows is assumed to be ten percent. The
following time line depicts this annuity. The time line also shows the point in time at which we calculate the value
of the annuity if we apply the present value of an ordinary annuity technique (PVOA) and if we apply the present
value of an annuity due technique (PVAD) to the cash flows:
0 1 2 3 4 5 6 7
|----------|----------|----------|----------|----------|----------|----------| Fig. 2.1
100 100 100
PV PV
OA AD
The computations for the present and future value of an annuity can be made in several ways including by formula,
TVM tables and financial calculators. By applying the present value of an annuity due technique to this stream of
cash flows, the value of the cash flows at time three is computed to be $273.55. Applying the present value of an
ordinary annuity calculation to this stream of cash flows, the value of the cash flows at time two on the time line is
computed to be $248.69.
Two alternatives are also available for calculating the future value of an annuity. The first alternative is to compute
the value of the cash flows at the time of the last cash flow by computing the future value of an ordinary annuity.
The second alternative is to compute the value of the cash flows one time period after the last cash flow by
computing the future value of an annuity due. The annuity consisting of three $100 cash flows is again shown on
the following time line. The time line also shows the point in time at which the value of the annuity is calculated if
we apply the future value of an ordinary annuity technique (FVOA) and if we apply the present value of an annuity
due technique (FVAD) to the annuity.
238
0 1 2 3 4 5 6 7
|----------|----------|----------|----------|----------|----------|----------| Fig. 2.2
100 100 100
FV FV
OA AD
The appropriate interest rate is again ten percent. Applying the future value of an annuity due calculations to this
stream of cash flows, the value of the cash flows at time six is computed to be $364.10. Applying the future value
of an ordinary annuity calculation to this stream of cash flows, the value of the cash flows at time five on the time
line is computed to be $331.00.
By combining Figures 2.1 and 2.2, a generalized illustration for selecting the appropriate annuity calculation
technique is created. The figure is augmented by showing the mode financial calculators must be set in to make the
calculations. The following figure facilitates easy identification of the appropriate computational technique for any
annuity problem. Students are required to commit this illustration to memory.
A B C D E F G H I
|----------|----------|----------|----------|----------|----------|----------|----------| Fig. 2.3
First Middle Last
Cash Flow Cash Flows Cash Flow
PV PV FV FV
OA AD OA AD
Mode END BEG END BEG
Figure 2.3 shows that the value of any annuity can be computed at four different points on the time line: 1) one time
period before the first cash flow (PVOA), 2) at the time of the first cash flow (PVAD), 3) at the time of the last cash
flow (FVOA), and 4) one time period following the last cash flow (FVAD). Letters are intentionally used in Figure
2.3 to help students generalize the illustration to any time period.
To illustrate the use of Figure 2.3, consider two annuities. Both annuities will provide cash flows of $500 per year
for the next five years. The first annuity (Fig. 2.4) will provide the first cash flow today. The second annuity (Fig.
2.5) will provide the first cash flow one year from today. The objective is to compute the value of either annuity
today, at time 0 on the time line.
0 1 2 3 4 5
|----------|----------|----------|----------|----------| Fig. 2.4
500 500 500 500 500
0 1 2 3 4 5 6
|----------|----------|----------|----------|----------|-----------| Fig. 2.5
500 500 500 500 500
For the first annuity (Fig. 2.4), the first cash flow occurs at time zero. Time zero is also the point in time at which
the value of the cash flows will be computed. As such, the present value will be computed at the time of the first
cash flow. Referring to Figure 2.3, it is seen that in order to compute the value of the cash flows at the time of the
first cash flow, the present value of an annuity due should be computed. For the second annuity (Fig. 2.4), the first
cash flow occurs at time one. Time zero is the point in time at which the value of the cash flows will be computed.
As such, the present value will be computed one time period before the first cash flow. Referring to Figure 2.3, it is
seen that in order to compute the value of the cash flows one time period before the first cash flow, the present value
of an ordinary annuity should be computed. The value of the second annuity could also be computed at time one on
the time line by applying the present value of an annuity due computations to the annuity.
A similar analysis can be made for computing future values. The value of the first annuity (Fig. 2.4) can be
computed at either time four or time five by using the annuity techniques. Referring to Figure 2.3, it is seen that in
239
order to calculate the value of the annuity at time four, the future value of an ordinary annuity is computed. In order
to compute the value of the annuity at time five, the future value of an annuity due is computed. The value of the
second annuity (Fig. 2.5) can be computed at either time five or time six using annuity techniques. Again referring
to figure 2.3, it is seen that in order to calculate the value of the annuity at time five, the future value of an ordinary
annuity is computed. In order to compute the value of the annuity at time six, the future value of an annuity due is
computed.
PERPETUITIES
The conditions for a series of cash flows to qualify as an annuity are not always met. Fortunately, other TVM
techniques are available when the conditions are not met. The correct technique depends upon which of the
conditions are not met. When conditions 1,3, and 4 of an annuity are met, but the second condition of an annuity is
not, the series of cash flows is referred to as a perpetuity. A perpetuity is a series of cash flows that continue
forever, where there is an equal time spacing between each of the cash flows and the dollar amount of each cash
flow is for an equal amount. As such, three of the annuity conditions are met. However, the second condition of an
annuity is violated because the cash flows go on forever. The investment will give the investor some equal cash
flow each year forever. An example of a perpetuity is an investment that will give the investor $100 each year
forever.
Once it is established that a series of cash flows qualifies as a perpetuity, the present value of the cash flows can be
computed using techniques designed specifically for a perpetuity. Two alternatives are available for calculating the
present value of a perpetuity. The correct technique depends upon when the next cash flow will be received. The
present value one time period before the receipt of a cash flow is due can be calculated as the present value of an
ordinary perpetuity. The present value immediately prior to the receipt of a cash flow can be calculated as the
present value of a perpetuity due.
Suppose that an investor wishes to calculate the present value a perpetuity that will pay her $100 per year each year
forever. The first cash flow will be received at the end of the third year. The appropriate discount rate is assumed to
be ten percent. The following time line illustrates this perpetuity. The time line also shows the point in time at
which the value of the perpetuity is calculated if we apply the present value of an ordinary perpetuity technique
(PVOP) and if we apply the present value of a perpetuity due technique (PVPD) to the cash flows.
0 1 2 3 4 5 6 7
|----------|----------|----------|----------|----------|----------|----------|………… Fig. 3.1
100 100 100 100 100 ………
PV PV
OP PD
CF
The formula for computing the present value of a perpetuity due is: PVPD = + CF . The formula for
i
CF
computing the present value of an ordinary perpetuity is PVOP = . Where CF denotes the amount of the cash
i
flows and i denotes the appropriate discount rate for the cash flows. Computing the present value of an ordinary
perpetuity for these cash flows, the value of the cash flows at time two is computed to be $1,000. Computing the
present value of a perpetuity due for these cash flows, the value of the cash flows at time three is computed to be
$1,100.
GROWING PERPETUITIES
Another combination of the conditions of an annuity that may not be met occurs when the first and third conditions
are met but the second and fourth conditions are not met. In this case, there are a series of cash flows which have an
equal amount of time between each of the cash flows. Thus the first and third conditions of an annuity are met.
However, the second assumption of an annuity is not met because the cash flows continue forever. In addition, the
fourth condition of an annuity is not met because the cash flows are not equal. While the cash flows are not equal, a
different condition does apply with regard to the magnitude of the cash flows. That condition is that the cash flows
240
must become larger by some constant percentage amount in each successive time period. When this combination of
conditions are present, the series of cash flows is referred to as a growing perpetuity. Specifically, a growing
perpetuity is a series of cash flows that continue forever, where there is a equal time spacing between each of the
cash flows, and the dollar amount of each cash flow increases by some constant percentage amount in each
successive time period. In a manner similar to a perpetuity, the value of the cash flows can be computed at two
different points on the time line. The present value of a growing perpetuity due technique (PVGPD) computes the
value of the cash flows immediately prior to receiving a cash flow. The present value of an ordinary growing
perpetuity technique (PVGOP) computes the value of the cash flows one time period prior to receiving a cash flow.
An example of a growing perpetuity is an investment that promises to pay $100 at time 3. The investment will
continue to provide annual cash flows forever, however, the cash flows will become five percent larger in each
successive year. The appropriate discount rate for this growing perpetuity is assumed to be ten percent. The
following time line illustrates this growing perpetuity. The time line also shows the points on the time line that the
value of the growing perpetuity is calculated if we apply the present value of an ordinary growing perpetuity and
present value of a growing perpetuity due calculations to the cash flows.
0 1 2 3 4 5 6 7
|----------|----------|----------|----------|----------|----------|----------|………. Fig. 4.1
100 105 110.25 115.76 121.55…..
PV PV
GOP GPD
The formula for calculating the present value of a growing ordinary perpetuity is PVGOPN = CFN + 1 . The formula
(i − g )
for calculating the present value of a growing perpetuity due is PVGPDN = CF N +1
+ CFN . Where N denotes the
(i − g )
time period and g denotes the percentage growth rate. If the present value of a growing perpetuity due technique is
applied to these cash flows, the value of the cash flows at time three on the time line is computed to be $2,200. If the
present value of a growing ordinary perpetuity technique is applied to these cash flows, the value of the cash flows
at time two on the time line is computed to be $2,000.
Another combination of the conditions for an annuity that may not be met occurs when the first and second
conditions of an annuity are met, but the third and fourth conditions of an annuity are not met. In this case, we have
a series of cash flows and the number of cash flows is limited. Thus, the first and second conditions of an annuity
are met. However, the time spacing between the cash flows may or may not be equal and the dollar amount of the
cash flows may or may not be equal in each year. As such, the third and fourth conditions of an annuity are not met.
When this combination of conditions occurs, the series of cash flows is called an uneven cash flow stream. Thus an
uneven cash flow stream is formally defined as a series of cash flows, where the number of cash flows is limited,
there may or may not be an equal time spacings between the cash flows and the cash flows may or may not be for
equal amounts. An example of an uneven cash flow stream is an investment that will give us $100 three years from
today, nothing at the end of the fourth year, $100 at the end of the fifth year and $200 at the end of the sixth year.
These cash flows are illustrated on the following time line:
0 1 2 3 4 5 6 7
|----------|----------|----------|----------|----------|----------|----------| Fig. 5.1
100 0 100 200
The present value and the future value of an uneven cash flow stream can be calculated. To calculate present value
or the future value of an uneven cash flow stream, each cash flow must be treated as a single sum and the present
value or future value of each single sum must be calculated individually. The time at which the value of the cash
flows is computed can be customized by adjusting the number of time periods used in the calculations. Many
calculators are equipped with a feature that allows the user to input all of the cash flows into the calculator at one
time. The calculator then simultaneously computes the present value of each of the individual cash flows and totals
241
them. Using these techniques, the present value of the cash flows at time three is computed to be $332.91. The
future value of this uneven cash flow stream at time seven on the time line is computed to be $487.41.
VISUAL AIDS
Table 1 and Figure 1 provide two visial aids for seleting the appropriate TVM technique for a problem. These visual
aids are often helpful for students to internally package the materials in their minds. Figure 1 is a flowchart that
walks the user through each of the cash flow characteristics. By answering yes or no to several questions, the user
can determine the appropriate TVM technique for any problem. In addition to pointing the student to the correct
technique, the table identifies two specific situations in which a combination of techniques must be utilized to solve
the problem. The precise combination of techniques that must be used is case specific. Table 1 shows the
conditions necessary for a series of cash flows to be classified as a single sum, an annuity, a perpetuity, a growing
perpetuity or an uneven cash flow stream. The rows indicate each of the four cash flow characteristics. The
columns indicate the four possible classifications of a series of cash flows. The cells indicate if the condition must
be present or absent in order for the cash flows to qualify for the classification.
CONCLUDING COMMENTS
Students frequently experience difficulty identifying the appropriate time value of money (TVM) technique to apply
to a problem. In this paper it is suggested that this difficulty might be due in part to imprecise definitions. Seven
textbooks are surveyed to determine common teaching techniques. Next, a new method for presenting TVM
techinques is presented. The method is based on precisely distingushing between single sums, annuities,
perpetuities, growing perpetuities and uneven cash flow streams. The distinguishment is based on the examination
of four characteristics of a series of cash flows. Visual aids are developed to provide students an easy means of
navigating the maze of TVM techniques. The technique developed here afford instructors a new tool in their arsenal
to teach students TVM skills. The technique is specifically intended to appeal to students that benefit from precise
definitions and visual aids.
242
FIGURE 1: FLOWCHART FOR SELECTING APPRPRIATE TIME VALUE OF MONEY TECHNIQUES
Stream of Cash Yes Limited Number Yes Equal Time Yes Equal Dollar Yes
Annuity
Flows? of Cash Flows? Spacing? Amount?
No No No No
No No
No
Combination Combination
Problem Problem
Block, Stanley and Geoffrey Hirt (1997). Foundations of Financial Management. 8th ed. Boston: Irwin.
Bloom, B. (1956). Taxonomy of Educational Objectives, Handbook I: Cognitive Domain, New York: McKay.
Bodie, Zvi and Robert Merton (2000). Finance. Upper Saddle River, NJ: Prentice Hall.
Brigham, Eugene, Louis Gapenski, Michael Ehrhardt (1999). Financial Management Theory and Practice, 9th ed.
Fort Worth: Dryden.
Eddy, Albert and Gene Swanson (1996), “A Hierarchy of Skills Approach to Teaching Accounting Present Value,”
Journal of Accounting Education 14(1) p. 123-131.
Emery, Gary (1998). Corporate Finance, Reading, Massachusetts: Addison-Wesley.
Keown, Arthur, et al. (1996). Basic Financial Management, 7th ed. Upper Saddle River, NJ: Prentice Hall 1996.
Lasher, William (2000). Practical Financial Management. 2nd ed. Cincinnati: South-Western.
Moyer, Charles, James McGuigan, William Kretlow (2001). Contemporary Financial Management. 8th ed.
Cincinnati: South-Western.
243
Chapter 8 Lecture Notes
244
Chapter 8: Time Value of Money
Usefulness
Time Value of Money Techniques are useful both for business
decisions and in your personal life. Time Value of Money
Techniques allow you to answer questions like:
245
Time Lines
A graphical technique to display the passage of time
0 1 2 3 4 5 6 7
|----------|----------|----------|----------|----------|----------|----------|
Key Language
Today point 0
One year from today point 1
Two years from today point 2
The beginning of the first year point 0
The end of the first year point 1
The beginning of the second year point 1
0 1 2 3 4 5 6 7
|----------|----------|----------|----------|----------|----------|----------|
* * *
The end of each of the next three years
0 1 2 3 4 5 6 7
|----------|----------|----------|----------|----------|----------|----------|
* * *
246
Nominal versus Periodic Interest
Periodic Interest Rate – The interest rate for some fraction of a full
year.
F = Fraction of a Year
1
Periodic Interest = 12 X = 1%
12
247
Simple Interest versus Compound Interest
Simple Interest: Interest on the principal amount that you deposit
in the bank only.
0 1 2
|--------------------------|------------------------------|
-100 10 10
Interest on $100 Interest on $100
$100 X .10 = $10 $100 X .10 = $10
248
Compound Interest Illustration
0 1 2
|--------------------------|------------------------------|
-100 10 11
Interest on $100 Interest on $100
$100 X .10 = $10 $100 X .10 = $10
249
Technique Selection
Depending upon the type of problem under consideration, different
Time Value of Money (TVM) Techniques must be used to solve
the problem. The first key to solving a TVM problem is to select
the correct technique. To do that, you must answer four questions
regarding the problem:
250
Flowchart for Selecting the Appropriate Time
Value of Money Technique
Stream of Cash Yes Limited Number Yes Equal Time Yes Equal Dollar Yes
Annuity
Flows? of Cash Flows? Spacing? Amount?
No No No No
No No
No
Combination Combination
Problem Problem
251
Techniques
a. Single Sum
b. Annuity
c. Perpetuity
252
d. Growing Perpetuity
i.e. I will deposit $100 in the bank this year $200 one year
from today, and $400 two years from today.
253
f. Combination Problems (Multiple types of combination
problems
254
Methods for Solving Time Value of Money
Problems
Advantages
a. you only need a simple calculator
b. Information in Books is generally presented using the
formula method.
Disadvantages
a. you must memorize the formulas
b. some problems are trial and error type of problems, so
it takes a great deal of time to solve the problem
Advantages
a. you do not need to memorize the formulas
b. you only need a simple calculator
Disadvantages
a. You must have access to the tables
b. Some problems are trial and error type of problems, so
it takes a great deal of time to solve the problem.
255
3. Calculator
Advantages
a. you do not need to memorize the formulas
b. you can solve any problem quickly
c. may be used when taking the CFA examination
Disadvantage
a. you must spend $30 on a calculator
b. your can not store information for multiple problems
in memory.
4. Spreadsheet
Advantages
a. you do not need to memorize the formulas
b. You can solve any problem quickly
c. You can store problems in memory for later use
Disadvantages
a. You must purchase a computer and spreadsheet
Program
b. It is not a particularly intuitive way to learn TVM
256
To Set your Calculator for Use in This Class
257
To Clear Your Calculator
a. To clear what is showing on your screen
Clr
i. CF
ii. 2nd CLR WRK
258
Single Sum
A single amount of money:
Stream of Cash Yes Limited Number Yes Equal Time Yes Equal Dollar Yes
Annuity
Flows? of Cash Flows? Spacing? Amount?
No No No No
No No
No
Combination Combination
Problem Problem
259
The future value or the present value of a single sum can be
computed:
0 1 2 3 4 5
|----------|----------|----------|----------|-----------|
-100 ?
PV = $100
N=5
I = 10
CPT FV = ?
FV = $161.05
Formula Method
FV=$100(1+0.10)5
FV= $161.05
260
b. Present Value of a Single Sum
0 1 2 3 4 5
|----------|----------|----------|----------|-----------|
? 100
FV = $100
N=3
I = 10
CPT PV = ?
PV = $75.13
Formula Method
𝐹𝐹𝐹𝐹
𝑃𝑃𝑃𝑃 = * you must memorize this formula
(1+𝑖𝑖)𝑁𝑁
100
𝑃𝑃𝑃𝑃 =
(1 + 0.10)3
100 100
𝑃𝑃𝑃𝑃 = =
(1 + 0.10)3 1.331
PV = $75.13
261
c. Solving for other variables in a single sum problem
0 1 2 3 4 5
|----------|----------|----------|----------|-----------|
-100 140
FV = $140.00
PV = -100
N=3
CPT I = ?
I = 11.87%
262
ii. Solving for N
0 ?
|----------|----------|----------|----------|-----------|
-$100 $200
PV = -100
FV = $200
I = 20
CPT N = ?
N = 3.8 years
Note: When you input both the present value and a future
value into your calculator in the same problem,
you must enter one as a negative number. I
recommend that you always enter the present
value as a negative number.
263
Effective Annual Interest Rates
Sometimes interest rates are quoted in non-annually compounded
terms. We would like to convert these quotations into an annually
compounded equivalent for direct comparison.
2nd Iconv
Nom = 9.5 ↵ ↓ ↓
C/Y = 12 ↵ ↓ ↓
EFF = ?, CPT, EFF = 9.925
264
Other Compounding Periods
Suppose that you will deposit $100 into the bank in an account that
pays 10 percent nominal interest compounded semi-annually.
How much money will you have in the bank three years from
today?
0 1 2 3 4 5
|----------|----------|----------|----------|-----------|
-100 ?
There are two methods you can use to solve this problem
2nd Iconv
Nom = 10 ↵ ↓ ↓
C/Y = 2 ↵ ↓↓
EFF = ?, CPT, EFF = 10.25
PV = -100
N=3
I = 10.25
CPT FV = ?
FV = 134.01
265
Method 2: Adjust the Calculation Method
Ncalc = N X M = 3 X 2 = 6
I 10
Icalc = = =5
M 2
Step 3: Solve for the Future Value using the Interest Rate
and Number of Periods Computed in Steps 1 and 2.
PV = 100
N=6
I=5
CPT FV = ?
FV = $134.01
266
Continuous Compounding
Suppose that you will deposit $100 into the bank in an account that
pays 10 percent nominal interest compounded continuously. How
much money will you have in the bank three years from today?
𝐹𝐹𝐹𝐹 = $134.99
Suppose that you wish to have $100 in the bank three years from
today. The account pays 10 percent nominal interest compounded
continuously. How much do you need to deposit today?
𝑃𝑃𝑃𝑃 = $74.08
267
Annuities
Stream of Cash Yes Limited Number Yes Equal Time Yes Equal Dollar Yes
Annuity
Flows? of Cash Flows? Spacing? Amount?
No No No No
No No
No
Combination Combination
Problem Problem
0 1 2 3 4 5 6
|----------|----------|----------|----------|-----------|------------|
100 100 100 100
268
Once we have confirmed that a series of cash flows is an annuity,
we can compute the value of the annuity at four different time
points:
Technique Mode
2. One time period before the first cash flow PVOA END
4. One Time Period after the last cash flow FVAD BEG
269
1. Annuity Technique Identifier for PV of an Ordinary Annuity
A B C D E F G H
|----------|----------|----------|----------|----------|----------|----------|
PVOA
Mode END
A B C D E F G H
|----------|----------|----------|----------|----------|----------|----------|
PVAD
Mode BEG
270
3. Annuity Technique Identifier for FV of an Ordinary Annuity
A B C D E F G H
|----------|----------|----------|----------|----------|----------|----------|
FVOA
Mode END
A B C D E F G H
|----------|----------|----------|----------|----------|----------|----------|
FVAD
Mode BEG
271
Combined Annuity Computation Technique Identifier
A B C D E F G H
|----------|----------|----------|----------|----------|----------|----------|
272
Consider the following annuity
0 1 2 3 4 5 6 7
|----------|----------|----------|----------|----------|----------|----------|
Note that when your calculator is set in the beginning mode, you
will see BGN in the display
273
Examples of Annuities
Suppose that I will deposit $100 into the bank at the end of each of
the next 5 years. The account will pay 7 percent nominal interest
compounded annually. How much money will I have in the bank 5
years from today?
0 1 2 3 4 5 6 7
|----------|----------|----------|----------|----------|----------|----------|
PMT = 100
N=5
I=7
CPT FV = ?
FV = $575.07
274
Suppose that I want to have $10,000 available at the end of each of
the next 5 years for graduate school. I want to deposit just enough
money into an account today for that purpose. The account will
pay 6 percent nominal interest compounded annually. How much
money do I need to deposit today?
0 1 2 3 4 5 6 7
|----------|----------|----------|----------|----------|----------|----------|
I want to know the value one time period before the first cash flow.
Thus I use the present value of an ordinary annuity (PVOA, END)
technique.
PMT = 10,000
N=5
I=6
CPT PV = ?
PV = $42,123.64
275
Suppose that I will deposit $1,000 into the bank at the end of each
of the next 4 years. The account will pay 6 percent nominal
interest compounded annually. How much money will I have in
the bank 5 years from today?
0 1 2 3 4 5 6 7
|----------|----------|----------|----------|----------|----------|----------|
PMT = 1,000
N=4
I=6
CPT FV = ?
FV = $4637.09
276
Suppose that I want to have $10,000 available at the beginning of
each of the next 5 years for graduate school. I want to deposit just
enough money into an account today for that purpose. The account
will pay 6 percent nominal interest compounded annually. How
much money do I need to deposit today?
0 1 2 3 4 5 6 7
|----------|----------|----------|----------|----------|----------|----------|
PMT = 10,000
N=5
I=6
CPT PV = ?
PV = $44,651.06
277
Solving for the Payment in an Annuity
0 1 2 3 4 5 6 7
|----------|----------|----------|----------|----------|----------|----------|
? ? ? ?
100,000
Beginning Mode
PV = 100,000
N=4
I=8
PMT = ? = $27,955.63
278
Suppose I want to have $50,000 in the bank 5 years from today. I
will make equal deposits into the bank at the end of each of the
next 5 years to fund the account. I will earn 8 percent nominal
interest compounded annually on any money in the account. How
much do I need to deposit each year?
0 1 2 3 4 5 6 7
|----------|----------|----------|----------|----------|----------|----------|
? ? ? ? ?
50,000
End Mode
FV = 50,000
N=5
I=8
PMT = ? = $8,522.82
279
Solving for the Interest Rate in an Annuity
0 1 2 3 4 5 ….. 20
|----------|----------|----------|----------|----------|----------|----------|
End Mode
PV = -200,000
N = 20
PMT = $25,000
I = ? = 10.93%
280
Example of Annuity
You have just won the Texas Lottery. You will receive 20 equal
annual payments of $500,000 each. You will receive the first
payment today. You have decided that you want to get as much
money as you can get today. So you go to the bank to exchange
these payments for a lump sum of money today. The bank tells
you they will pay you based on an 8% nominal compounded
annually discount rate.
0 1 2 3 4…..….19
|----------|----------|----------|----------|----------|
500k 500k 500k 500k 500k 500k
?
Solution
BEG MODE
PMT = 500,000
N = 20
I=8
CPT PV = ?
PV = $5,301,799.60
281
Loan Amortization
Amortizing a loan is the process of designing a loan so that each of
your payments will be for the same amount.
This is done by varying the amount of interest that you pay and the
amount of principal that you pay on the loan each month.
While your payments will be for the same amount each month:
Ncalc = N X M = 30 X 12 = 360
I 6
Icalc = = = 0.5 %
M 12
282
PV = 100000
I = 0.5
N = 360
PMT = ? = 599.55
283
The Principal (5) is computed as the payment minus the interest
284
Amortization Schedule
Month Beg Bal Payment Interest Prin End Bal
1 100000.00 599.55 500.00 99.55 99900.45
2 99900.45 599.55 499.50 100.05 99800.40
3 99800.40 599.55 499.00 100.55 99699.85
4 99699.85 599.55 498.50 101.05 99598.80
5 99598.80 599.55 497.99 101.56 99497.25
6 99497.25 599.55 497.49 102.06 99395.18
7 99395.18 599.55 496.98 102.57 99292.61
8 99292.61 599.55 496.46 103.09 99189.52
9 99189.52 599.55 495.95 103.60 99085.92
10 99085.92 599.55 495.43 104.12 98981.80
11 98981.80 599.55 494.91 104.64 98877.16
12 98877.16 599.55 494.39 105.16 98771.99
13 98771.99 599.55 493.86 105.69 98666.30
14 98666.30 599.55 493.33 106.22 98560.09
15 98560.09 599.55 492.80 106.75 98453.34
16 98453.34 599.55 492.27 107.28 98346.05
17 98346.05 599.55 491.73 107.82 98238.23
18 98238.23 599.55 491.19 108.36 98129.87
19 98129.87 599.55 490.65 108.90 98020.97
20 98020.97 599.55 490.10 109.45 97911.53
21 97911.53 599.55 489.56 109.99 97801.54
22 97801.54 599.55 489.01 110.54 97690.99
23 97690.99 599.55 488.45 111.10 97579.90
24 97579.90 599.55 487.90 111.65 97468.25
25 97468.25 599.55 487.34 112.21 97356.04
26 97356.04 599.55 486.78 112.77 97243.27
27 97243.27 599.55 486.22 113.33 97129.94
28 97129.94 599.55 485.65 113.90 97016.04
29 97016.04 599.55 485.08 114.47 96901.57
30 96901.57 599.55 484.51 115.04 96786.52
31 96786.52 599.55 483.93 115.62 96670.91
32 96670.91 599.55 483.35 116.20 96554.71
33 96554.71 599.55 482.77 116.78 96437.94
34 96437.94 599.55 482.19 117.36 96320.57
35 96320.57 599.55 481.60 117.95 96202.63
36 96202.63 599.55 481.01 118.54 96084.09
37 96084.09 599.55 480.42 119.13 95964.96
38 95964.96 599.55 479.82 119.73 95845.24
39 95845.24 599.55 479.23 120.32 95724.91
40 95724.91 599.55 478.62 120.93 95603.99
41 95603.99 599.55 478.02 121.53 95482.46
42 95482.46 599.55 477.41 122.14 95360.32
43 95360.32 599.55 476.80 122.75 95237.57
44 95237.57 599.55 476.19 123.36 95114.21
285
45 95114.21 599.55 475.57 123.98 94990.23
46 94990.23 599.55 474.95 124.60 94865.63
47 94865.63 599.55 474.33 125.22 94740.41
48 94740.41 599.55 473.70 125.85 94614.56
49 94614.56 599.55 473.07 126.48 94488.08
50 94488.08 599.55 472.44 127.11 94360.97
51 94360.97 599.55 471.80 127.75 94233.23
52 94233.23 599.55 471.17 128.38 94104.85
53 94104.85 599.55 470.52 129.03 93975.82
54 93975.82 599.55 469.88 129.67 93846.15
55 93846.15 599.55 469.23 130.32 93715.83
56 93715.83 599.55 468.58 130.97 93584.86
57 93584.86 599.55 467.92 131.63 93453.23
58 93453.23 599.55 467.27 132.28 93320.95
59 93320.95 599.55 466.60 132.95 93188.00
60 93188.00 599.55 465.94 133.61 93054.39
61 93054.39 599.55 465.27 134.28 92920.12
62 92920.12 599.55 464.60 134.95 92785.17
63 92785.17 599.55 463.93 135.62 92649.54
64 92649.54 599.55 463.25 136.30 92513.24
65 92513.24 599.55 462.57 136.98 92376.26
66 92376.26 599.55 461.88 137.67 92238.59
67 92238.59 599.55 461.19 138.36 92100.23
68 92100.23 599.55 460.50 139.05 91961.18
69 91961.18 599.55 459.81 139.74 91821.44
70 91821.44 599.55 459.11 140.44 91680.99
71 91680.99 599.55 458.40 141.15 91539.85
72 91539.85 599.55 457.70 141.85 91398.00
73 91398.00 599.55 456.99 142.56 91255.44
74 91255.44 599.55 456.28 143.27 91112.17
75 91112.17 599.55 455.56 143.99 90968.18
76 90968.18 599.55 454.84 144.71 90823.47
77 90823.47 599.55 454.12 145.43 90678.03
78 90678.03 599.55 453.39 146.16 90531.87
79 90531.87 599.55 452.66 146.89 90384.98
80 90384.98 599.55 451.92 147.63 90237.36
81 90237.36 599.55 451.19 148.36 90089.00
82 90089.00 599.55 450.44 149.11 89939.89
83 89939.89 599.55 449.70 149.85 89790.04
84 89790.04 599.55 448.95 150.60 89639.44
85 89639.44 599.55 448.20 151.35 89488.09
86 89488.09 599.55 447.44 152.11 89335.98
87 89335.98 599.55 446.68 152.87 89183.11
88 89183.11 599.55 445.92 153.63 89029.47
89 89029.47 599.55 445.15 154.40 88875.07
90 88875.07 599.55 444.38 155.17 88719.90
91 88719.90 599.55 443.60 155.95 88563.95
92 88563.95 599.55 442.82 156.73 88407.22
286
93 88407.22 599.55 442.04 157.51 88249.70
94 88249.70 599.55 441.25 158.30 88091.40
95 88091.40 599.55 440.46 159.09 87932.31
96 87932.31 599.55 439.66 159.89 87772.42
97 87772.42 599.55 438.86 160.69 87611.73
98 87611.73 599.55 438.06 161.49 87450.24
99 87450.24 599.55 437.25 162.30 87287.94
100 87287.94 599.55 436.44 163.11 87124.83
101 87124.83 599.55 435.62 163.93 86960.90
102 86960.90 599.55 434.80 164.75 86796.16
103 86796.16 599.55 433.98 165.57 86630.59
104 86630.59 599.55 433.15 166.40 86464.19
105 86464.19 599.55 432.32 167.23 86296.96
106 86296.96 599.55 431.48 168.07 86128.90
107 86128.90 599.55 430.64 168.91 85959.99
108 85959.99 599.55 429.80 169.75 85790.24
109 85790.24 599.55 428.95 170.60 85619.64
110 85619.64 599.55 428.10 171.45 85448.19
111 85448.19 599.55 427.24 172.31 85275.88
112 85275.88 599.55 426.38 173.17 85102.71
113 85102.71 599.55 425.51 174.04 84928.68
114 84928.68 599.55 424.64 174.91 84753.77
115 84753.77 599.55 423.77 175.78 84577.99
116 84577.99 599.55 422.89 176.66 84401.33
117 84401.33 599.55 422.01 177.54 84223.79
118 84223.79 599.55 421.12 178.43 84045.35
119 84045.35 599.55 420.23 179.32 83866.03
120 83866.03 599.55 419.33 180.22 83685.81
121 83685.81 599.55 418.43 181.12 83504.69
122 83504.69 599.55 417.52 182.03 83322.66
123 83322.66 599.55 416.61 182.94 83139.73
124 83139.73 599.55 415.70 183.85 82955.88
125 82955.88 599.55 414.78 184.77 82771.10
126 82771.10 599.55 413.86 185.69 82585.41
127 82585.41 599.55 412.93 186.62 82398.79
128 82398.79 599.55 411.99 187.56 82211.23
129 82211.23 599.55 411.06 188.49 82022.74
130 82022.74 599.55 410.11 189.44 81833.30
131 81833.30 599.55 409.17 190.38 81642.92
132 81642.92 599.55 408.21 191.34 81451.58
133 81451.58 599.55 407.26 192.29 81259.29
134 81259.29 599.55 406.30 193.25 81066.04
135 81066.04 599.55 405.33 194.22 80871.82
136 80871.82 599.55 404.36 195.19 80676.63
137 80676.63 599.55 403.38 196.17 80480.46
138 80480.46 599.55 402.40 197.15 80283.31
139 80283.31 599.55 401.42 198.13 80085.18
140 80085.18 599.55 400.43 199.12 79886.05
287
141 79886.05 599.55 399.43 200.12 79685.93
142 79685.93 599.55 398.43 201.12 79484.81
143 79484.81 599.55 397.42 202.13 79282.69
144 79282.69 599.55 396.41 203.14 79079.55
145 79079.55 599.55 395.40 204.15 78875.40
146 78875.40 599.55 394.38 205.17 78670.23
147 78670.23 599.55 393.35 206.20 78464.03
148 78464.03 599.55 392.32 207.23 78256.80
149 78256.80 599.55 391.28 208.27 78048.53
150 78048.53 599.55 390.24 209.31 77839.22
151 77839.22 599.55 389.20 210.35 77628.87
152 77628.87 599.55 388.14 211.41 77417.46
153 77417.46 599.55 387.09 212.46 77205.00
154 77205.00 599.55 386.03 213.52 76991.48
155 76991.48 599.55 384.96 214.59 76776.88
156 76776.88 599.55 383.88 215.67 76561.22
157 76561.22 599.55 382.81 216.74 76344.47
158 76344.47 599.55 381.72 217.83 76126.65
159 76126.65 599.55 380.63 218.92 75907.73
160 75907.73 599.55 379.54 220.01 75687.72
161 75687.72 599.55 378.44 221.11 75466.61
162 75466.61 599.55 377.33 222.22 75244.39
163 75244.39 599.55 376.22 223.33 75021.06
164 75021.06 599.55 375.11 224.44 74796.62
165 74796.62 599.55 373.98 225.57 74571.05
166 74571.05 599.55 372.86 226.69 74344.36
167 74344.36 599.55 371.72 227.83 74116.53
168 74116.53 599.55 370.58 228.97 73887.56
169 73887.56 599.55 369.44 230.11 73657.45
170 73657.45 599.55 368.29 231.26 73426.19
171 73426.19 599.55 367.13 232.42 73193.77
172 73193.77 599.55 365.97 233.58 72960.19
173 72960.19 599.55 364.80 234.75 72725.44
174 72725.44 599.55 363.63 235.92 72489.51
175 72489.51 599.55 362.45 237.10 72252.41
176 72252.41 599.55 361.26 238.29 72014.12
177 72014.12 599.55 360.07 239.48 71774.64
178 71774.64 599.55 358.87 240.68 71533.97
179 71533.97 599.55 357.67 241.88 71292.09
180 71292.09 599.55 356.46 243.09 71049.00
181 71049.00 599.55 355.24 244.31 70804.69
182 70804.69 599.55 354.02 245.53 70559.17
183 70559.17 599.55 352.80 246.75 70312.41
184 70312.41 599.55 351.56 247.99 70064.42
185 70064.42 599.55 350.32 249.23 69815.20
186 69815.20 599.55 349.08 250.47 69564.72
187 69564.72 599.55 347.82 251.73 69313.00
188 69313.00 599.55 346.56 252.99 69060.01
288
189 69060.01 599.55 345.30 254.25 68805.76
190 68805.76 599.55 344.03 255.52 68550.24
191 68550.24 599.55 342.75 256.80 68293.44
192 68293.44 599.55 341.47 258.08 68035.36
193 68035.36 599.55 340.18 259.37 67775.98
194 67775.98 599.55 338.88 260.67 67515.31
195 67515.31 599.55 337.58 261.97 67253.34
196 67253.34 599.55 336.27 263.28 66990.06
197 66990.06 599.55 334.95 264.60 66725.46
198 66725.46 599.55 333.63 265.92 66459.53
199 66459.53 599.55 332.30 267.25 66192.28
200 66192.28 599.55 330.96 268.59 65923.69
201 65923.69 599.55 329.62 269.93 65653.76
202 65653.76 599.55 328.27 271.28 65382.48
203 65382.48 599.55 326.91 272.64 65109.84
204 65109.84 599.55 325.55 274.00 64835.84
205 64835.84 599.55 324.18 275.37 64560.47
206 64560.47 599.55 322.80 276.75 64283.72
207 64283.72 599.55 321.42 278.13 64005.59
208 64005.59 599.55 320.03 279.52 63726.07
209 63726.07 599.55 318.63 280.92 63445.15
210 63445.15 599.55 317.23 282.32 63162.83
211 63162.83 599.55 315.81 283.74 62879.09
212 62879.09 599.55 314.40 285.15 62593.94
213 62593.94 599.55 312.97 286.58 62307.36
214 62307.36 599.55 311.54 288.01 62019.34
215 62019.34 599.55 310.10 289.45 61729.89
216 61729.89 599.55 308.65 290.90 61438.99
217 61438.99 599.55 307.19 292.36 61146.63
218 61146.63 599.55 305.73 293.82 60852.82
219 60852.82 599.55 304.26 295.29 60557.53
220 60557.53 599.55 302.79 296.76 60260.77
221 60260.77 599.55 301.30 298.25 59962.52
222 59962.52 599.55 299.81 299.74 59662.79
223 59662.79 599.55 298.31 301.24 59361.55
224 59361.55 599.55 296.81 302.74 59058.81
225 59058.81 599.55 295.29 304.26 58754.55
226 58754.55 599.55 293.77 305.78 58448.77
227 58448.77 599.55 292.24 307.31 58141.47
228 58141.47 599.55 290.71 308.84 57832.63
229 57832.63 599.55 289.16 310.39 57522.24
230 57522.24 599.55 287.61 311.94 57210.30
231 57210.30 599.55 286.05 313.50 56896.80
232 56896.80 599.55 284.48 315.07 56581.74
233 56581.74 599.55 282.91 316.64 56265.09
234 56265.09 599.55 281.33 318.22 55946.87
235 55946.87 599.55 279.73 319.82 55627.05
236 55627.05 599.55 278.14 321.41 55305.64
289
237 55305.64 599.55 276.53 323.02 54982.62
238 54982.62 599.55 274.91 324.64 54657.98
239 54657.98 599.55 273.29 326.26 54331.72
240 54331.72 599.55 271.66 327.89 54003.83
241 54003.83 599.55 270.02 329.53 53674.30
242 53674.30 599.55 268.37 331.18 53343.12
243 53343.12 599.55 266.72 332.83 53010.29
244 53010.29 599.55 265.05 334.50 52675.79
245 52675.79 599.55 263.38 336.17 52339.62
246 52339.62 599.55 261.70 337.85 52001.76
247 52001.76 599.55 260.01 339.54 51662.22
248 51662.22 599.55 258.31 341.24 51320.98
249 51320.98 599.55 256.60 342.95 50978.04
250 50978.04 599.55 254.89 344.66 50633.38
251 50633.38 599.55 253.17 346.38 50287.00
252 50287.00 599.55 251.43 348.12 49938.88
253 49938.88 599.55 249.69 349.86 49589.02
254 49589.02 599.55 247.95 351.60 49237.42
255 49237.42 599.55 246.19 353.36 48884.06
256 48884.06 599.55 244.42 355.13 48528.93
257 48528.93 599.55 242.64 356.91 48172.02
258 48172.02 599.55 240.86 358.69 47813.33
259 47813.33 599.55 239.07 360.48 47452.85
260 47452.85 599.55 237.26 362.29 47090.56
261 47090.56 599.55 235.45 364.10 46726.47
262 46726.47 599.55 233.63 365.92 46360.55
263 46360.55 599.55 231.80 367.75 45992.80
264 45992.80 599.55 229.96 369.59 45623.21
265 45623.21 599.55 228.12 371.43 45251.78
266 45251.78 599.55 226.26 373.29 44878.49
267 44878.49 599.55 224.39 375.16 44503.33
268 44503.33 599.55 222.52 377.03 44126.30
269 44126.30 599.55 220.63 378.92 43747.38
270 43747.38 599.55 218.74 380.81 43366.57
271 43366.57 599.55 216.83 382.72 42983.85
272 42983.85 599.55 214.92 384.63 42599.22
273 42599.22 599.55 213.00 386.55 42212.67
274 42212.67 599.55 211.06 388.49 41824.18
275 41824.18 599.55 209.12 390.43 41433.75
276 41433.75 599.55 207.17 392.38 41041.37
277 41041.37 599.55 205.21 394.34 40647.03
278 40647.03 599.55 203.24 396.31 40250.71
279 40250.71 599.55 201.25 398.30 39852.41
280 39852.41 599.55 199.26 400.29 39452.13
281 39452.13 599.55 197.26 402.29 39049.84
282 39049.84 599.55 195.25 404.30 38645.54
283 38645.54 599.55 193.23 406.32 38239.21
284 38239.21 599.55 191.20 408.35 37830.86
290
285 37830.86 599.55 189.15 410.40 37420.46
286 37420.46 599.55 187.10 412.45 37008.02
287 37008.02 599.55 185.04 414.51 36593.51
288 36593.51 599.55 182.97 416.58 36176.92
289 36176.92 599.55 180.88 418.67 35758.26
290 35758.26 599.55 178.79 420.76 35337.50
291 35337.50 599.55 176.69 422.86 34914.64
292 34914.64 599.55 174.57 424.98 34489.66
293 34489.66 599.55 172.45 427.10 34062.56
294 34062.56 599.55 170.31 429.24 33633.32
295 33633.32 599.55 168.17 431.38 33201.94
296 33201.94 599.55 166.01 433.54 32768.40
297 32768.40 599.55 163.84 435.71 32332.69
298 32332.69 599.55 161.66 437.89 31894.80
299 31894.80 599.55 159.47 440.08 31454.73
300 31454.73 599.55 157.27 442.28 31012.45
301 31012.45 599.55 155.06 444.49 30567.96
302 30567.96 599.55 152.84 446.71 30121.25
303 30121.25 599.55 150.61 448.94 29672.31
304 29672.31 599.55 148.36 451.19 29221.12
305 29221.12 599.55 146.11 453.44 28767.68
306 28767.68 599.55 143.84 455.71 28311.96
307 28311.96 599.55 141.56 457.99 27853.97
308 27853.97 599.55 139.27 460.28 27393.69
309 27393.69 599.55 136.97 462.58 26931.11
310 26931.11 599.55 134.66 464.89 26466.22
311 26466.22 599.55 132.33 467.22 25999.00
312 25999.00 599.55 129.99 469.56 25529.44
313 25529.44 599.55 127.65 471.90 25057.54
314 25057.54 599.55 125.29 474.26 24583.28
315 24583.28 599.55 122.92 476.63 24106.65
316 24106.65 599.55 120.53 479.02 23627.63
317 23627.63 599.55 118.14 481.41 23146.22
318 23146.22 599.55 115.73 483.82 22662.40
319 22662.40 599.55 113.31 486.24 22176.16
320 22176.16 599.55 110.88 488.67 21687.49
321 21687.49 599.55 108.44 491.11 21196.38
322 21196.38 599.55 105.98 493.57 20702.81
323 20702.81 599.55 103.51 496.04 20206.77
324 20206.77 599.55 101.03 498.52 19708.26
325 19708.26 599.55 98.54 501.01 19207.25
326 19207.25 599.55 96.04 503.51 18703.74
327 18703.74 599.55 93.52 506.03 18197.70
328 18197.70 599.55 90.99 508.56 17689.14
329 17689.14 599.55 88.45 511.10 17178.04
330 17178.04 599.55 85.89 513.66 16664.38
331 16664.38 599.55 83.32 516.23 16148.15
332 16148.15 599.55 80.74 518.81 15629.34
291
333 15629.34 599.55 78.15 521.40 15107.94
334 15107.94 599.55 75.54 524.01 14583.93
335 14583.93 599.55 72.92 526.63 14057.30
336 14057.30 599.55 70.29 529.26 13528.03
337 13528.03 599.55 67.64 531.91 12996.12
338 12996.12 599.55 64.98 534.57 12461.56
339 12461.56 599.55 62.31 537.24 11924.31
340 11924.31 599.55 59.62 539.93 11384.38
341 11384.38 599.55 56.92 542.63 10841.76
342 10841.76 599.55 54.21 545.34 10296.42
343 10296.42 599.55 51.48 548.07 9748.35
344 9748.35 599.55 48.74 550.81 9197.54
345 9197.54 599.55 45.99 553.56 8643.98
346 8643.98 599.55 43.22 556.33 8087.65
347 8087.65 599.55 40.44 559.11 7528.53
348 7528.53 599.55 37.64 561.91 6966.63
349 6966.63 599.55 34.83 564.72 6401.91
350 6401.91 599.55 32.01 567.54 5834.37
351 5834.37 599.55 29.17 570.38 5263.99
352 5263.99 599.55 26.32 573.23 4690.76
353 4690.76 599.55 23.45 576.10 4114.67
354 4114.67 599.55 20.57 578.98 3535.69
355 3535.69 599.55 17.68 581.87 2953.82
356 2953.82 599.55 14.77 584.78 2369.04
357 2369.04 599.55 11.85 587.70 1781.33
358 1781.33 599.55 8.91 590.64 1190.69
359 1190.69 599.55 5.95 593.60 597.09
360 597.09 599.55 2.99 596.56 0.53
292
Now suppose that instead of making the $599.55 monthly
payments, we make a $699.55 monthly payment. How much
sooner will we pay off the loan?
PMT = 699.55
PV = -100,000
I = 0.5
CPT N = ?
293
More Time Lines
You have just entered the following information into your
calculator. The cash flows you are entering into the calculator
occur, as shown on the following time line. (Note, your calculator
is set in the beg mode)
PMT = $1,000
N=5
I = 10
FV = ?
PV = $6,715.61
I = 10
N=2
FV = ?
0 1 2 3 4 5 6 7 8
|---------|---------|---------|---------|---------|---------|---------|-------|
1,000 1,000 1,000 1,000 1,000
294
Perpetuities
Characteristics of a Perpetuity
Stream of Cash Yes Limited Number Yes Equal Time Yes Equal Dollar Yes
Annuity
Flows? of Cash Flows? Spacing? Amount?
No No No No
No No
No
Combination Combination
Problem Problem
295
Example Time Line for a Perpetuity
0 1 2 3 4 5 ∞
|----------|----------|----------|----------|-----------|……----
100 100 100 100 100
296
Example 1: Suppose you are considering making an investment
that promises to pay you $1,000 per year into infinity. You
will receive the first payment one year from today. You
require a 12 percent return on this type of investment. What
should you be willing to pay for this investment today?
0 1 2 3 4 5 ∞
|----------|----------|----------|----------|-----------|……----
1000 1000 1000 1000 1000
?
PMT 1 1000
PVOP = = = $8,333.33
kreq .12
297
Example 2: Suppose you are considering making an investment
that promises to pay you $1,000 per year into infinity. You
will receive the first payment 5 minutes after you purchase
the investment. You require a 12 percent return on this type
of investment. What should you be willing to pay for this
investment today?
0 1 2 3 4 5 ∞
|----------|----------|----------|----------|-----------|……----
1000 1000 1000 1000 1000 1000
?
298
Growing Perpetuity
Characteristics of a Growing Perpetuity
Stream of Cash Yes Limited Number Yes Equal Time Yes Equal Dollar Yes
Annuity
Flows? of Cash Flows? Spacing? Amount?
No No No No
No No
No
Combination Combination
Problem Problem
299
Example Time Line for a Growing Perpetuity
0 1 2 3 4 5 ∞
|----------|----------|----------|----------|-----------|……----
100 110 121 133.10 146.41
300
Example: Suppose you expect to receive a payment of $10 one
year from today from an investment. You will continue to receive
payments each year forever. The payments will become 5 percent
larger in each successive year. You require a 10 percent return on
this investment. What is the present value of this stream of cash
flows? (that is what is the value of the cash flows at time zero.
0 1 2 3 4 5 ∞
|----------|----------|----------|----------|-----------|……----
10 10.50 11.025 11.58 12.16
PMT 1 PMT − 0 (1 + g )
PVGOP = =
kreq − g kreq − g
Which form of the formula you use depends upon the information
you have available in the problem. PMT1 or PMT-0
PMT 1 10
PVGOP = = = $200
kreq − g .10 − .05
301
Growing Ordinary Perpetuities
Example
-0 0 1 2 3 4 5 ∞
|---|----------|----------|----------|----------|-----------|……----
$9.524 $10 10.50 11.025 11.58 12.16
PMT 1 PMT − 0 (1 + g )
PVGOP = =
kreq − g kreq − g
302
Example of Growing Perpetuity Due
PMT 0 (1 + g )
PVGPD = + PMT 0
kreq − g
PMT 0 (1 + g ) 10(1.05)
PVGPD = + PMT 0 = + 10 = $220
kreq − g .10 − .05
303
Uneven Cash Flow Streams
Stream of Cash Yes Limited Number Yes Equal Time Yes Equal Dollar Yes
Annuity
Flows? of Cash Flows? Spacing? Amount?
No No No No
No No
No
Combination Combination
Problem Problem
304
Example of a time line for an Uneven Cash Flow Stream
0 1 2 3 4 5 6 7
|----------|----------|----------|----------|----------|----------|----------|
305
Present Value of an Uneven Cash Flow Stream
0 1 2 3 4
|----------|----------|----------|----------|
We can solve for the present value of an uneven cash flow stream
using the Cash Flow Register in our calculator as follows:
CF
CF0 = 100 ↵ ↓
C01 = 200 ↵ ↓↓
C02 = 0 ↵ ↓↓
C03 = 500 ↵ ↓↓
C04 = 500 ↵ ↓↓
NPV
I = 12 ↵ ↓
CPT NPV = ? NPV = $952.22
306
Future Value of an Uneven Cash Flow Stream
Suppose we will deposit the following cash flows in the bank into
an account that pays 12 percent nominal interest compounded
annually. How much money will we have in the bank four years
from today?
0 1 2 3 4
|----------|----------|----------|----------|
PV = 952.22
I = 12
N=4
CPT FV = ? = $1,498.34
307
Example of an Uneven Cash Flow Problem
Suppose you deposit $5,000 per year into an account at the end of
each of the next 3 years, and $6,000 into the account each year for
the following 7 years. How much money will you have in the
account at the end of the 10th year? Assume that we earn 10
percent nominal interest compounded annually on our investments.
0 1 2 3 4 5 6 7 8 9 10
|-------|-------|-------|-------|-------|------|------|------|------|------|
$5K $5K $5K $6K $6K $6K $6K $6K $6K $6K
?
CF
CF0 = 0 ↵ ↓
C01 = 5,000 ↵ ↓
F01 = 3 ↵ ↓
C02 = 6,000 ↵ ↓
F02 = 7 ↵
NPV
I = 10 ↵ ↓
NPV
CPT NPV = 34,380.55
308
Step 2: Compute the Future Value of a Single Sum
PV = 34,380.55
I = 10
N = 10
CPT FV = ?
FV = $89,174.29
309
Solving for the Interest Rate in an Uneven Cash Flow Problem
In order to solve this problem, we enter the cash flows into the
calculator as before
CF0 = -1000 ↵ ↓
C01 = 500 ↵ ↓↓
C02 = 700 ↵ ↓↓
C03 = 400 ↵ ↓↓
IRR
CPT
IRR = 28.61%
Note that solving for the interest rate is only possible if there is
exactly one sign change in the cash flows. It is not possible if
there are no cash flow sign changes or if there is more than one
sign change like there is in the following example
CF0 = -1000 ↵ ↓
C01 = 500 ↵ ↓ ↓
C02 = 700 ↵ ↓ ↓
C03 = -400 ↵ ↓ ↓
310
Combination Problems
Sometimes we have an unusual problem that does not fit into any
of the above classifications. In this case we need to use more than
one technique to solve the problem.
Stream of Cash Yes Limited Number Yes Equal Time Yes Equal Dollar Yes
Annuity
Flows? of Cash Flows? Spacing? Amount?
No No No No
No No
No
Combination Combination
Problem Problem
311
Example of a Combination Problem
0 1 2 3 4 5 6 7 8 ∞
|--------|--------|--------|--------|--------|--------|--------|--------|-----|-
$100 $100 $100 $110 $121 $133.1 ………
?
312
Step 2: Examine the cash flows in two parts. The first part is
after time period three. The cash flows after time period three
are a growing ordinary perpetuity.
PMT 1 PMT − 0 (1 + g )
PVGOP0 = =
kreq − g kreq − g
PMT 1
PVGOP0 =
kreq − g
PMT 4
PVGOP3 =
kreq − g
So we have:
110
PVGOP3 = = $5,500
.12 − .10
313
Step 3: Redraw the Time Line to reflect our computations
0 1 2 3 4 5 6 7 8 9
|--------|--------|--------|--------|--------|--------|--------|--------|--------|
$100 $100 $100
$5,500
$5,600
0 1 2 3 4 5 6 7 8 9
|--------|--------|--------|--------|--------|--------|--------|--------|--------|
$100 $100 $5,600
?
Step 4: The above cash flows are an uneven cash flow stream. So
we can compute the present value of an uneven cash flow
stream.
C01 = 100
C02 = 100
C03 = 5600
NPV
I = 12
NPV = ? = $4,154.97
314
Notes on Using Microsoft Excel to complete time value of
money computations.
You can use the function fx on the Excel menus to access the
function list. From here, you can select, FV, PV, PMT, IRR or
NPV, which will bring up a dialog box for you. You enter the data
and the computer will output the appropriate answer.
Time CF
0 -100
1 50
2 150
3 200
In the NPV dialog box, you will enter cash flows 0-3 for
"Value 1", and 0.12 for the rate.
315
The correct way to solve the problem:
In the NPV dialog box, you will enter cash flows 1-3 for "Value
1", and 0.12 for the rate. DO NOT INCLUDE THE VALUE FOR
TIME 0 HERE.
=NPV(D4,D7:D9)
Where "D4" = 0.12 and "D7:D9" is the series of cash flows, 50,
100, 200
You must edit the formula Excel produced (by subtracting the time
zero cash flow) as follows to get the correct answer.
316
Chapter 8 Homework and Solutions
317
Chapter 8 Homework
1. Which buttons do you need to push on your calculator to set your calculator for use in this class?
3. You will deposit $1,000 today into an account. Calculate how much money you will have in the
account under each of the following conditions.
A. You earn 6 percent nominal interest compounded annually. You leave the money in the
account for one year.
B. You earn 6 percent nominal interest compounded annually. You leave the money in the
account for two years.
C. You earn 6 percent nominal interest compounded annually. You leave the money in the
account for thirty years.
D. You earn 6 percent nominal interest compounded annually. You leave the money in the
account for forty years.
E. Do the above answers tell you anything about the plans you should make for retirement?
4. Now suppose that rather than earning six percent interest, you can earn ten percent nominal
interest compounded annually. Rework the above problem using this information.
5. Now suppose that you can earn 10 percent nominal interest compounded semi-annually on your
investments. Rework parts A-D of Problem 3 using this information.
6. Your grandmother has just purchased an investment as a gift for you (The investment is a
government bond). The investment is set up so that you will not get any money today, however;
you will get a lump sum amount at some time in the future. You want to know what the
investment is worth to you today under each of the following conditions.
A. You will receive $1,000,000 from the investment 1 year from today (you have got to love
that!!!!). The appropriate interest rate is 6 percent nominal compounded annually.
B. You will receive $1,000,000 from the investment two years from today. The appropriate
interest rate is 6 percent nominal compounded annually.
C. You will receive $1,000,000 from the investment 30 years from today. The appropriate
interest rate is 6 percent nominal compounded annually.
D. You will receive $1,000,000 from the investment 40 years from today . The appropriate
interest rate is 6 percent nominal compounded annually.
E. When would you like to receive the money and why?
7. Now suppose the appropriate interest rate is 10 percent nominal compounded annually rather than
6 percent in the above problem.
I. Rework parts A-D of the above problem using the new information.
II. What do these results tell us about how the discount rate affects the present value?
318
8. Now suppose the appropriate interest rate is 10 percent nominal compounded monthly in the
above problem.
I. Rework parts A-D of the above problem using the new information.
II. What does this problem tell us about compounding periods and present values.
9. You go to the First Hawaiian Bank. They tell you that you can place your money into one of two
accounts. The accounts are identical except for the interest rate that they pay. Compare each of
the following pairs of accounts to determine which of the two accounts is best for you.
10. You go to the bank today to deposit $10,000 into an account that pays 5 percent nominal interest,
compounded continuously. How much money will you have in the bank ten years from today if
you do not take any money out of the account?
11. You wish to have $2,000,000 available for retirement 30 years from today. The bank tells you
they will pay you 9 percent nominal interest, compounded continuously on the account. How
much do you need to deposit today to achieve your goal?
12. What are the four conditions that must be met in order for a series of cash flows to be an annuity?
13. Draw a time line that illustrates the point in time when we calculate the value of an annuity if we
calculate the present value of an ordinary annuity, the present value of an annuity due, the future
value of an ordinary annuity, and the future value of an annuity due.
14. THIS HOMEWORK QUESTION HAS BEEN REMOVED. A NEW QUESTION WILL BE
ADDED TO REPLCE IT NEXT SEMESTER.
15. THIS HOMEWORK QUESTION HAS BEEN REMOVED. A NEW QUESTION WILL BE
ADDED TO REPLCE IT NEXT SEMESTER.
16. You have just won the largest Power Ball lottery of all time. You won a $150,000,000 lottery
Congratulations!!!!. Of course, you have decided that your education is important and you will
continue your studies…. Particularly your finance studies so you know how to invest all of that
money wisely. The lottery is paid in 20 equal annual installments of $7,500,000 each. You will
receive the first payment as soon as the armored car that you have rented gets you and the winning
lottery ticket to the lottery office. You are a little bit greedy though, and decide that you would
like to get your hands on as much money as you possibly can today. So you stop at the bank and
tell them that you will give them all of the payments for a single lump sum today. The bank says
they will buy the payments from you. The bank says they will use a 7.5 percent nominal annually
compounded interest to calculate the present value of the payments, which is what you will
receive. How much will the bank give you today?
17. You are not entirely satisfied with the amount that the bank has indicated they will give you in the
319
previous problem. Just before you sign the papers, you change your mind and decide not to sell
the payments to the bank. The bank, eager to get your business says that they will give you
$87,600,000 today instead of the amount you calculated in the previous problem. What interest
rate is the bank using to calculate the present value of the payments now?
18. Calculate the present value (that is the value at time 0) of the following:
A. An investment that will give you $1,000 at the end of each of the next 10 years. The
appropriate interest rate is 10 percent nominal compounded annually.
B. An investment that will give you $1,000 at the end of each of the next 10 years. The
appropriate interest rate is 20 percent nominal compounded annually.
C. An investment that will give you $1,000 at the end of each of the next 20 years. The
appropriate interest rate is 10 percent nominal compounded annually.
D. An investment that will give you $1,100 at the end of each of the next 10 years. The
appropriate interest rate is 10 percent nominal compounded annually.
19. Now suppose that you will receive the payments in the above problem at the beginning of each
year rather than at the end of each year. Re-compute the answers for the above problem using this
new information.
A. You deposit $1,000 per year at the end of each of the next 20 years into an account that
pays 10 percent nominal interest compounded annually. How much will you have in the
account at the end of the 20th year?
B. You deposit $2,000 per year at the end of each of the next 20 years into an account that
pays 10 percent nominal interest compounded annually. How much will you have in the
account the end of the 20th year?
C. You deposit $1,000 per year at the end of each of the next 10 years into an account that
pays 10 percent nominal interest compounded annually. How much will you have in the
account at the end of the 10th year?
D. You deposit $1,000 per year at the end of each of the next 20 years into an account that
pays 12 percent nominal interest compounded annually. How much will you have in the
account at the end of the 20th year?
A. You deposit $1,000 per year at the beginning of each of the next 20 years into an account
that pays 10 percent nominal interest compounded annually. How much will you have in
the account at the end of the 20th year?
B. You deposit $2,000 per year at the beginning of each of the next 20 years into an account
that pays 10 percent nominal interest compounded annually. How much will you have in
the account the end of the 20th year?
C. You deposit $1,000 per year at the beginning of each of the next 10 years into an account
that pays 10 percent nominal interest compounded annually. How much will you have in
the account at the end of the 10th year?
D. You deposit $1,000 per year at the beginning of each of the next 20 years into an account
that pays 12 percent nominal interest compounded annually. How much will you have in
the account at the end of the 20th year?
320
22. Suppose I deposit $10,000 into the bank at the end of each of the next 6 years. The account will
pay 10 percent nominal interest compounded annually. How much money will I have in the bank
7 years from today?
23. You are considering buying a car. The car dealer says the price of your dream machine is
$20,000. You will borrow all $20,000. You plan to make monthly payments on the car for the
next 5 years. The first payment will be made today. The interest rate on this loan is 6.99 percent
nominal compounded monthly.
B. You are also considering a lower priced car. How much will your payments be if you
borrow $14,000 instead of $20,000. All other features of the loan will be the same.
C. You decide you really don’t want to buy a car after all. Instead, you will buy a house. In
order to buy the house you will borrow $100,000. The loan will be for 30 years. You
will make monthly payments with the first payment due one month after you buy the
house. The interest rate on the loan is 6.75 percent compounded monthly. How much
will the payments on the house be?
24. You recently purchased a new home. You paid $150,000 today for the home and financed the entire
purchase at an 8 percent nominal interest rate compounded monthly. You are scheduled to pay off
the loan by making monthly payments for the next thirty years. The first payment will be due one
month from today. As such, your monthly payment will be $1,100.65. You decide however, that
instead of making the required $1100.65 payment each month, you will increase the amount of the
payment you will make. How long will it take you to pay off the loan under each of the following
conditions.
25. What conditions must be met for a series of cash flows to be a perpetuity?
26. A. You are considering making an investment in a stock that will give you $100 each year
forever. You will receive the first payment one year from today. You have determined
that you require a 10 percent nominal return on this type of investment. What is the
present value of these cash flows?
B. You have re-evaluated the above decision and determined that you require a 12 percent
nominal return on this investment. What is the present value of these cash flows now?
C. You are looking into another investment that will give you $500 each year forever. You
will receive the first payment one year from today. You require a 25 percent nominal
return on this type of investment. What is the present value of these cash flows?
27. Rework the above problem assuming that you will get the first payment in 5 minutes rather than
one year from today.
28. What are the conditions necessary for a series of cash flows to be a growing perpetuity?
321
29.
A. Suppose you are considering investing in a stock. The stock will give you a $100
payment one year from today. The stock will continue to give you one payment per year
forever. However, each year the payment will become 10 percent larger. You require a
15 percent rate of return on this investment. What is the present value of the cash flows
you will receive from this stock?
B. Suppose you re-examine the above cash flows and determine that you will get $110 one
year from today rather than $100. The required rate of return is still 15 percent and the
payments will become larger by 10 percent per year. What is the present value of the
cash flows you will receive from this stock?
C. Suppose you re-examine the above problem and determine that you will get $110 one
year from today. However, the required rate of return should be 20 percent instead of 15
percent and the growth rate should be 10 percent. What is the present value of the cash
flows you will receive from this stock?
D. Suppose yet again you re-examine the above problem and determine that you will get
$110 one year from today. The required rate of return is 20 percent, but the growth rate
will only be 5 percent. What is the present value of the cash flows you will receive from
this stock?
30. That **0(///^^5%%% finance professor. He gave you the wrong information for the above
problem. The payments that are given (100 and 110 in the above problem) are the payments that
were just received 5 minutes ago, not the payments you will receive one year from today. Re-
compute the answers to parts A-D using the correct information.
31. You will receive $100 from an investment 5 minutes after you purchase it. You will continue to
receive payments each year forever, however the payments will become 7 percent larger in each
successive year. You require a 14 percent return on this type of investment. What should you be
willing to pay to buy this investment today?
32. What are the conditions for a series of cash flows to be an uneven cash flow stream?
33. What is the present value (at time 0) of the following cash flow stream?
322
34. Using the information from the above problem. Compute the future value of the cash flows at the
end of the fifth year for parts A-D.
35. How long will it take for your money to double at each of the following interest rates?
36. You are considering buying a house. In order to buy the house you will borrow $100,000 today at
a nominal interest rate of 6.75 percent, compounded monthly. You will make monthly payments
with the first payment due one month after you buy the house.
37. You will put $1000 in the bank at the end of each of the next 4 years. You are saving the money
to go to graduate school. You have consulted your finance professor who says the type of degree
you are interested in will take two years to earn. You will start graduate school 4 years from
today. You want to take two equal payments out of the account, one at the beginning of each year
of graduate school. You will take the first payment out on the day that you start graduate school
and the second payment out one year later. You can earn 10 percent interest compounded
annually on your investments. How much will you be able to take out of the account each year to
pay for graduate school?
38. We will deposit $500 in to an account at the end of each of the next 5 years. We will stop making
deposits at the end of the 5th year, however, we will not take any money out of the account. How much
money will we have in the account at the end of the 10th year. We will earn 10 percent interest on the
account. Show how to solve the problem in two ways. The first way will be to use an ordinary
annuity, and the second way will be to use an annuity due.
39. Your son Bob is 12 years old today. You are planning for his college education. Bob will start school
on his 19th birthday. You wish to set aside some money early to send Bob to four years of school.
You have decided that you will give Bob 16,000 per year for each of his first two years of college, and
20,000 per year for each of his last two years of college. You will give these amounts to Bob at the
beginning of each school year.
You will make 7 equal annual deposits to fund the account. The first payment will be made one year
from today and the last payment will be made the day Bob leaves for college. You wish to have just
enough money in the bank to fund Bobs entire education on the day that he leaves for school. Any
money that is in the account will continue to earn interest while Bob is in school. Because of a new
program, the bank has agreed to give you a 10 percent, nominal compounded annually, return on your
investments throughout the entire time period. How much do you need to deposit into the account in
each of the 7 years in order to fund Bobs education?
40. Marvin Urlacher is 25 years old today. Urlacher wants to be prepared for his retirement, and so is
doing his planning early. He has already saved $10,000 toward retirement. Urlacher plans to
323
save $4,000 per year at the end of each of the next 15 years and $5,000 per year at the end of each
of the following 15 years, and 6,000 per year for the following 5 years. Urlacher plans to retire on
his 60th birthday. On the day he retires, he plans to spend $200,000 of the money he has
accumulated to purchase a house. Urlacher will live 25 years beyond retirement. He wishes to
have a contingency fund of $100,000 available (in addition to his home) at the time of his death to
pay for funeral and other expenses that may arise in relation to his death. How much money can
Urlacher withdraw at the beginning of each year of retirement and still meet his other goals? Each
of the withdrawals will be for an equal amount. Assume a 10 percent nominal interest rate
compounded annually throughout the entire problem. Make sure to draw a time line to show when
the payments are being made and received.
41.
Suppose you will receive $200 from an investment at the end of each of the next four years. After that you
will continue to receive payments each year forever, but the payments will become 10 percent larger in
each successive year. You require a 15 percent return on this type of investment. What should you be
willing to pay to buy this investment today? (that is, What is the present value of these cash flows at time
0?)
324
MICROSOFT CORP
BALANCE_SHEET
Period End: Jun 30, 2016
(In Millions)
2016 2015
Assets
Current assets:
Cash and cash equivalents $ 6,510 $ 5,595
Short-term investments (including securities loaned of $204 and $75) 106,730 90,931
------------------------------------------------------------------------------------------------------------------------------- -----------------
Total cash, cash equivalents, and short-term investments 113,240 96,526
Stockholders' equity:
Common stock and paid-in capital - shares authorized 24,000;
outstanding 7,808 and 8,027 68,178 68,465
Retained earnings 2,282 9,096
Accumulated other comprehensive income 1,537 2,522
325
MICROSOFT CORP
INCOME_STATEMENT
Period End: Jun 30, 2016
Revenue:
Product $ 61,502 $ 75,956 $ 72,948
Service and other 23,818 17,624 13,885
------------------------------------------------------------------------------------------ ---------------- ----------------
Total revenue 85,320 93,580 86,833
------------------------------------------------------------------------------------------ ---------------- ----------------
Cost of revenue:
Product 17,880 21,410 16,681
Service and other 14,900 11,628 10,397
------------------------------------------------------------------------------------------ ---------------- ----------------
Total cost of revenue 32,780 33,038 27,078
------------------------------------------------------------------------------------------ ---------------- ----------------
Gross margin 52,540 60,542 59,755
Research and development 11,988 12,046 11,381
Sales and marketing 14,697 15,713 15,811
General and administrative 4,563 4,611 4,677
Impairment, integration, and restructuring 1,110 10,011 127
------------------------------------------------------------------------------------------ ---------------- ----------------
Operating income 20,182 18,161 27,759
Other income (expense), net (431) 346 61
------------------------------------------------------------------------------------------ ---------------- ----------------
Income before income taxes 19,751 18,507 27,820
Provision for income taxes 2,953 6,314 5,746
------------------------------------------------------------------------------------------ ---------------- ----------------
Net income $ 16,798 $ 12,193 $ 22,074
----------------- ---------------- ----------------
326
1. To set the number of decimal places press 2nd Format 9 Enter
To set the number of payments per year 2nd P/Y 1 Enter
2. A. CE/C
B. 2nd CLR TVM
C. CF, 2nd CLR Work
3. A. PV = 1,000
N=1
I=6
FV = ? = $1,060
B. PV = 1,000
N=2
I=6
FV = ? = $1,123.60
C. PV = 1,000
N = 30
I=6
FV = ? = $5,743.49
D. PV = 1,000
N = 40
I=6
FV = ? = $10,285.72
4.
A. PV = 1,000
N=1
I = 10
FV = ? = $1,100
B. PV = 1,000
N=2
I = 10
FV = ? = $1,210
C. PV = 1,000
N = 30
I = 10
FV = ? = $17,449.40
D. PV = 1,000
N = 40
I = 10
FV = ? = $45,259.26
E. Yes, not only is it important to begin planning for retirement early, it is important to
monitor carefully the return we are getting on our investments.
5. A. PV = 1,000
327
N = 2 (2X1)
I = 5 (10/2)
FV = ? = $1,102.50
B. PV = 1,000
N = 4 (2X2)
I = 5 (10/2)
FV = ? = $1,215.51
C. PV = 1,000
N = 60 (30X2)
I = 5 (10/2)
FV = ? = $18,679.19
D. PV = 1,000
N = 80
I = 5 (10/2)
FV = ? = $49,561.44
6. A. FV = 1,000,000
N=1
I=6
PV = ? = $943,396.23
B. FV = 1,000,000
N=2
I=6
PV = ? = $889,996.44
C. FV = 1,000,000
N = 30
I=6
PV = ? = $174,110.13
D. FV = 1,000,000
N = 40
I=6
PV = ? = $97,222.19
E. You would like the money as soon as possible because money received today is worth more
than money received in the future.
7.
I.
A. FV = 1,000,000
N=1
I = 10
PV = ? = $909,090.91
B. FV = 1,000,000
N=2
I = 10
PV = ? = $826,446.28
C. FV = 1,000,000
328
N = 30
I = 10
PV = ? = $57,308.55
D. FV = 1,000,000
N = 40
I = 10
PV = ? = $22,094.93
II. The higher the discount rate is, the lower the present value will be.
8.
I.
A. FV = 1,000,000
N = 12
I = 0.8333333333
PV = ? = $905,212.43
B. FV = 1,000,000
N = 24
I = 0.8333333333
PV = ? = $819,409.54
C. FV = 1,000,000
N = 360
I = 0.833333333333
PV = ? = $50,409.83
D. FV = 1,000,000
N = 480
I = 0.833333333333
PV = ? = $18,621.74
II. The more frequently we compound the interest rate, the lower the present value will be.
9. You can solve this problem by using the interest rate conversion function in your calculator.
2nd Iconv
Nom = 10 ↵ ↓↓
C/Y = 12 ↵ ↓
EFF = CPT = 10.47
329
D. 10 percent nominal compounded quarterly = 10.38 percent compounded annually.
10.2 percent nominal compounded monthly = 10.69 percent compounded annually.
So, 10.2 percent nominal compounded monthly is better.
13.
0 1 2 3 4 5 6 7
|----------|----------|----------|----------|----------|----------|----------|
First Last
PMT PMT
PV PV FV FV
OA AD OA AD
14. THIS QUESTION HAS BEEN REMOVED AND WILL BE REPLACED NEXT SEMESTER
15. THIS QUESTION HAS BEEN REMOVED AND WILL BE REPLACED NEXT SEMESTER
PMT = 7,500,000
N = 20
I = 7.5
PV = ? = $82,193,086.58
PMT = 7,500,000
N = 20
PV = -87,600,000
I = ? = 6.57 percent
A. PMT = 1,000
N = 10
I = 10
330
PV = ? = $6,144.57
B. PMT = 1,000
N = 10
I = 20
PV = ? = $4,192.47
C. PMT = 1,000
N = 20
I = 10
PV = ? = $8,513.56
D. PMT = 1,100
N = 10
I = 10 PV = ? = $6,759.02
19. Your calculator should be set in the beg mode
A. PMT = 1,000
N = 10
I = 10
PV = ? = $6,759.02
B. PMT = 1,000
N = 10
I = 20
PV = ? = $5,030.97
C. PMT = 1,000
N = 20
I = 10
PV = ? = $9,364.92
D. PMT = 1,100
N = 10
I = 10
PV = ? = $7,434.93
A. PMT = 1,000
N = 20
I = 10
FV = ? = $57,275
B. PMT = 2,000
N = 20
I = 10
FV = ? = $114,550.00
C. PMT = 1,000
N = 10
I = 10
FV = ? = $15,937.42
D. PMT = 1,000
331
N = 20
I = 12
FV = ? = $72,052.44
A. PMT = 1,000
N = 20
I = 10
FV = ? = $63002.50
B. PMT = 2,000
N = 20
I = 10
FV = ? = $126,005.00
C. PMT = 1,000
N = 10
I = 10
FV = ? = $17,531.17
D. PMT = 1,000
N = 20
I = 12
FV = ? = $80,698.74
22. You want to know the value one time period after the last cash flow. Thus you compute the future
value of an annuity due (FVAD, BEG).
PMT = 10,000
N=6
I = 10
CPT FV = ?
FV = $84,871.71
A. PV = 20,000
N = 60 (12X5)
I = 0.5825 (6.99/12)
PMT = ? = $393.64
B. PV = 14000
N = 60 (12X5)
I = 0.5825 (6.99/12)
PMT = ? = $275.55
PV = 100,000
N = 360 (12X30)
I = 0.5625 (6.75/12)
PMT = ? = $648.60
332
24. Solving this problem is slightly tricky. In order to solve the problem, you must make sure that you
enter the information into your calculator with the correct signs. In this case, we have both a
present value and a future value (the future value in this case is in the form of payments). Just as
we did before, we need to enter either the present value or the future value with a negative sign.
Recall that the loan was originally set up to have 360 payments.
A. PV = -150,000
I = 8/12 = .666666667
PMT = 1,200
N = ? = 269.66 Payments
Note, that by increasing our payment by $99.35 per month, the number of payments we
need to make decreased by (360-269.66) 90.34 payments.
B. PV = -150,000
I = 8/12 = 0.666666667
PMT = 1,500
N = ? = 165.34 Payments
C. PV = -150,000
I = 8/12 = 0.666666667
PMT = 2,202
N = ? = 91.10 Payments
D. PV = -150000
I = 8/12 = 0.666666667
PMT = 5000
N = ? = 33.58 Payments
B. = 100/0.12 = $833.33
C. = 500/0.25 = $2,000
333
B. = 110/(0.15-0.10) = $2,200
C. = 110/(0.20-0.10) = $1,100
D. = 110/(0.20-0.05) = $733.33
B. = 110(1+0.10)/(0.15-0.10) = $2,420
D. = 110(1+0.05)/(0.20-0.05) = $770.00
= (100(1+0.07)/(0.14-0.07))+100
= $1,628.57
33. In order to solve this problem, you must first enter the cash flows into your cash flow register.
You do this by first pressing the CF button on your calculator. Then enter the cash flows as
follows.
A. PV = $3,170.69
B. PV = $3,010.33
C. PV = 2,859.96
D. PV = 2,718.77
34. Because you have calculate the present value of the cash flows in the previous problem, you must
simply compute the future value of a single sum.
A. PV = 3,170.69
N=5
I = 12
FV = ? = $5,587.84
B. PV = 3010.33
334
N=5
I = 14
FV = ? = $5,796.13
C. PV = 2,859.96
N=5
I = 16
FV = ? = $6,006.89
D. PV = 2,718.77
N=5
I = 18
FV = ? = $6,219.88
35. A. PV = -1
FV = 2
I = 10
N = ? = 7.27 years
B. PV = -1
FV = 2
I = 12
N = ? = 6.12 years
C. PV = -1
FV = 2
I = 14
N = ? = 5.29 years
D. PV = -1
FV = 2
I = 16
N = ? = 4.67 years
36. A. PV = 100,000
N = 360
I = 0.5625
PMT = ? = $648.60
B. PV = 100,000
N = 180
I = 0.5625
PMT = ? = $884.91
C.
D.
335
Month Beginning Balance Payment Interest Principle End Balance
1 100,000 884.91 562.50 322.41 99,677.59
2 99,677.59 884.91 560.69 324.22 99,353.37
3 99,353.37 884.91 558.86 326.05 99,027.32
0 1 2 3 4 5 6 7
37. |---------|-------|-------|-------|-------|-------|-------|
1000 1000 1000 1000
PMT PMT
The first thing we need to do to solve this problem will be to calculate the future value of the payments.
We will take the first payment out of the account at time 4, so it will be convenient to know the value of the
$1,000 payments at time 4. In order to do this we want to calculate the future value of an ordinary annuity.
1,000 = pmt
4 =n
I = 10
FV = ? = $4,641
So $4,641 is the value of the $1,000 payments at point 4 on the time line. In order to calculate the value of
the payments that we can take out of the account at time 4 and time 5, we will want to compute the
payments of an annuity due. We will use an annuity due. Why?, because we know the present value at
time 4. Time 4 is also the point in time that we want to take out the first payment. So we know the present
value at the time of the first payment. And thus we must use an annuity due.
Beg mode
PV = $4641
N=2
I = 10
PMT = ???? = $2,431
0 1 2 3 4 5 6 7 8 9 10
38. |---------|-------|-------|-------|-------|-------|-------|-------|-------|--------|
500 500 500 500 500
PMT = 500
N=5
I = 10
FV = ?? = 3,052.55
336
This is the value of the payments at time 5 on the time line. There are 5 more years until the end of the 10th
year. Thus we will calculate the future value of a single sum as follows.
PV = 3052.55
N=5
I = 10
FV = ?? = 4,916.16
This is the value of the payments at time 6 on the time line. There are 4 more years until the end of the 10th
year. Thus we will calculate the future value of a single sum as follows.
PV = 3,357.81
N=4
I = 10
FV = ?? = 4,916.16
Note you may get a slightly different answer for this problem because of rounding differences.
39.
12 13 14 15 16 17 18 19 20 21 22
|-----------|----------|----------|----------|----------|-----------|----------|----------|----------|-----------|
Step 1 is to calculate the present value of all of the money he will need to have when Bob turns 19
years old. You can do this by using the cash flow register in your calculator.
CF0 = 16,000
CF1 = 16,000
CF2 = 20,000
CF3 = 20,000
I = 10
NPV = $62,100.68
Step 2: Now we can calculate the amount of the payments that the father will have to deposit into the
account. We can use an ordinary annuity to do this. The reason that we use an ordinary annuity is
because we know the future value ($62,100.68) at the time that the father will make the last deposit
into the account.
FV = $62,100.68
N=7
I = 10
PMT = ? = $6,545.75
337
40.
There are numerous potential ways to solve this problem. I will show you one method,
However, you may use any method that gives you the correct answer.
Step 1 Calculate the future value of the deposits Urlacher will make into the account. We can do this
by calculating the future value of an uneven cash flow stream. Recall that in order to
calculate the future value of an uneven cash flow stream, we must first calculate the present
value of an uneven cash flow stream then calculate the future value of a single sum.
CF0 = 10000
CF1 = 4000
F01 = 15
CF2 = 5000
F02 = 15
CF3= 6,000
F03=5
NPV
I = 10
NPV = ? = $50,831.96
PV = $50,831.96
N = 35
I = 10
FV = $1,428,501.95
Step 3: Calculate the present value of the 100,000 contingency fund that we want to have available at
when we die at the end of the 85 th year. We want to know the value of this money on
Marvin's 60th birthday.
FV = 100000
N = 25
I = 10
PV = ? = $9,229.60
Step 4: Subtract the Present Value of the Contingency fund from the money available for
retirement needs.
338
= Money that can be used for retirement living $1,219,272.35
So, on Marvins’s 60th birthday he will have $1,228,501.95 in the bank. Of this $1,228,501.95,
Marvin will have to set aside $9,229.60. By setting aside $9,229.60 into an account that pays
10 percent interest, Marvin will have the 100,000 available upon his death. So, Marvin can
use $1,228,501.95 - $9,229.60 = $1,219,272.35 for his retirement.
Step 5: Now we can calculate the amount of the payments that Marvin can take out of the
account each year. We will use an annuity due for this calculation, because Marvin
wants to take the first payment out of the account on his 60th birthday, the time that
we know the present value to be $1,219,272.35.
PV = 1,219,272.35
N = 25
I = 10
PMT = ? = $122,113.53
Thus, Marvin can take $122,113.53 out of the account each year for retirement and
still have $100,000 available upon his death.
0 1 2 3 4 5 ∞
|--------|--------|--------|--------|--------|-------
$200 $200 $200 $200 220 ………
0 1 2 3 4 5 6 7 8 9
|--------|--------|--------|--------|--------|--------|--------|--------|--------|
$200 $200 $200 $200
4,400
4,600
0 1 2 3 4 5 6 7 8 9
|--------|--------|--------|--------|--------|--------|--------|--------|--------|
$200 $200 200 $4,600
Step 4: The above cash flows are an uneven cash flow stream. So
we can compute the present value of an uneven cash flow stream.
C01 = 200
C02 = 200
C03 = 200
C04 = 4600
I = 15 NPV = ? = $3086.71
339
42. A. This problem involves solving for the interest rate in a single sum problem.
0 1 2
|------------------------|------------------------|
-86,833 85,320
PV = -86,833
FV = 85,320
N=2
I=?
I = -0.87504 percent.
So Microsoft’s sales growth was negative, declining by 0.87504 percent per year.
*. Notice that the numbers were not converted to millions. This is because it would create a
number too large to input into the calculator. However, the result is the same if we use the “in
millions” numbers.
B. This problem involves solving for the interest rate in a single sum problem.
0 1 2
|------------------------|------------------------|
-22,074 16,798
PV = -22,074
FV = 16,798
N=2
I=?
I = -12.766 percent.
So Microsoft’s net income growth was negative, declining by 12.766 percent per year.
Thus, Microsoft’s net income declined faster than sales. This indicates issues with both sales
and profitability.
340
Chapter 10: Stock Valuation
341
The General Valuation Model
The General Valuation Model tells us that the value of any asset
can be computed by calculating the present value of the cash flows
(benefits) that the asset will provide us in the future.
The value of your car is the present value of the benefits you
receive from driving the vehicle
The value of your children is the joy you receive from watching
them grow up.
342
Valuing Your Business
The amount that you could realize in disposing your business is the
value of your company. There are several methods available to
dispose of your business. The best method depends upon the
answer to several questions.
If the Company is worth more dead than alive – You should look
to the balance sheet to determine the value of your company.
If the Company is worth more alive than dead – you should look to
the income statement to determine the value of your company.
a. other investors
b. employees
343
Notes:
Liquidation
-- Do not ignore the costs of disposing of the assets
a. real estate transaction fees
b. cost of having an employee monitor the sales
c. insurance costs
344
Valuation Models:
1. Dividend Valuation Models:
-- requires an assumption
345
Constant Growth Valuation Model
^ D − 0(1 + g ) D1
Po = =
Kreq − g Kreq − g
Where:
^
P 0 = the value of a share of the firms stock
D-0 = the amount of the most recent dividend of the company
D1 = the dividend we will receive at time 1
g = the growth rate in earnings or dividends
Kreq = the required rate of return of investors
Kreq = Ks (your book uses Ks, I use kreq) they both refer to the
same thing.
^ 10(1 + .06)
Po =
.12 − .06
= $176.67
346
The Zero Growth Dividend Valuation Model
^ D − 0(1 + g )
Po =
Kreq − g
^ D − 0(1 + 0)
Po =
Kreq − 0
^ D − 0(1 + 0)
Po =
Kreq − 0
^ D1
Po = D − 0 =
Kreq Kreq
347
Suppose that we want to determine the value of a stock that just
paid a dividend of $10 per share. These dividends are expected to
be the same amount each year forever. You require a 12 percent
return on this type of investment.
^ $10
Po =
0.12
= $83.33
348
Supernormal Growth Model
Step 1: Calculate the dividends for one more year than the
company will have supernormal growth.
Dt = Dt-1(1+g)
D1 = D-0(1+g) = $10 (1+0.3) = $13.00
D2 = D1(1+g) = $13.00 (1+0.3) = $16.90
D3 = D2(1+g) = $16.90 (1+0.06) = $17.91
0 1 2 3
|--------------|--------------|--------------|-------------> .....
13.00 16.90 17.91 ..........
349
Step 2: Calculate the value of the shares at time that the
dividends start growing at a constant rate.
Recall that after the second year the dividends will grow at a
constant rate, so we can use the constant growth model to
calculate the value of the company at the end of the second
year.
^ D3
P2 =
Kreq − g
$17.91
=
.12 − .06
= $298.50
350
Step 3: Calculate the present value of an uneven Cash Flow
Stream.
0 1 2
|--------------|--------------|
$13.00 $16.90
$298.50
$315.40
CF0 = 0
CF1 = $13.00
CF2 = $315.40
351
2. Comparables Method
Suppose that you find that the Price to Earnings ratio for
Subway is 18.
^
P o = EPS X PEC
= $10 X 18
= $180.00
352
From Share Price to Firm Value
^ ^
V o = P o X SO
^
V o = Value of Equity Stake in Company
SO = Shares Outstanding
^
V o = $250.29 X 100,000,000
^
V o = $25,029,000,000
353
Computing the Required Rate of Return
Suppose that we have determined that the return on the market will
be 11 percent per year for the foreseeable future. The risk free rate
of interest is estimated to be 6 percent. We look in Value Line and
find the beta for the stock we are analyzing to be 1.2.
Kreq = 6 + 1.2(11 - 6)
= 12 percent.
354
Chapter 10: Homework Assignment and
Solution
355
Chapter 10 Homework:
Problem 1: You are considering investing in a common stock. You will receive a payment of $30 one year
from today from the stock. You will continue to receive payments from the stock each year
forever but the payments will become 10 percent larger each year. You require a 12 percent rate
of return on this type of investment. How much is the most you should be willing to pay to buy a
share of this stock?
Problem 2: I am considering buying a share of stock. The stock will pay me $100 per year forever with
the first payment to be made at the end of the first year. I think the appropriate discount rate
(required rate of return) for this stock is 12 percent. How much is this stock worth today?
Problem 3: Your broker offers to sell you some shares of Bahnsen and Co common stock that paid a
dividend of $2.00 yesterday. You expect the dividend to grow at the rate of 5 percent per year for
the next three years, and if you buy the stock, you plan to hold it for three years and then sell it for
$34.73. You require a 12 percent rate of return on this type of investment. What is the value of
the stock today?
Problem 4: The Vegas Gambling Company has just developed a new game of chance that is very popular
with customers, and very profitable for the casino. Because of this development, The Vegas
Gambling Company is expected to experience a 12 percent annual dividend growth rate for the
next 4 years. By the end of 4 years, other firms will have developed comparable technologies.
After which, The Vegas Gambling Company’s dividend growth rate will slow to 5 percent per
year indefinitely. Investors require a return of 12 percent on The Vegas Gambling Company
Stock. The Vegas Gambling Company paid a dividend yesterday of $2.00 per share. The firm has
20 million shares outstanding. What is the current value of a share of The Vegas Gambling
Company stock? What is the value of the equity stake in the entire company?
Problem 5: The Bah Bah Black Sheep Corporation is expanding rapidly. The company is considered to be
similar to the Microsoft Corporation by some investors, but is somewhat riskier. The firm
currently retains all of its earnings. It has not paid a dividend since the inception of the company.
However, investors expect that the company will begin paying dividends in the future.
Specifically investors expect the first dividend of $2.00 to be paid 8 years from today. The
dividend is expected to grow at a 50 percent per year rate during Years 9 and 10. After Year 10,
the company should grow at a constant rate of 5 percent per year. The required return on the stock
is 20 percent, what is the value of the stock today?
Problem 6: You are considering purchasing the Big Kine Doughnut Company. You are debating how
much to pay for the company. Last year the Big Kine Doughnut Company had earnings of $15
per share. You examine the Wall Street Journal and find that the Price to earnings ratio for Krispy
Kreme Doughnut Company, a company you think is similar to the Big Kine Doughnut Company
to be 22. The Big Kine Doughnut Company has 50,000 shares outstanding. Based on this
information, what is the appropriate price for the Big Kine Doughnut Company?
356
Solutions to Chapter 10 Homework
Problem 1: This is a constant growth valuation problem. We are given D1 = 30, Ks = 12, and g = 10
Problem 3: This is a supernormal growth problem. However, because you are given the stock price three
years from today, part of the work has been done for you.
Calculation of Dividends
D1 = Do (1 + g) = 2(1+.05) = 2.10
D2 = D1(1 + g) = 2.10 (1 + .05) = 2.205
D3 = D2 (1 + g) = 2.205 (1 + .05 = 2.315
So we will buy the stock today, after one year we will receive a dividend of 2.10. After the second
year we will receive a dividend of 2.205. After the third year we will receive the last dividend of
2.315 plus we will sell the stock for 34.73. So at the end of the third year we will get a total of
37.045. We want to know the present value of these cash flows. We can treat this as an uneven
cash flow stream and use our calculator to calculate the present value of the cash flows as follows.
CF0 = 0
C01 = 2.10
C02 = 2.205
C03 = 37.045
NPV, I = 12, Enter, ↓, CPT NPV = 30.00, So the stock is worth $30.00 today.
Problem 4: This is a supernormal growth model. The supernormal growth will last for 4 years so we need
to calculate the amount of the dividend for the next 5 years. (calculations are rounded to the
nearest penny.
Now we must compute the price of the stock at the end of the fourth year using the constant
growth model.
So at the end of the fourth year we will get a dividend of $3.15 and the stock will be worth $47.29,
for a total of $50.44. We can draw a time line to show the cash receipts
0 1 2 3 4
|-----------|---------------|-----------------|--------------|
0 2.24 2.51 2.81 3.15
47.29
$50.44
357
Now we can use the cash flow function in our calculator to compute the present value of an
uneven cash flow stream.
CF0 = 0
C01 = 2.24
C02 = 2.51
C03 = 2.81
C04 = 50.44 NPV, I = 12, Enter, ↓, CPT NPV = $38.06, So the stock is worth $38.06
today.
Problem 5:
This is a supernormal growth model. The supernormal growth will last for 10 years, so we need to
calculate the first 11 dividends:
D1 = given = 0
D2 = given = 0
D3 = given = 0
D4 = given = 0
D5 = given = 0
D6 = given = 0
D7 = given = 0
D8 = given = $2.00
D9 = D8 (1 +g) = 2 (1.5) = 3
D10 = D9 (1 +g) = 3 (1.5) = 4.5
D11 = D10 (1 +g) = 4.5 (1.05) = 4.73
Now we must compute the price of the stock at the end of the tenth year using the constant growth
model.
So at the end of the tenth year we will get a dividend of $4.50 and the stock will be worth $31.53,
for a total of $36.03. We can draw a time line to show the cash receipts
0 1 2 3 4 5 6 7 8 9 10
|-----------|------------|-----------|----------|-----------|-----------|------------|-----------|----------|-----------|
0 0 0 0 0 0 0 0 $2.00 $3.00 $36.03
358
Now we can use the cash flow function in our calculator to compute the present value of an
uneven cash flow stream.
CF0 = 0
C01 = 0
C02 = 0
C03 = 0
C04 = 0
C05 = 0
C06 = 0
C07 = 0
C08 = $2
C09 = $3
C10 = $36.03
NPV, I = 20, Enter, ↓, CPT NPV = $6.87, So the stock is worth $6.87 per share today.
Problem 6
^
P o = EPS X PEC
= $15 X 22
^ ^
V o = P o X SO
= $330 X 50,000
= $16,500,000
359
Chapter 9: Bonds and Their
Valuation
360
Chapter 9: Bonds and their Valuation
2. Issue Bonds
361
Bond Terminology
-- The amount of time until the firm agrees to pay the face
value back to the investors.
Maturity Date
-- the specific date that the firm will pay the face value to the
investors
Issue Date: The day that the company issues the bonds (borrows
the money)
Coupon Payments
Coupon Rate
362
Call Provision
-- Call Protection
-- Call Premium
Convertible Bonds
Default Risk
-- The chance that the company that borrows the money will
not pay the investors as agreed upon.
Kreq = the required rate of return of investors for buying the bond
(making the loan)
363
Payoff to Investors From A Bond
0 1 2 3 4 10
|----------|----------|----------|----------|-----…-----|
364
Bond Valuation
Recall that The General Valuation Model tells us that the value of
any asset can be computed by calculating the present value of the
cash flows (benefits) that the asset will provide us in the future.
0 1 2 3 4 12
|----------|----------|----------|----------|-----…-----|
365
Stream of Cash Yes Limited Number Yes Equal Time Yes Equal Dollar Yes
Annuity
Flows? of Cash Flows? Spacing? Amount?
No No No No
No No
No
Combination Combination
Problem Problem
366
Computing the Present Value
CF
CF0 =0
C01 = 100 ↵ ↓
F01 = 11 ↵ ↓
C02 = 1100 ↵
NPV
I = 10 ↵ ↓
CPT NPV = ? NPV = $1,000
Notice that the current price of the bond is equal to the face value.
367
Bond Price Changes when Interest Rates
Change
Suppose we buy the bond from the previous problem: That is, we
have a $1,000 face value bond with 12 years to maturity and a 10
percent annual coupon rate. One minute after the bond was issued
interest rates, and thus investors required rates of return, go up to
14 percent. What is the value of this bond now?
CF
CF0 =0
C01 = 100 ↵ ↓
F01 = 11 ↵ ↓
C02 = 1100 ↵
NPV
I = 14 ↵ ↓
CPT NPV = ? NPV = $773.59
368
Suppose we buy the bond from the previous problem: That is, we
have a $1,000 face value bond with 12 years to maturity and a 10
percent annual coupon rate. One minute after the bond was issued
interest rates, and thus investors required rates of return, go down
to 6 percent. What is the value of this bond now?
CF
CF0 =0
C01 = 100 ↵ ↓
F01 = 11 ↵ ↓
C02 = 1100 ↵
NPV
I=6↵ ↓
CPT NPV = ? NPV = $1,335.35
369
Notes: Bond Price Changes when interest rates change
The longer a bond has until maturity, the more the price of the
bond will change when interest rates change
CF
CF0 =0
C01 = 100 ↵ ↓
F01 = 99 ↵ ↓
C02 = 1100 ↵
NPV
I=6↵ ↓
CPT NPV = ? NPV = $1,664.70
Notice, the longer the bond had to maturity the more the bond
price increased when interest rates declined. The longer the bond
had to maturity, the more the price decreased when interest rates
increased.
370
Semi-Annual Coupon Payments and Bond
Valuation
0 1 2 3 4 12
|----|-----|-----|----|----|-----|----|-----|-----…-----|
371
To solve this problem we must use the adjust the calculation
method that we learned in Chapter 8
Recall that
Ncalc = N X M = 12 X 2 = 24 (23 + 1)
I 10
Icalc = = = 5%
M 2
CF
CF0 =0
C01 = 50 ↵ ↓
F01 = 23 ↵ ↓
C02 = 1050 ↵
NPV
I=5↵ ↓
CPT NPV = ? NPV = $1,000.00
372
Suppose we are interested in a bond with a $1,000 face value, 12
years to maturity and a 10 percent annual coupon rate. Coupon
payments are made semi-annually. One minute after the bonds
were issued, interest rates, and thus investors required rates of
return go up to 12 percent annually on this investment. What is the
value of this bond now?
CF
CF0 =0
C01 = 50 ↵ ↓
F01 = 23 ↵ ↓
C02 = 1050 ↵
NPV
I=6↵ ↓
CPT NPV = ? NPV = $874.50
373
Computing the Rate Earned on a Bond
Investment
We would like to know the return that we will receive if we
purchase a bond at a certain price.
1. Yield to Maturity
374
Suppose that I am considering purchasing a bond with a
$1,000 face value. The bond has a 9% annual coupon
rate, and coupon payments are made annually. The
bond has 10 years to maturity and a current price of
$887. What interest rate will we earn if we buy the
bond and hold it until it matures?
0 1 2 3 4 10
|----------|----------|----------|----------|-----…-----|
CF0 = -887 ↵ ↓
C01 = 90 ↵ ↓
F01 = 9 ↵ ↓
C02 = 1090 ↵ ↓
IRR
CPT
IRR = 10.91 %
375
2. Yield to Call
376
Suppose that I am considering purchasing a bond with a
$1,000 face value. The bond has a 9% annual coupon
rate, and coupon payments are made annually. The
bond has 10 years to maturity and a current price of
$914. Suppose that the bond can be called after 5 years
for 103% of par or $1,030. What interest rate will we
earn if we buy the bond and hold it until it is called?
0 1 2 3 4 5
|----------|----------|----------|----------|----------|
CF0 = -914 ↵ ↓
C01 = 90 ↵ ↓
F01 = 4 ↵ ↓
C02 = 1120 ↵ ↓
IRR
CPT
IRR = 11.85 %
Usually investors will receive the lower of the Yield to Call or the
Yield to Maturity.
377
Advanced Problems
14 15 16 17 18 19 20
|----------|----------|----------|----------|----------|----------|
CF0 = -1,059 ↵ ↓
C01 = 30 ↵ ↓
F01 = 5 ↵ ↓
C02 = 1030 ↵ ↓
IRR
CPT
IRR = 1.949%
378
Suppose that you are considering purchasing a bond that has a
$1,000 face value, and a 3 percent annual coupon rate. Coupon
payments are made semiannually. When it was issued, the bond
had 15 years to maturity. It is 8 years after the bonds were issued.
The 16th coupon payment was just made. The bonds are callable
after they have been outstanding for ten years for $1050. If you
buy the bond today for $1,025, what is your yield to call?
The bond originally had 15 years to maturity. However, the first 8
years of the bond are gone, so they are irrelevant to you. The first
8 years are history. You should consider only those cash flows
that occur after the 8th year (which is today). The bond can be
called after 10 years, so there is two years remaining from today
until the bonds can be called.
8 9 10
|-------------|----------|----------|----------|
CF0 = -1,025 ↵ ↓
C01 = 15 ↵ ↓
F01 = 3 ↵ ↓
C02 = 1065 ↵ ↓
IRR, CPT
IRR = 2.055%
2.055 X 2 = 4.11
379
Chapter 9 Homework and
Solutions
380
Chapter 9 Homework: Bond Valuation Problems:
Problem 1: Suppose the Master Blaster company wishes to issue some bonds. The bonds will have a
$1,000 face value, 30 years to maturity and a 6 percent annual coupon. How much will Master
Blaster receive for each of the bonds if?
Problem 2: Now Suppose that Master Blaster bonds have an 8 percent coupon rate rather than a 6 percent
coupon rate. Re-compute your answers to parts A, B and C from problem 1.
Problem 3: Suppose that the Jerry Rigger Oil Company wishes to issue some bonds. The bonds will have
a $1,000 face value, 20 years to maturity and a 10 percent annual coupon rate. Coupon payments
will be made semi-annually. How much will the bonds sell for if
Problem 4: Suppose that Parker Ranch Company sold an issue of bonds that had an original maturity of 10
years. The bonds had a $1,000 face value and a 10 percent annual coupon rate. The bonds makes
semi-annual coupon payments.
a. Three years after the bonds were issued, (immediately after the 6th coupon payment), the
going rate of interest (the required rate of return) on bonds similar to this fell to 8 percent
annually. What price would you expect the bonds to sell for?
b. Suppose that two years after the bonds were issued (immediately after the fourth coupon
payment) the going annual rate of interest on bonds (the required rate of return) similar to this
increases to 12 percent annually. What price would you expect the bonds sell for?
Problem 5: The Pokemon Company has two bond issues outstanding. Both bonds have a 10 percent
annual coupon rate and a $1,000 face value. Bond long term has a maturity of 15 years. Bond
short term has a maturity of 1 year. Compute the value of each bond when the required rate of
return is:
a. 5 percent annually
b. 8 percent annually
c. 12 percent anually
Problem 6: The Hicory Dickory Dock Company’s bonds were issued with an original maturity of 10 years
and a $1,000 face value. The bonds have an 8 percent annual coupon rate. There are currently 5
years remaining until maturity. The fifth coupon payment was just made. What is the yield to
maturity on this bond if the current price of the bonds is:
a. $904
b. $1,144
Problem 7: The Bevis and B-head company issued bonds with an original maturity of 30 years and were
callable after being outstanding for 5 years at a price of 1,090. The face value of the bonds is
$1,000. The bonds have a 9 percent annual coupon. The bonds were issued three years ago. The
third coupon payment was just made. Today, you are considering purchasing one of the bonds at a
price of $1,175. You wish to know what rate of return you will be earning on the bonds.
a. Compute the yield to maturity for the bond.
b. Compute the yield to call for the bond.
381
Solutions to Chapter 9 Homework
Problem 1:
a. CF
CF0 =0 ↵ ↓
C01 = 60 ↵ ↓
F01 = 29 ↵ ↓
C02 = 1060 ↵
b. CF
CF0 =0 ↵ ↓
C01 = 60 ↵ ↓
F01 = 29 ↵ ↓
C02 = 1060 ↵
c. CF
CF0 =0 ↵ ↓
C01 = 60 ↵ ↓
F01 = 29 ↵ ↓
C02 = 1,060 ↵
NPV
I=7↵ ↓, CPT NPV = ? NPV = $875.91
Problem 2:
a. CF
CF0 =0 ↵ ↓
C01 = 80 ↵ ↓
F01 = 29 ↵ ↓
C02 = 1,080 ↵
b. CF
CF0 =0 ↵ ↓
C01 = 80 ↵ ↓
F01 = 29 ↵ ↓
C02 = 1,080 ↵
c. CF
CF0 =0 ↵ ↓
C01 = 80 ↵ ↓
F01 = 29 ↵ ↓
C02 = 1080 ↵
382
Problem 3:
a. CF
CF0 =0 ↵ ↓
C01 = 50 ↵ ↓
F01 = 39 ↵ ↓
C02 = 1050 ↵
NPV
I=6↵ ↓
CPT NPV = ? NPV = $849.54
b.
CF
CF0 =0 ↵ ↓
C01 = 50 ↵ ↓
F01 = 39 ↵ ↓
C02 = 1050 ↵
NPV
I=5↵ ↓
CPT NPV = ? NPV = $1,000
c.
CF
CF0 =0 ↵ ↓
C01 = 50 ↵ ↓
F01 = 39 ↵ ↓
C02 = 1050 ↵
NPV
I=4↵ ↓
CPT NPV = ? NPV = $1,197.93
383
Problem 4:
The annual coupon payment can be computed as the par value * the coupon rate = 1000 * .1 =
100. However in this case we will get half of the payment 100/2 = 50 every 6 months.
0 1 2 3 …. 7 8 9 10
|-----|------|-------|-------|------|-------|------|------|------|------|------|------|------|------|
50 50 50 50 50 50 50 50 50 50 50 50 50 50
1,000
a. The timeline for the bond after three years have passed:
3 … 7 8 9 10
|-------|-- ---|------|------|------|------|------|------|
50 50 50 50 50 50 50 50
? 1,000
CF
CF0 =0 ↵ ↓
C01 = 50 ↵ ↓
F01 = 13 ↵ ↓
C02 = 1050 ↵
2 3 …. 7 8 9 10
|------|-------|------|------|------|------|------|------|------|------|
50 50 50 50 50 50 50 50 50 50
? 1,000
CF
CF0 =0 ↵ ↓
C01 = 50 ↵ ↓
F01 = 15 ↵ ↓
C02 = 1050 ↵
384
Problem 5:
0 1
|--------|
1,000
100
0 1 … 5 6 7 8 9 … 15
|--------|--------|--------|--------|--------|--------|--------|
1,000
? 100 100 100 100 100 100 100
a. CF CF
CF0 = 0 ↵ ↓ CF0 = 0 ↵ ↓
C01 = 1100 ↵ ↓ C01 = 100 ↵ ↓
F01 = 14 ↵ ↓
C02 = 1100 ↵ ↓
NPV NPV
I=5↵ ↓ I=5↵ ↓
CPT NPV = ? NPV = $1,047.62 CPT NPV = ? NPV = $1,518.98
b. CF CF
CF0 = 0 ↵ ↓ CF0 = 0
C01 = 1100 ↵ ↓ C01=100 ↵ ↓
F01 = 14 ↵ ↓
C02 = 1100 ↵ ↓
NPV NPV
I=8↵ ↓ I=8↵ ↓
CPT NPV = ? NPV = $1,018.52 CPT NPV = ? NPV = $1,171.19
c. CF CF
CF0 = 0 ↵ ↓ CF0 = 0 ↵ ↓
C01 = 1100 ↵ ↓ C01=100 ↵ ↓
F01 = 14 ↵ ↓
C02 = 1100↵ ↓
NPV NPV
I = 12 ↵ ↓ I = 12↵ ↓
CPT NPV = ? NPV = $982.14 CPT NPV = ? NPV = $863.78
385
Problem 6:
0 1 2 3 4 5 6 7 8 9 10
|-------|--------|--------|-------|-------|-------|--------|--------|--------|--------|
? 80 80 80 80 80 80 80 80 80 80
1,000
Timeline for the bond after it has been outstanding for 5 years.
5 6 7 8 9 10
|-------|--------|--------|--------|--------|
? 80 80 80 80 80
1,000
a. CF0 = -904 ↵ ↓
C01 = 80 ↵ ↓
F01 = 4 ↵ ↓
C02 = 1080 ↵ ↓
IRR
CPT
IRR = 10.57 %
b. CF0 = -1144 ↵ ↓
C01 = 80 ↵ ↓
F01 = 4 ↵ ↓
C02 = 1080 ↵ ↓
IRR
CPT
IRR = 4.70 %
386
Problem 7:
0 1 2 3 4 5 6 7 8 9 … 30
|-------|--------|--------|-------|-------|-------|--------|--------|--------|--------|
? 90 90 90 90 90 90 90 90 90 90
1,000
There are 27 years remaining until the bond matures and two years remaining until the call date. The bond
was originally a 30 year maturity/5 year callable bond, however, three years have passed since the bond
was issued.
3 4 5 6 7 8 9 … 30
|-------|-------|-------|--------|--------|--------|--------|
90 90 90 90 90 90 90
-1,175 1,000
CF0 = -1,175 ↵ ↓
C01 = 90 ↵ ↓
F01 = 26 ↵ ↓
C02 = 1090 ↵ ↓
IRR
CPT
IRR = 7.47 %
b. Timeline if the bonds are called at the end of the 5th year. Notice there are two years remaining on the
bond until it can be called.
3 4 5
|-------|-------|
90 90
-1,175 1,090
CF0 = -1,175 ↵ ↓
C01 = 90 ↵ ↓
F01 = 1 ↵ ↓
C02 = 1180 ↵ ↓
IRR
CPT
IRR = 4.12 %
387
CHAPTER 14:
Cash Flow Estimation
388
Capital Budgeting:
The process of evaluating projects for your company. If you have observed
businesspersons calculating numbers on a napkin in Kens, they are probably
doing capital budgeting. Unfortunately, they are usually doing it incorrectly.
Today we will show you how to do it correctly.
Four Steps
1. Get an Idea
2. Gather information
389
Step 1:
B. Talk to Friends
D. Entrepreneurship Magazines
390
Step 2:
a. You need good information. The analysis is a garbage in -
garbage out type of analysis.
a. Start-up costs
b. Operating Cash Flows
i. revenues
ii. expenses
c. Terminal cash flows
i. potential suppliers
ii. the internet
iii. government provided information (SBDC)
iv. ask the competition (be prepared for a limited
response
Some of the cash flows you have identified may not be relevant for
Capital Budgeting Purposes.
Sunk Costs should not be considered. Sunk costs are those costs
that have already been expended, and cannot be recovered if you
decide not to do the project.
391
Information for the Project:
Step 3: You must Compute three different Cash Flows for the Project
Part 1: Initial Cash Flow: The cash flows associated with starting
your project
Part 2: Operating Cash Flows: The year-to-year cash flows from the
project
Part 3: Terminal Cash Flows: The cash flows associated with closing
your project
392
Annual Cash Flows for the Project
Year 0 1 2 3 4
Total Revenues $402,500 $402,500 $402,500 $402,500
- Cost of Goods Sold $201,250 $201,250 $201,250 $201,250
- Sales and Administrative Costs $80,500 $80,500 $80,500 $80,500
- Depreciation $108,900 $148,500 $49,500 $23,100
= Earnings before Interest & Tax (EBIT) $11,850 -$27,750 $71,250 $97,650
- Interest $0 $0 $0 $0
= Earnings Before Taxes (EBT) $11,850 -$27,750 $71,250 $97,650
- Tax (@20 percent) $2,370 -$5,550 $14,250 $19,530
= Net Income $9,480 -$22,200 $57,000 $78,120
+ Depreciation $108,900 $148,500 $49,500 $23,100
=Net Operating Cash Flows $118,380 $126,300 $106,500 $101,220
393
Part 1: Initial Cash Flows
394
Part 2: Operating Cash Flow
A. Depreciation Computations
When we buy a machine, we spend money. The money is spent
today, however; the machine will last for more than one year.
So we calculate the part of the machine that wore out this year.
That part of the machine that wore out this year is an expense
and is called depreciation.
Semi-Trucks 3 Years
Automobiles 5 Years
Office Furniture 7 Years
Residential Real Estate 27.5 Years
Commercial Real Estate 39 Years
395
Method I: Straight Line Depreciation. The concept behind
straight line depreciation is that our machine will wear out by
the same amount each year.
DepreciableBase − SalvageValue
AnnualDepreciation =
DepreciableLife
$330,000 − 35,000
AnnualDepreciation =
3
AnnualDepreciation = $98,333.33
396
Method II: MACRS Depreciation
397
Terminal Cash Flows:
1. Recovery of Inventory
2. Recovery of Accounts Receivable
3. Recovery of Working Cash
4. Payment of outstanding accounts
5. Sale of Equipment
6. Tax on Sale of Equipment
5. Various other terminal cash flows might occur.
398
Tax on Sale of Equipment
Year 1 $108,900
Year 2 $148,500
Year 3 $49,500
Year 4 $23,100
Depreciation Taken $330,000
399
Terminal Cash Flow
Methods for determining if the cash flows are sufficient to warrant starting
the business or the project:
400
Chapter 14: Homework and
Solutions
401
Chapter 14 Homework
1. You are planning on purchasing a new sewing machine for your business. The sewing
machine costs $10,000 and is expected to have a $1,000 salvage value. The
sewing machine is classified as three year MACRS property. How much is the
depreciation for each of the first four years?
2. Use the same information as from the above problem, but you have decided to use
straight line depreciation instead of MACRS. The machine will be depreciated
over three years. What is the depreciation for each of the three years?
3. On August 15, 2010 you purchased a machine that cost $15,000. You have been
using the MACRS (3 year life) depreciation on the machine and are in the 34
percent tax bracket. On August 15, 2012 you decide to sell the machine. You sell
the machine for $12,000. You will take the regular 2012 depreciation in your
operating cash flow in the normal fashion. How much in taxes will you have to
pay (or will recover) in 2012 because you sold the machine?
4. You are trying to determine how much working capital you will need for your new
salsa factory. You expect to buy most of your supplies on credit, so you expect to
owe an average of $10,000 to your suppliers at any given time. You expect to
have about 25,000 of inventory on hand at any given time. You will also sell
some of the salsa on credit. You expect that your customers will owe you
$15,000 at any given time. You will need to have $2,000 available for use in the
cash registers at the retail outlet. Compute the net working capital requirements
for this company.
5. You are considering starting a new doughnut shop in the Prince Kuhio Mall. Based
on your sales estimate, you believe that you will be able to sell 220,000 doughnuts
per year at a price of $1.10 per doughnut. You have estimated labor costs to be
$50,000 for the year and it will cost $48,000 per year to rent the space in the Mall
(rent will be paid at the end of each year). The cost of the ingredients to make the
doughnuts will be $.25 per doughnut. You will need to have $10,000 of inventory
and you will need to have $3,000 in cash for the cash registers. Inventory and
cash for the registers will need to be on hand the day you start the business. You
will have to purchase $50,000 of equipment with cash you have saved to
manufacture the doughnuts. This equipment is classified as three year MACRS
property. You have organized your company as a sole proprietorship, so you will
pay 34 percent taxes on the earnings of the firm. You are planning to quit the
business at the end of the third year, at which time you will sell the equipment for
$25,000. When you close the business, you will recover the original inventory
investment & the cash register cash. Create a cash flow schedule for the above
project. You require a 12 percent return on this type of investment.
402
6. You wish to evaluate the possible purchase of a gas chromatograph and total
hydrocarbon analyzer for your geology company. The equipment’s base price is
$40,000, and it would cost another $10,000 to modify it for special use by your
firm. The equipment qualifies for three-year MACRS depreciation, which is what
you will use for your capital budgeting analysis. The equipment, as modified,
would be sold after three years for $20,000. In order to operate the equipment,
you would need to increase the firm’s net working capital (in the form of spare
parts and inventory) by $4,000. The purchase of the equipment would have no
effect on the firm’s revenues. However, it would reduce the expenses of the firm
by $12,000 per year as the firm would no longer need to contract certain work
out. When you sell the equipment, you will recover the $4,000 of net working
capital. The firm’s marginal tax rate is 20 percent. Create a cash flow schedule
for the above project.
403
Chapter 14 Homework Solutions
Question 1
Cost 10000
3 year depreciable life
MACRS
Year Allowance Depreciation
1 0.33 3300
2 0.45 4500
3 0.15 1500
4 0.07 700
Question 2
= (10000-1000)/3
= 3000
Year Depreciaiton
1 3000
2 3000
3 3000
4 0
404
Question 3
MACRS
Year Year Allowance Depreciation
2010 1 0.33 $4,950
2011 2 0.45 $6,750
2012 3 0.15 $2,250
2013 4 0.07 $1,050
Cost $15,000
- Depreciation Taken $13,950
= Book Value $1,050
Question 4
405
Question 5
Year 0 1 2 3
Total Revenues $242,000 $242,000 $242,000
Ingredients $55,000 $55,000 $55,000
Labor $50,000 $50,000 $50,000
Rent $48,000 $48,000 $48,000
Depreciation $16,500 $22,500 $7,500
Earnings before Interest and Taxes
(EBIT) $72,500 $66,500 $81,500
Interest $0 $0 $0
Earnings Before Taxes (EBT) $72,500 $66,500 $81,500
Tax (@34 percent) $24,650 $22,610 $27,710
Net Income $47,850 $43,890 $53,790
+ Depreciation $16,500 $22,500 $7,500
Net Operating Cash Flows $64,350 $66,390 $61,290
406
Question 5 Continued
Depreciation Computations
Depreciable
Base $50,000
MACRS
Year ALLOW Depreciation
1 0.33 $16,500
2 0.45 $22,500
3 0.15 $7,500
4 0.07 $3,500
Cost $50,000
- Depreciation Taken $46,500
= Book Value $3,500
407
Question 6 Cash Flows for the
Project
Year 0 1 2 3
Total Revenues $0 $0 $0
Operating Expenses -$12,000 -$12,000 -$12,000
Depreciation $16,500 $22,500 $7,500
Earnings before Interest and Taxes
(EBIT) -$4,500 -$10,500 $4,500
Interest $0 $0 $0
Earnings Before Taxes (EBT) -$4,500 -$10,500 $4,500
Tax (@20 percent) -$900 -$2,100 $900
Net Income -$3,600 -$8,400 $3,600
+ Depreciation $16,500 $22,500 $7,500
Net Operating Cash Flows $12,900 $14,100 $11,100
408
Question 6 Continued
Depreciation Computations
Depreciable
Base $50,000
MACRS
Year ALLOW Depreciation
1 0.33 $16,500
2 0.45 $22,500
3 0.15 $7,500
4 0.07 $3,500
Cost $50,000
- Depreciation Taken $46,500
= Book Value $3,500
409
CHAPTER 13:
410
The Required Rate of Return
Where:
Wst is the percentage of funds we will obtain from stock
Wd is the percentage of our funds that we will borrow
Kreq is the cost of obtaining money from stock issues (the required rate of
return)
Kd is the before tax cost of borrowing money
T is the firms tax rate
Suppose that our company will obtain 75 percent of needed funds from stock
issues. We will obtain 25 percent of our needed funds by borrowing money.
Suppose further that the cost of obtaining money from stocks is 12 percent
and the before tax cost of obtaining money by borrowing is 6.67 percent.
The firms tax rate is 40 percent.
= 9+1
= 10
411
Where do you get the parameters for computing the WACC
This is the rate that the bank will charge you on a bank loan
This is the tax rate of the firm or the individual. You can
estimate this from tables provided by the IRS, based on the income of
the company or the individual.
Suppose that the current risk free rate of interest (CD rate) is 3
percent. Suppose the additional return required to make this
investment rather than a risk free investment is 4 percent.
Then Kreq = 3 +4 = 7
In this Kreq is equal to the rate of return you could receive if you
invested in some alternate project, maybe the stock market. The
stock market has historically produced about a 12 percent return.
Thus you might take the return on the stock market as Kreq.
i. liquidity premium
ii. risk premium
412
C. CAPM method
The CAPM argues that the appropriate required rate of return can
be computed as:
Kreq=Krf+B(Km-Krf)
Suppose that the return on the market is 12 percent and the risk
free rate of interest is 3 percent. Suppose that the project you are
considering is about as risky as the stock of the Pride Petroleum
Company. Pride Petroleum company has a Beta of 1.15. What is
the required rate of return
Kreq=Krf+B(Km-Krf)
= 3 + 1.15(12-3) = 13.35%
413
Classifications of Projects
Mutually Exclusive:
Accepting one project implies that you will not accept another project
Independent:
You might accept both projects.
Normal Project
You have only one cash flow sign change
0 -$10,000
1 $5,000
2 $7,000
3 $9,000
Non-Normal Project
The project has more than one cash flow sign change
0 -$10,000
1 $5,000
2 $7,000
3 $9,000
4 -$3,000
414
Payback Period:
Tells us the number of years it will take us to get our money back.
Decision Rule:
a. For independent Projects: The decision rule is not defined
b. For Mutually Exclusive Projects: Accept the Project with the
lowest payback period
Strengths:
1. Easy to Compute
2. Easy to Understand
3. Always Gives one answer
Weakness:
415
Computations:
− LastNegCumCashFlow
The Payback Period is PB = BaseYear +
FollowingCashFlow
− (−23,820)
PB = 3 +
194,220
23,820
PB = 3 +
194,220
PB = 3.123Years
416
Significance of Ignoring the Time Value of Money
Period A B
0 -10,000 -10,000
1 5,000 2,000
2 2,000 5,000
3 3,000 3,000
417
Significance of ignoring Cash Flows that occur after the payback period
Period A B
0 -10,000 -10,000
1 5,000 5,000
2 6,000 5,000
3 1,000,000
418
Internal Rate of Return:
The interest rate that equates all future cash flows to the cost of
starting the project.
Decision Rule:
Strengths:
Weaknesses:
419
Computations: Assume that the Cost of Obtaining Money for the Project is
10 percent.
CF0 = -$375,000
C01 = $118,380
C0 2 = $126,300
C0 3 = $106,500
C0 4 = $194,220
420
Net Present Value
Computes the Present Value of All Cash Flows from the Project and
Compares them to the cost of the Project
Decision Rule:
Strengths:
Weaknesses:
1. Gives misleading answers when projects have different sizes.
2. Gives misleading answers when projects have different lives.
3. Requires a Financial Calculator
4. Assumes that we can reinvest the funds we get back from the
project at the required rate of return.
421
Computations: Assume that the Cost of Obtaining Money for the Project is
10 percent.
CF0 = -$375,000
C01 = $118,380
C0 2 = $126,300
C0 3 = $106,500
C0 4 = $194,220
NPV I = 10
NPV = $49,668.25
Because the Net Present Value is Above $0, we should accept the project.
422
Significance of Misleading answers
423
Profitability Index.
PI = NPV
Cost of Starting Project
424
What to do when projects have unequal lives
Sometimes projects have unequal lives. That is: If you buy the good
machine you can expect it to last 6 years. If you buy the inexpensive
machine, you should expect it to last 3 years.
Suppose that we have the following cash flows. Further suppose that we
have a 11 percent WACC
Year Cash Flow___
A B
0 -$40,000 -$80,000
1 $26,000 $29,000
2 $10,000 $16,000
3 $28,000 $15,000
4 $24,000
5 $10,000
6 $52,000
A B
NPV $12,013.01 $19,625.34
IRR 27.59% 18.55%
As in projects with differing sizes the NPV will be misleading in such cases.
In addition, the Profitability Index may be misleading. So we must adjust
our calculations.
What to do?
In this case, you find the lowest common denominator for the
two projects. If we have a project with a 6-year life and another
with a 3-year life, the lowest common denominator is 6 (two
three year projects makes one 6-year project)
425
Year Cash Flow___
A B
0 -$40,000 -$80,000
1 $26,000 $29,000
2 $10,000 $16,000
3 $28,000 -$40,000 $15,000
4 $26,000 $24,000
5 $10,000 $10,000
6 $28,000 $52,000
A with repetition B
NPV $20,796.81 $19.625.34
IRR 27.59% 18.55%
426
Chapter 13 Homework and Solutions
427
Chapter 13 Homework
Problem 1: We are considering two projects. The first project is a restaurant. The restaurant will cost
$10,000 to start. The restaurant will provide a return of $3,000 per year each year for the next
5 years. The second project is a tourism bus. The bus will cost $25,000 to start. The bus will
provide a return of $7,400 per year, each year for the next five years. The cost of capital for the
company is 12 percent. Because your funds are limited, you feel that it is feasible to undertake
only one of the projects.
Problem 2:
You are considering buying a delivery truck. The truck you are considering buying costs $52,125 to buy
and will return $12,000 per year for 8 years. The cost of capital for your company is 12 percent.
Problem 3:
The CR Taxi Co is considering two mutually exclusive investments. The projects expected cash flows are
as follows:
Problem 4: Suppose you are considering investing in the following two projects. The projects are mutually
exclusive. Use the replacement chain approach that we discussed in class to determine which
project should be accepted. The cost of capital for the project is 16%
428
Problem 5: Suppose that our company will get 30 percent of the funds we need from stock issues and
70 percent of the funds we need from borrowing. The firms tax rate is 40 percent. The cost of
obtaining money from stocks is 10 percent, and the cost of obtaining money from borrowing is 5
percent. What is the weighted average cost of capital for our company?
429
Solutions to Chapter 13 Homework:
Problem 1:
A.
Restaurant
Year Cash flow cumulative cash flow
00 (10,000) (10,000)
01 3000 (7,000)
02 3000 (4,000)
03 3000 (1,000)
04 3000 2,000
05 3000 5,000
Tourism Van
B Restaurant
First, you must enter the cash flows for the problem into the calculator
Now hit the IRR button then the CPT button to get the internal rate of return of 15.238 percent
Tourism Van
CF0 = 25000 +- ↵ ↓
C01 = 7400 ↵ ↓↓
C02 = 7400 ↵ ↓↓
C03 = 7400 ↵ ↓↓
C04 = 7400 ↵ ↓↓
C05 = 7400 ↵ ↓↓
430
C. Restaurant
You must enter the cash flows into the calculator in the same manner as you did for part B (if they
are already entered, you do not need to re-enter them)
Now hit the NPV button and input I = 12 and hit the enter button ↵ followed by the arrow down
button ↓, followed by the CPT button to get the net present value of $814.33.
Tourism Van
NPV = $1,675.34
Notice that the various methods give us conflicting results the payback period, the discounted payback
period and the internal rate of return suggest that the restaurant is a better project. The net present value
suggests that the tourism van is a better project. The Internal Rate of Return suggests that the restaurant is
a better project. However we must recognize that the NPV is somewhat distorted in this example because
of the different sizes of the projects. When there is a large difference in the scale of the projects, I
recommend following the suggestion provided by the internal rate of return. So we should start the
restaurant.
Problem 2:
B. IRR = 16.00
C. NPV = 7486.68
431
Problem 3
Notice that project B is a better project when the cost of capital is low. Project A is a better project
when the cost of capital is high.
4. We begin by assuming that we can repeat project Short, four years from today, giving us the following
cash flows
Combining Project short with the repeated project short gives us:
Note that if you have the BAII+ Professional, the IRR on project short will come out to 37.29 percent. If
you have the BAII+, you will get an error.
5. The weighted average cost of capital is given by: WACC = Wst(Kreq) + Wd(Kd)(1-T).
432
Chapter 11: Capital Budgeting when
the Cost of Capital is Not Fixed
433
Capital Budgeting when the Cost of Capital is Not Fixed
So far we have assumed that the cost of capital is fixed regardless
of how much money our company uses for its projects.
When the Cost of Capital varies, our capital budgeting procedures must be
modified.
Example #1:
Suppose that our company has the following projects that it is considering.
Suppose also that our company can obtain funds as follows: (Marginal Cost
of Capital schedule)
434
Steps in Finding the Solution:
Step 4: Select projects where the return on the project is higher than the
marginal cost of obtaining funds for the project
435
Step 1: Create an Opportunity Graph
IRR
P1
20
P2
18
P3
16
15 P4
Total
Funds
$100,000 $400,000 $600,000 $750,000 Used
WACC
20
F-C
18
16
15
F-B
11 F-A
10
Total
Funds
Raise
$100,000 $400,000 $600,000 $750,000
436
Step 3: Combine the two graphs
IRR &
WACC P1
20
F-C
P2
18
P3
16
15 P4
F-B
11 F-A
10
Total
Funds
Raised
$100,000 $400,000 $600,000 $750,000 & Used
Step 4: Select projects where the return on the project is higher than the
marginal cost of obtaining funds for the project.
437
Example #2:
Suppose that our company has the following projects that it is considering.
Suppose also that our company can obtain funds as follows: (Marginal Cost
of Capital schedule)
We should accept projects 1 and 2. We should not accept project #4. In this
case, it is not clear if we should accept Project #3 or not. How do we
determine if we should accept project 3?
438
Step 1: Create an Opportunity Graph
IRR
P1
20
P2
18
16
15
P3
P4
10
Total
Funds
$100,000 $450,000 $650,000 $800,000 Used
WACC
20
F-C
18
16
15
F-B
11 F-A
10
Total
Funds
Raised
$100,000 $500,000 $700,000
439
Step 3: Combine the two graphs
IRR &
WACC P1
20
F-C
P2
18
16
15
F-B
11 F-A
10 P3
Total
P4
Funds
Raised
$100 $450 $500 $650 $700 $800 & Used
Step 4: Select projects where the return on the project is higher than the
marginal cost of obtaining funds for the project.
440
To determine if we should accept project 3, we must compute the Decision
Criteria as follows:
Where
DC = Decision Criteria
$LC = The number of dollars to be invested in the project that will be
financed by low cost funds
$HC = The number of dollars to be invested in the project that will be
financed by high cost funds
IRR = the Internal Rate of Return on the project in question
WACCL = the WACC when low cost funds are used
WACCH = the WACC when the high cost funds are used
Decision Rule:
If DC is greater than or equal to 0, you should accept the project.
If DC is less than 0, you should reject the project.
DC = 50,000(.105-.10) + 150,000(.105-.11)
= 250 + (-750)
= -500
441
There are seven steps in completing the Marginal Cost of Capital Graph.
Example #3:
Suppose that our optimal capital structure is 40 percent debt and 60 percent
equity. Our company wants to be sure to maintain this mix of financing. We
have contacted our bank and they have told us that we can borrow $100,000
at an interest rate of 10 percent. We can borrow any amount of additional
money at an interest rate of 11 percent. Our company has $10,000 of
retained earnings. The current price of the company’s stock is $10 per share.
Next years dividend is expected to be $0.75 per share. This dividend is
expected to grow at a 10 percent rate forever. If we issue new common
stock, the float cost will be $2.00 per share. We can issue an unlimited
amount of new common stock at this price. The firms tax rate is 30 percent
442
Step 1: Computing the Break Points
A break point is the total amount of capital raised when a given source of
financing runs out.
443
Computing Break Points (Note in this example there are two break points,
other examples may have more or less than two break points)
Recall that there are two sources of debt for our company.
Also recall that we will obtain 40 percent of our money from debt.
There are two sources of debt for our company, so we must compute a break
point for debt.
If we obtain 40 percent of our money from debt and 60 percent of our money
from equity. We will have a total of $250,000 when we must begin to use
the expensive debt.
Alternative Interpretation: If we need more than $250,000 for our projects,
we will need to borrow some money at 11 percent interest.
444
We must also compute a break point for equity. There are two sources of
equity for our company.
Recall that our company wants to obtain 60 percent of its funds from equity
445
Step 2: Starting a Marginal Cost of Capital Schedule:
(note: always start from the lowest cost sources of funds and go to the higher
cost sources of funds)
(note: there will always be one more level of funding than there was Break
Points)
446
Step 3: Computing the Component Costs of Capital
Part #1. For Debt – the component cost of debt will be given to you
as the interest rate the bank will charge you on a loan.
Po = D1/(kreq-g)
Kreq = (D1/Po) + g
Kreq = (D1/(Po-F)) + g
447
Step 4: Summary of Component Costs of Capital
This is a summary of the information computed in Step 3
448
Step 5: Computing the WACC
You must compute a WACC for each level of funding that you identified in
your Cost of Capital Schedule. So for our example we must compute the
WACC 3 times.
Where:
You need to compute a WACC for each level of funding that you identified
in your Cost of Capital Schedule.
449
A. When we need from 0 to $16,666.67, we will obtain our funds from
retained earnings and the low cost debt.
= 10.5 + 2.8
= 13.3 percent
= 11.625 + 2.8
= 14.425 percent
C. When we need more than $250,000 we will obtain funds from New
Common Stock and High Cost Debt
= 11.625 + 3.08
= 14.705
450
Step 6: Complete the Marginal Cost of Capital Schedule
20
18
16 F-C
15 F-B
11 F-A
10
Total
Funds
Raised
$16.7 $250
451
Create an Opportunity Graph
IRR
P1
20
P2
18
P3
16
15 P4
Total
Funds
Used
$100,000 $400,000 $600,000 $750,000
P3
16
P4
15
F-B
F-C
11
F-A
Total
10 Funds
Raised
16.7 100 250 400 600 750 & Used
452
Chapter 11 Homework
453
Homework For Chapter 11
Problem 1:
Suppose that our company has the following projects that it is considering.
Suppose also that our company can obtain funds for our company (Marginal
Cost of Capital schedule)
454
Problem #2
Suppose that our company has the following projects that it is considering.
Project 1
Year CF
0 -$100,000
1 $50,000
2 $95,000
Project 2
Year CF
0 -$150,000
1 $75,000
2 $120,000
3 $90,000
Project 3
Year CF
0 -$300,000
1 $100,000
2 $500,000
Project 4:
Year CF
0 $-200,000
1 $100,000
2 $200,000
3 $300,000
Suppose also that our company can obtain funds for our company (Marginal
Cost of Capital schedule)
455
Which projects should our company accept?
Problem #3:
Suppose that our optimal capital structure is 30 percent debt and 70 percent
equity. Our company wants to be sure to maintain this mix of financing. We
have contacted our bank and they have told us that we can borrow $200,000
at an interest rate of 10 percent. We can borrow any amount of additional
money at an interest rate of 11 percent. Our company has $100,000 of
retained earnings. The current price of the companies stock is $15 per share.
Next years dividend is expected to be $1.10. This dividend is expected to
grow at a 11 percent rate forever. If we issue new common stock, the float
cost will be $2.00 per share. We can issue an unlimited amount of new
common stock at this price. The firms tax rate is 40 percent. Which projects
should the firm undertake
456
Problem #4
Suppose that our optimal capital structure is 20 percent debt and 80 percent
equity. Our company wants to be sure to maintain this mix of financing. We
have contacted our bank and they have told us that we can borrow $200,000
at an interest rate of 10 percent. We can borrow an additional $300,000 at
an 11 percent interest rate. We can borrow additional money at an interest
rate of 12 percent. Our company has $100,000 of retained earnings. The
current price of the companies stock is $15 per share. Next years dividend is
expected to be $1.10. This dividend is expected to grow at a 11 percent rate
forever. If we issue $150,000 of new common stock, the float cost will be
$2.00 per share. If we issue more than $150,000 of new common stock, the
float cost will be $3.00 per share We can issue an unlimited amount of new
common stock with the $3.00 float. The firms tax rate is 40 percent. Which
projects should the firm undertake?
457
Problem #5
Suppose that our company has the following projects that it is considering.
Suppose also that our company can obtain funds for our company (Marginal
Cost of Capital schedule)
458
Solution to Chapter 11 Homework
18
16 P2
14
P3
10
Total
Funds
Used
500 1,000 1,400
25 F-D
18
16
15
F-C
F-B
11 F-A
10
Total
Funds
400 700 1,000 Raised
459
Problem 1: Combined graphs
18
P2
16
15
F-B
F-C P3
11 F-A
10
Total
Funds
Raised
400 500 700 1,000 1,400 & Used
460
Problem #2
In order to solve this problem, we must first compute the internal rate of
return for each of the projects
For Project 1
CF0 = $100,000 +- ↵ ↓
C01 = $50,000 ↵ ↓↓
C02 = $95,000 ↵ ↓↓
Now press the IRR button followed by the compute button on your
calculator
461
Problem 2 Combined Graph
IRR & WACC
P4
65
P3
46 F-C
F-D
38
P2
25
F-B P1
11 F-A
10
Total
Funds
Raised &
200 400 500 650 750 Used
462
Problem 3 Solution
Also recall that we will obtain 30 percent of our money from debt.
There are two sources of debt for our company, so we must compute a break
point for debt.
We must also compute a break point for equity. There are two sources of
equity for our company.
Recall that our company wants to obtain 70 percent of its funds from equity
463
Step 2: Starting a Marginal Cost of Capital Schedule:
(note: always start from the lowest cost sources of funds and go to the higher
cost sources of funds)
(note: there will always be one more level of funding than there was Break
Points)
Kreq = (D1/Po) + g
Kreq = (D1/(Po-F)) + g
464
= 0.1946 or 19.46 percent
Step 4: Summary of Component Costs of Capital
This is a summary of the information computed in Step 3
You must compute a WACC for each level of funding. For this problem we
must compute the WACC 3 times. (Because there are two break points).
= 12.831 + 1.8
= 14.631 percent
= 13.622 + 1.8
= 15.422 percent
465
C. When we need more than $666,666.67 we will obtain funds
from New Common Stock and High Cost Debt
= 13.622 + 1.98
= 15.602
20
P2
18
16 P3
15 F-C
F-B Total
14 P4 Funds
F-A Raised
& Used
142 300 667 800 1,200 1,400
466
Problem 4 Solution
There are three sources of debt for our company, so we must compute two
break point for debt.
Notice that when there are multiple break points from one source of
financing, we must include both amounts in computing the second break
point.
We must also compute two break points for equity. There are three sources
of equity for our company.
Recall that our company wants to obtain 80 percent of its funds from equity.
467
Break Point when New Common Stock with a $2.00 float cost runs
out.
(note: there will always be one more level of funding than there was Break
Points)
Kreq = (D1/Po) + g
468
C. The Cost of New Common Stock Issues
Kreq = (D1/(Po-F)) + g
Kreq = (D1/(Po-F)) + g
469
Step 5: Computing the WACC
You must compute a WACC for each level of funding. For this problem we
must compute the WACC 5 times.
= 14.66 + 1.2
= 15.86 percent
= 15.57 + 1.2
= 16.77 percent
470
= 16.14 + 1.2
= 17.34 percent
= 16.14 + 1.32
= 17.46 percent
= 16.14 + 1.44
= 17.58 percent
471
Problem 4 Combined Graph
IRR & WACC
P1
30
P2
25
P3 F-E
20
F-C
F-B
F-D P4
15
F-A
Total
Funds
Raised
& Used
125 500 1,000 1,200 1,800 2,500 2,600
472
Problem 5 Solution
16
15
P3
11 F-A P4
10
Total
Funds
Raised
$100 $450 $600 $650 $800 & Used
= 1,600,000 + (-150,000)
= $1,450,000
473